Mental Health Unit I: questions from Quizlet and NCLEX questions from Online Resources

अब Quizwiz के साथ अपने होमवर्क और परीक्षाओं को एस करें!

According to the DSM-V, which statement made by an adult client supports the criteria for generalized anxiety disorder? Select all that apply. A. I've been really anxious for at least 2 years now. B. My anxiety has to be genetic; my mom was a terrible worrier too. C. My marriage is in trouble because I'm always so irritable. D. I've had a good physical and my health care provider says I'm in good health. E. Its hard falling asleep and even harder staying asleep; I'm restless all night.

A,C,D,E The DSM-V criteria for generalized anxiety disorder include excessive anxiety and worry (apprehensive expectation), occurring more days than not for at least 6 months; sleep disturbance (difficulty falling or staying asleep, or restless, unsatisfying sleep) and irritability; the anxiety, worry, or physical symptoms cause clinically significant distress or impairment in social, occupational, or other important areas of functioning; the disturbance is not attributable to the physiological effects of a substance (e.g., a drug of abuse, a medication) or another medical condition. Family history is not a recognized criterion for generalized anxiety disorder.

4. A patient says, Please dont share information about me with the other people. How should the nurse respond? a. I will not share information with your family or friends without your permission, but I share information about you with other staff. b. A therapeutic relationship is just between the nurse and the patient. It is up to you to tell others what you want them to know. c. It depends on what you choose to tell me. I will be glad to disclose at the end of each session what I will report to others. d. I cannot tell anyone about you. It will be as though I am talking about my own problems, and we can help each other by keeping it between us.

ANS: A A patient has the right to know with whom the nurse will share information and that confidentiality will be protected. Although the relationship is primarily between the nurse and patient, other staff needs to know pertinent data. The other incorrect responses promote incomplete disclosure on the part of the patient, require daily renegotiation of an issue that should be resolved as the nurse-patient contract is established, and suggest mutual problem solving. The relationship must be patient centered.

2. Which action by a psychiatric nurse best supports a patient's right to be treated with dignity and respect? a. Consistently addressing a patient by title and surname. b. Strongly encouraging a patient to participate in the unit milieu. c. Discussing a patient's condition with another health care provider in the elevator. d. Informing a treatment team that a patient is too drowsy to participate in care planning.

ANS: A A simple way of showing respect is to address the patient by title and surname rather than assuming that the patient would wish to be called by his or her first name. Discussing a patient's condition with a health care provider in the elevator violates confidentiality. Informing a treatment team that the patient is too drowsy to participate in care planning violates patient autonomy. Encouraging a patient to participate in the unit milieu exemplifies beneficence and fidelity.

29. A nurse makes an initial visit to a homebound patient diagnosed with a serious mental illness. A family member offers the nurse a cup of coffee. Select the nurses best response. a. Thank you. I would enjoy having a cup of coffee with you. b. Thank you, but I would prefer to proceed with the assessment. c. No, but thank you. I never accept drinks from patients or families. d. Our agency policy prohibits me from eating or drinking in patients homes.

ANS: A Accepting refreshments or chatting informally with the patient and family represent therapeutic use of self and help to establish rapport. The distracters fail to help establish rapport.

24. A community mental health nurse has worked with a patient for 3 years but is moving out of the city and terminates the relationship. When a novice nurse begins work with this patient, what is the starting point for the relationship? a. Begin at the orientation phase. b. Resume the working relationship. c. Initially establish a social relationship. d. Return to the emotional catharsis phase.

ANS: A After termination of a long-term relationship, the patient and new nurse usually have to begin at ground zero, the orientation phase, to build a new relationship. If termination is successfully completed, the orientation phase sometimes progresses quickly to the working phase. Other times, even after successful termination, the orientation phase may be prolonged.

20. A patient in a support group says, Im tired of being sick. Everyone always helps me, but I will be glad when I can help someone else. This statement reflects: a. altruism. b. universality. c. cohesiveness. d. corrective recapitulation.

ANS: A Altruism refers to the experience of being helpful or useful to others, a condition that the patient anticipates will happen. The other options are also therapeutic factors identified by Yalom.

14. A therapy group adds new members as others leave. What type of group is evident? a. Open b. Closed c. Homogeneous d. Heterogeneous

ANS: A An open group is a group that adds members throughout the life of the group as other members leave and as more persons who would benefit from the group become available. A closed group does not add new members; the membership is established at the beginning and, except for the occasional losses as some members leave, does not change thereafter. A homogeneous group includes members who are similar, and a heterogeneous group includes dissimilar members; not enough data are provided here to determine which applies in this case.

17. A patient says, Ive done a lot of cheating and manipulating in my relationships. Select a nonjudgmental response by the nurse. a. How do you feel about that? b. I am glad that you realize this. c. Thats not a good way to behave. d. Have you outgrown that type of behavior?

ANS: A Asking a patient to reflect on feelings about his or her actions does not imply any judgment about those actions, and it encourages the patient to explore feelings and values. The remaining options offer negative judgments.

1. A psychiatric nurse best applies the ethical principle of autonomy by: a. exploring alternative solutions with a patient, who then makes a choice. b. suggesting that two patients who were fighting be restricted to the unit. c. intervening when a self-mutilating patient attempts to harm self. d. staying with a patient demonstrating a high level of anxiety.

ANS: A Autonomy is the right to self-determination, that is, to make ones own decisions. By exploring alternatives with the patient, the patient is better equipped to make an informed, autonomous decision. The distracters demonstrate beneficence, fidelity, and justice.

23. Which technique is most applicable to aversion therapy? a. Punishment b. Desensitization c. Role modeling d. Positive reinforcement

ANS: A Aversion therapy is akin to punishment. Aversive techniques include pairing of a maladaptive behavior with a noxious stimulus, punishment, and avoidance training.

11. A Mexican-American patient puts a picture of the Virgin Mary on the bedside table. Under which section of the assessment should the nurse document this behavior? a. Culture b. Ethnicity c. Verbal communication d. Nonverbal communication

ANS: A Cultural heritage is expressed through language, works of art, music, dance, ethnic clothing, customs, traditions, diet, and expressions of spirituality. This patient's prominent placement of the picture is an example of expression of cultural heritage. Verbal and nonverbal communications apply to all areas of an assessment.

28. A Category V tornado hits a community, destroying many homes and businesses. Which nursing intervention would best demonstrate compassion and caring? a. Encouraging persons to describe their memories and feelings about the event b. Arranging transportation to the local community mental health center c. Referring a local resident to a community food bank d. Coordinating psychiatric home care services

ANS: A Disaster victims benefit from telling their story. Nurses show compassion by listening and offering hope. The distracters identify other aspects of psychological first aid and services on the mental health continuum.

2. Which statement shows a nurse has empathy for a patient who made a suicide attempt? a. You must have been very upset when you tried to hurt yourself. b. It makes me sad to see you going through such a difficult experience. c. If you tell me what is troubling you, I can help you solve your problems. d. Suicide is a drastic solution to a problem that may not be such a serious matter.

ANS: A Empathy permits the nurse to see an event from the patients perspective, understand the patients feelings, and communicate this to the patient. The incorrect responses are nurse- centered (focusing on the nurses feelings rather than the patients), belittling, and sympathetic.

9. While talking with a patient diagnosed with major depression, a nurse notices the patient is unable to maintain eye contact. The patients chin lowers to the chest, while the patient looks at the floor. Which aspect of communication has the nurse assessed? a. Nonverbal communication b. A message filter c. A cultural barrier d. Social skills

ANS: A Eye contact and body movements are considered nonverbal communication. There are insufficient data to determine the level of the patients social skills or whether a cultural barrier exists.

32. A nurse and patient discuss a problem the patient has kept secret for many years. Afterward the patient says, I feel so relieved that I finally told somebody. Which term best describes the patients feeling? a. Catharsis b. Superego c. Cognitive distortion d. Counter-transference

ANS: A Freud initially used talk therapy, known as the cathartic method. Today we refer to catharsis as getting things off our chests. The superego represents the moral component of personality.

18. A patient is having difficulty making a decision. The nurse has mixed feelings about whether to provide advice. Which principle usually applies? Giving advice: a. is rarely helpful. b. fosters independence. c. lifts the burden of personal decision making. d. helps the patient develop feelings of personal adequacy.

ANS: A Giving advice fosters dependence on the nurse and interferes with the patients right to make personal decisions. It robs patients of the opportunity to weigh alternatives and develop problem-solving skills. Furthermore, it contributes to patient feelings of personal inadequacy. It also keeps the nurse in control and feeling powerful.

1. Inpatient hospitalization for persons with mental illness is generally reserved for patients who: a. present a clear danger to self or others. b. are noncompliant with medication at home. c. have limited support systems in the community. d. develop new symptoms during the course of an illness.

ANS: A Hospitalization is justified when the patient is a danger to self or others, has dangerously decompensated, or needs intensive medical treatment. The distracters do not necessarily describe patients who require inpatient treatment.

23. A community mental health nurse has worked for months to establish a relationship with a delusional, suspicious patient. The patient recently lost employment and could no longer afford prescribed medications. The patient says, Only a traitor would make me go to the hospital. Select the nurses best initial intervention. a. With the patients consent, contact resources to provide medications without charge temporarily. b. Arrange a bed in a local homeless shelter with nightly on-site supervision. c. Hospitalize the patient until the symptoms have stabilized. d. Ask the patient, Do you feel like I am a traitor?

ANS: A Hospitalization may damage the nurse-patient relationship, even if it provides an opportunity for rapid stabilization. If medication is restarted, the patient may possibly be stabilized in the home setting, even if it takes a little longer. Programs are available to help patients who are unable to afford their medications. A homeless shelter is inappropriate and unnecessary. Hospitalization may be necessary later, but a less restrictive solution should be tried first, since the patient is not dangerous. A yes/no question is non-therapeutic communication.

27. A patient repeatedly stated, Im stupid. Which statement by that patient would show progress resulting from cognitive behavioral therapy? a. Sometimes I do stupid things. b. Things always go wrong for me. c. I always fail when I try new things. d. Im disappointed in my lack of ability.

ANS: A Im stupid is a cognitive distortion. A more rational thought is Sometimes I do stupid things. The latter thinking promotes emotional self-control. The distracters reflect irrational or distorted thinking.

3. A patient diagnosed with major depression has lost 20 pounds in one month, has chronic low self-esteem, and a plan for suicide. The patient has taken an antidepressant medication for 1 week. Which nursing intervention has the highest priority? a. Implement suicide precautions. b. Offer high-calorie snacks and fluids frequently. c. Assist the patient to identify three personal strengths. d. Observe patient for therapeutic effects of antidepressant medication.

ANS: A Implementing suicide precautions is the only option related to patient safety. The other options, related to nutrition, self-esteem, and medication therapy, are important but are not priorities.

16. A patient tells the nurse at the clinic, "I haven't been taking my antidepressant medication as directed. I leave out the midday dose. I have lunch with friends and don't want them to ask me about the pills." Select the nurse's most appropriate intervention. a. Investigate the possibility of once-daily dosing of the antidepressant. b. Suggest to the patient to take the medication when no one is watching. c. Explain how taking each dose of medication on time relates to health maintenance. d. Add the following nursing diagnosis to the plan of care: Ineffective therapeutic regimen management, related to lack of knowledge.

ANS: A Investigating the possibility of once-daily dosing of the antidepressant has the highest potential for helping the patient achieve compliance. Many antidepressants can be administered by once-daily dosing, a plan that increases compliance. Explaining how taking each dose of medication on time relates to health maintenance is reasonable but would not achieve the goal; it does not address the issue of stigma. The self-conscious patient would not be comfortable doing this. A better nursing diagnosis would be related to social stigma. The question asks for an intervention, not analysis.

2. A leader plans to start a new self-esteem building group. Which intervention would be most helpful for assuring mutual respect within the group? a. Describe the importance of mutual respect in the first session and make it a group norm. b. Exclude potential members whose behavior suggests they are likely to be disrespectful. c. Give members a brochure describing the purpose, norms, and expectations of the group. d. Explain that mutual respect is expected and confront those who are not respectful.

ANS: A It is helpful to motivate members to behave respectfully by describing how mutual respect benefits all members and is necessary for the group to be fully therapeutic. Setting a tone and expectation of mutual respect from the outset is the most helpful intervention listed. Excluding members because of how they might behave could exclude members who would have been appropriate, depriving them of the potential benefits of the group. Conveying expectations by brochure is less effective than doing so orally, because it lacks the connection to each member a skilled leader can create to motivate members and impart the expectation of respect. Confronting inappropriate behavior is therapeutic but only addresses existing behavior rather than preventing all such undesired behavior.

17. A patient in alcohol rehabilitation reveals to the nurse, I feel terrible guilt for sexually abusing my 6-year-old before I was admitted. Select the nurses most important action. a. Anonymously report the abuse by phone to the local child protection agency. b. Reply, Im glad you feel comfortable talking to me about it. c. File a written report with the agencys ethics committee. d. Respect nurse-patient relationship confidentiality.

ANS: A Laws regarding child abuse reporting discovered by a professional during the suspected abusers alcohol or drug treatment differ by state. Federal law supersedes state law and prohibits disclosure without a court order except in instances in which the report can be made anonymously or without identifying the abuser as a patient in an alcohol or drug treatment facility.

8. A soldier returns to the United States from active duty in a combat zone in Afghanistan. The soldier is diagnosed with post-traumatic stress disorder (PTSD). The nurse's highest priority is to screen this soldier for which problem? a. Major depressive disorder b. Bipolar disorder c. Schizophrenia d. Dementia

ANS: A Major depressive disorder frequently co-occurs with PTSD. The incidence of the disorders identified in the distractors is similar to the general population.

11. During a one-on-one interaction with the nurse, a patient frequently looks nervously at the door. Select the best comment by the nurse regarding this nonverbal communication. a. I notice you keep looking toward the door. b. This is our time together. No one is going to interrupt us. c. It looks as if you are eager to end our discussion for today. d. If you are uncomfortable in this room, we can move someplace else.

ANS: A Making observations and encouraging the patient to describe perceptions are useful therapeutic communication techniques for this situation. The other responses are assumptions made by the nurse.

26. A patient would benefit from therapy in which peers as well as staff have a voice in determining patients privileges and psychoeducational topics. Which approach would be best? a. Milieu therapy b. Cognitive therapy c. Short-term dynamic therapy d. Systematic desensitization

ANS: A Milieu therapy is based on the idea that all members of the environment contribute to the planning and functioning of the setting. The distracters are individual therapies that do not fit the description.

10. A nurse is concerned that an agencys policies are inadequate. Which understanding about the relationship between substandard institutional policies and individual nursing practice should guide nursing practice? a. Agency policies do not exempt an individual nurse of responsibility to practice according to professional standards of nursing care. b. Agency policies are the legal standard by which a professional nurse must act and therefore override other standards of care. c. Faced with substandard policies, a nurse has a responsibility to inform the supervisor and discontinue patient care immediately. d. Interpretation of policies by the judicial system is rendered on an individual basis and therefore cannot be predicted.

ANS: A Nurses are professionally bound to uphold standards of practice regardless of lesser standards established by a health care agency or a state. Conversely, if the agency standards are higher than standards of practice, the agency standards must be upheld. The courts may seek to establish the standard of care through the use of expert witnesses when the issue is clouded.

28. In order to release information to another health care facility or third party regarding a patient diagnosed with a mental illness, the nurse must obtain: a. a signed consent by the patient for release of information stating specific information to be released. b. a verbal consent for information release from the patient and the patients guardian or next of kin. c. permission from members of the health care team who participate in treatment planning. d. approval from the attending psychiatrist to authorize the release of information.

ANS: A Nurses have an obligation to protect patients privacy and confidentiality. Clinical information should not be released without the patients signed consent for the release.

13. A patient who had been experiencing significant stress learned to use progressive muscle relaxation and deep breathing exercises. When the patient returns to the clinic 2 weeks later, which finding most clearly shows the patient is coping more effectively with stress? a. The patients systolic blood pressure has changed from the 140s to the 120s mm Hg. b. The patient reports, I feel better, and that things are not bothering me as much. c. The patient reports, I spend more time napping or sitting quietly at home. d. The patients weight decreased by 3 pounds.

ANS: A Objective measures tend to be the most reliable means of gauging progress. In this case, the patients elevated blood pressure, an indication of the bodys physiological response to stress, has diminished. The patients report regarding activity level is subjective; sitting quietly could reflect depression rather than improvement. Appetite, mood, and energy levels are also subjective reports that do not necessarily reflect physiological changes from stress and may not reflect improved coping with stress. The patients weight change could be a positive or negative indicator; the blood pressure change is the best answer.

13. A nurse explains to the family of a mentally ill patient how a nurse-patient relationship differs from social relationships. Which is the best explanation? a. The focus is on the patient. Problems are discussed by the nurse and patient, but solutions are implemented by the patient. b. The focus shifts from nurse to patient as the relationship develops. Advice is given by both, and solutions are implemented. c. The focus of the relationship is socialization. Mutual needs are met, and feelings are shared openly. d. The focus is creation of a partnership in which each member is concerned with growth and satisfaction of the other.

ANS: A Only the correct response describes elements of a therapeutic relationship. The remaining responses describe events that occur in social or intimate relationships.

22. A nurse leads a psychoeducational group for patients in the community diagnosed with schizophrenia. A realistic outcome for group members is that they will: a. discuss ways to manage their illness. b. develop a high level of trust and cohesiveness. c. understand unconscious motivation for behavior. d. demonstrate insight about development of their illness.

ANS: A Patients with schizophrenia almost universally have problems associated with everyday living in the community, so discussing ways to manage the illness would be an important aspect of psychoeducation. Discussing concerns about daily life would be a goal to which each could relate. Developing trust and cohesion is desirable but is not the priority outcome of a psychoeducational group. Understanding unconscious motivation would not be addressed. Insight would be difficult for a patient with residual schizophrenia because of the tendency toward concrete thinking.

16. A group begins the working phase. One member has a childhood history of neglect and ridicule by parents. Which comment would the group leader expect from this member? a. My boss is always expecting more of me than the others, but talking to him would only make it worse. b. Im sorry for talking all the time, but there is so much going on in my life. I cant remember what I already said. c. Thanks for the suggestions everyone. Maybe some of them will help. It wont hurt to give them a try. d. This group is stupid. Nobody here can help anybody else because we are all so confused. Its a waste.

ANS: A People who frequently complain, yet reject help or suggestions when offered, tend to have histories of severe deprivation as children, often accompanied by neglect or abuse. The other comments reflect dynamics other than the help-rejecting complainer, such as the monopolizer who apologizes for talking too much, the person who is insightful and agrees to try a peers suggestion, and the demoralizing member.

5. Which technique will best communicate to a patient that the nurse is interested in listening? a. Restating a feeling or thought the patient has expressed. b. Asking a direct question, such as Did you feel angry? c. Making a judgment about the patients problem. d. Saying, I understand what youre saying.

ANS: A Restating allows the patient to validate the nurses understanding of what has been communicated. Restating is an active listening technique. Judgments should be suspended in a nurse-patient relationship. Close-ended questions such as Did you feel angry? ask for specific information rather than showing understanding. When the nurse simply states that he or she understands the patients words, the patient has no way of measuring the understanding.

14. A soldier in a combat zone tells the nurse, "I saw a child get blown up over a year ago, and now I keep seeing bits of flesh everywhere. I see something red and the visions race back to my mind." Which phenomenon associated with post-traumatic stress disorder (PTSD) is this soldier describing? a. Re-experiencing b. Hyperarousal c. Avoidance d. Psychosis

ANS: A Spontaneous or cued recurrent, involuntary, and intrusive distressing memories of the traumatic events are often associated with PTSD. The soldier has described intrusive thoughts and visions associated with re-experiencing the traumatic event. This description does not indicate psychosis, hypervigilance, or avoidance.

13. Three members of a therapy group share covert glances as other members of the group describe problems. When one makes a statement that subtly criticizes another speaker, the others nod in agreement. Which group dynamic should the leader suspect? a. Some members are acting as a subgroup instead of as members of the main group. b. Some of the members have become bored and are disregarding others. c. Three members are showing their frustration with slower members. d. The leadership of the group has been ineffective.

ANS: A Subgroups, small groups isolated within a larger group and functioning separately from it, sometimes form within therapy groups. When this occurs, subgroup members are cohesive with other subgroup members but not with the members of the larger group. Members of the subgroup may be bored or frustrated or expressing passive aggression, but the primary dynamic is the splitting off from the main group.

3. Select the example of tertiary prevention. a. Helping a person diagnosed with a serious mental illness learn to manage money b. Restraining an agitated patient who has become aggressive and assaultive c. Teaching school-age children about the dangers of drugs and alcohol d. Genetic counseling with a young couple expecting their first child

ANS: A Tertiary prevention involves services that address residual impairments, with a goal of improved independent functioning. Restraint is a secondary prevention. Genetic counseling and teaching school-age children about substance abuse and dependence are examples of primary prevention.

9. A patient is suspicious and frequently manipulates others. To which psychosexual stage do these traits relate? a. Oral b. Anal c. Phallic d. Genital

ANS: A The behaviors in the stem develop as the result of attitudes formed during the oral stage, when an infant first learns to relate to the environment. Anal-stage traits include stinginess, stubbornness, orderliness, or their opposites. Phallic-stage traits include flirtatiousness, pride, vanity, difficulty with authority figures, and difficulties with sexual identity. Genital-stage traits include the ability to form satisfying sexual and emotional relationships with members of the opposite sex, emancipation from parents, a strong sense of personal identity, or the opposites of these traits.

33. Which patient is the best candidate for brief psychodynamic therapy? a. An accountant with a loving family and successful career who was involved in a short extramarital affair b. An adult with a long history of major depression who was charged with driving under the influence (DUI) c. A woman with a history of borderline personality disorder who recently cut both wrists d. An adult male recently diagnosed with anorexia nervosa

ANS: A The best candidates for psychodynamic therapy are relatively healthy and well-functioning individuals, sometimes referred to as the worried well, who have a clearly circumscribed area of difficulty and are intelligent, psychologically minded, and well-motivated for change. Patients with psychosis, severe depression, borderline personality disorders, and severe character disorders are not appropriate candidates for this type of treatment.

17. A nurse talks with the caregiver of a combat veteran diagnosed with severe traumatic brain injuries. The caregiver says, "I don't know how much longer I can do it. My whole life is consumed with taking care of my partner." Select the nurse's best response. a. "How are you taking care of yourself?" b. "Let's review your partner's diagnostic results." c. "I have some web-based programs for you to visit." d. "Your partner is lucky to have someone so devoted."

ANS: A The caregiver is the focus of the nurse's attention. The caregiver is suffering. The nurse must be empathetic and assess how the caregiver is caring for self. Reassurance and isolated computer activities do not help. The partner is already aware of the diagnostic results.

18. A professor's 4-year-old child has a fever of 101.6° F, diarrhea, and complains of stomach pain. The professor is scheduled to teach three classes today. Which nursing diagnosis best applies to this scenario? a. Decisional conflict b. Unilateral neglect c. Disabled family coping d. Ineffective management of the therapeutic regimen

ANS: A The caregiver is the focus of the nurse's attention. The professor is under stress, related to the conflict between his parenting and professional roles. This scenario presents a decisional conflict. No evidence suggests that family coping is disabled or that a therapeutic regimen is not managed. Unilateral neglect refers to the awareness of the body.

22. A person in the community asks, Why arent people with mental illness kept in state institutions anymore? Select the nurses best response. a. Less restrictive settings are available now to care for individuals with mental illness. b. There are fewer persons with mental illness, so less hospital beds are needed. c. Most people with mental illness are still in psychiatric institutions. d. Psychiatric institutions violated patients rights.

ANS: A The community is a less restrictive alternative than hospitals for treatment of persons with mental illness. The distracters are incorrect and part of the stigma of mental illness.

6. Which finding best indicates that the goal Demonstrate mentally healthy behavior was achieved? A patient: a. sees self as capable of achieving ideals and meeting demands. b. behaves without considering the consequences of personal actions. c. aggressively meets own needs without considering the rights of others. d. seeks help from others when assuming responsibility for major areas of own life.

ANS: A The correct response describes an adaptive, healthy behavior. The distracters describe maladaptive behaviors.

4. A 4-year-old grabs toys from siblings and says, I want that now! The siblings cry, and the childs parent becomes upset with the behavior. According to Freudian theory, this behavior is a product of impulses originating in which system of the personality? a. Id b. Ego c. Superego d. Preconscious

ANS: A The id operates on the pleasure principle, seeking immediate gratification of impulses. The ego acts as a mediator of behavior and weighs the consequences of the action, perhaps determining that taking the toy is not worth the mothers wrath. The superego would oppose the impulsive behavior as not nice. The preconscious is a level of awareness.

13. A nurse assesses soldiers in a combat zone in Afghanistan. When is it most important for the nurse to screen for signs and symptoms of traumatic brain injury (TBI)? a. After a fall, vehicle crash, or exposure to a blast b. Before departing Afghanistan to return to the United States c. One year after returning to the United States from Afghanistan d. Immediately upon return to the United States from Afghanistan

ANS: A The military estimates that up to 20% of the combat veterans in both Afghanistan and Iraq suffer some degree of TBI. TBI exhibits signs shortly after the injury, and these signs usually resolve in days or weeks. Screening after an exposure to an explosion and before returning to the United States is important.

9. A patient has talked constantly throughout the group therapy session, often repeating the same comments. Other members were initially attentive then became bored, inattentive, and finally sullen. Which comment by the nurse leader would be most effective? a. Say to everyone, Most of you have become quiet. I wonder if it might be related to concerns you may have about how the group is progressing today. b. Say to everyone, One person has done most of the talking. I think it would be helpful for everyone to say how that has affected your experience of the group. c. Say to everyone, I noticed that as our group progressed, most members became quiet, then disinterested, and now seem almost angry. What is going on? d. Say to the talkative patient, You have been doing most of the talking, and others have not had a chance to speak as a result. Could you please yield to others now?

ANS: A The most effective action the nurse leader can take will be the one that encourages the group to solve its own problem. Pointing out changes in the group and asking members to respond to them lays the foundation for a discussion of group dynamics. Asking members to respond to the talkative patient puts that patient in an awkward position, likely increasing her anxiety. As anxiety increases, monopolizing behavior tends to increase as well, so this response would be self-defeating. Asking members what is going on is a broader opening and might lead to responses unrelated to the issue that bears addressing; narrowing the focus to the group process more directly addresses what is occurring in the group. Focusing on the talkative patient would be less effective and involves the leader addressing the issue instead of members first attempting to do so themselves (giving them a chance to practice skills such as assertive communication).

14. When a female Mexican American patient and a female nurse sit together, the patient often holds the nurses hand. The patient also links arms with the nurse when they walk. The nurse is uncomfortable with this behavior. Which analysis is most accurate? a. The patient is accustomed to touch during conversation, as are members of many Hispanic subcultures. b. The patient understands that touch makes the nurse uncomfortable and controls the relationship based on that factor. c. The patient is afraid of being alone. When touching the nurse, the patient is reassured and comforted. d. The patient is trying to manipulate the nurse using nonverbal techniques.

ANS: A The most likely answer is that the patients behavior is culturally influenced. Hispanic women frequently touch women they consider to be their friends. Although the other options are possible, they are less likely.

15. A nurse uses Maslows hierarchy of needs to plan care for a patient with mental illness. Which problem will receive priority? The patient: a. refuses to eat or bathe. b. reports feelings of alienation from family. c. is reluctant to participate in unit social activities. d. is unaware of medication action and side effects.

ANS: A The need for food and hygiene are physiological and therefore take priority over psychological or meta-needs in care planning.

22. As a nurse discharges a patient, the patient gives the nurse a card of appreciation made in an arts and crafts group. What is the nurses best action? a. Recognize the effectiveness of the relationship and patients thoughtfulness. Accept the card. b. Inform the patient that accepting gifts violates policies of the facility. Decline the card. c. Acknowledge the patients transition through the termination phase but decline the card. d. Accept the card and invite the patient to return to participate in other arts and crafts groups.

ANS: A The nurse must consider the meaning, timing, and value of the gift. In this instance, the nurse should accept the patients expression of gratitude.

20. A patient with acute depression states, God is punishing me for my past sins. What is the nurses most therapeutic response? a. You sound very upset about this. b. God always forgives us for our sins. c. Why do you think you are being punished? d. If you feel this way, you should talk to your minister.

ANS: A The nurse reflects the patients comment, a therapeutic technique to encourage sharing for perceptions and feelings. The incorrect responses reflect probing, closed-ended comments, and giving advice, all of which are non-therapeutic.

22. A patient diagnosed with schizophrenia has been stable for 2 months. Today the patients spouse calls the nurse to report the patient has not taken prescribed medication and is having disorganized thinking. The patient forgot to refill the prescription. The nurse arranges a refill. Select the best outcome to add to the plan of care. a. The patients spouse will mark dates for prescription refills on the family calendar. b. The nurse will obtain prescription refills every 90 days and deliver to the patient. c. The patient will call the nurse weekly to discuss medication-related issues. d. The patient will report to the clinic for medication follow-up every week.

ANS: A The nurse should use the patients support system to meet patient needs whenever possible. Delivery of medication by the nurse should be unnecessary for the nurse to do if patient or a significant other can be responsible. The patient may not need more intensive follow-up as long as medication is taken as prescribed.

7. A 40-year-old adult living with parents states, "I'm happy but I don't socialize much. My work is routine. When new things come up, my boss explains them a few times to make sure I understand. At home, my parents make decisions for me, and I go along with them." A nurse should identify interventions to improve this patient's: a. self-concept. b. overall happiness. c. appraisal of reality. d. control over behavior.

ANS: A The patient feels the need for multiple explanations of new tasks at work and, despite being 40 years of age, allows both parents to make all decisions. These behaviors indicate a poorly developed self-concept. Although the patient reports being happy, the subsequent comments refute that self-appraisal. The patient's comments do not indicate that he/she is out of touch with reality. The patient's needs are broader than control over own behavior.

9. At what point in the nurse-patient relationship should a nurse plan to first address termination? a. During the orientation phase b. At the end of the working phase c. Near the beginning of the termination phase d. When the patient initially brings up the topic

ANS: A The patient has a right to know the conditions of the nurse-patient relationship. If the relationship is to be time-limited, the patient should be informed of the number of sessions. If it is open-ended, the termination date will not be known at the outset, and the patient should know that the issue will be negotiated at a later date. The nurse is responsible for bringing up the topic of termination early in the relationship, usually during the orientation phase.

30. A nurse performed these actions while caring for patients in an inpatient psychiatric setting. Which action violated patients rights? a. Prohibited a patient from using the telephone b. In patients presence, opened a package mailed to patient c. Remained within arms length of patient with homicidal ideation d. Permitted a patient with psychosis to refuse oral psychotropic medication

ANS: A The patient has a right to use the telephone. The patient should be protected against possible harm to self or others. Patients have rights to send and receive mail and be present during package inspection. Patients have rights to refuse treatment.

8. A nurse surveys the medical records for violations of patients' rights. Which finding signals a violation? a. No treatment plan is present in record. b. Patient belongings are searched at admission. c. Physical restraint is used to prevent harm to self. d. Patient is placed on one-to-one continuous observation.

ANS: A The patient has the right to have a treatment plan. Inspecting a patient's belongings is a safety measure. Patients have the right to a safe environment, including the right to be protected against impulses to harm self that occur as a result of a mental disorder.

8. A nurse surveys medical records. Which finding signals a violation of patients rights? a. A patient was not allowed to have visitors. b. A patients belongings were searched at admission. c. A patient with suicidal ideation was placed on continuous observation. d. Physical restraint was used after a patient was assaultive toward a staff member.

ANS: A The patient has the right to have visitors. Inspecting patients belongings is a safety measure. Patients have the right to a safe environment, including the right to be protected against impulses to harm self.

18. When a nurse assesses an older adult patient, answers seem vague or unrelated to the questions. The patient also leans forward and frowns, listening intently to the nurse. An appropriate question for the nurse to ask would be: a. Are you having difficulty hearing when I speak? b. How can I make this assessment interview easier for you? c. I notice you are frowning. Are you feeling annoyed with me? d. Youre having trouble focusing on what Im saying. What is distracting you?

ANS: A The patients behaviors may indicate difficulty hearing. Identifying any physical need the patient may have at the onset of the interview and making accommodations are important considerations. By asking if the patient is annoyed, the nurse is jumping to conclusions. Asking how to make the interview easier for the patient may not elicit a concrete answer. Asking about distractions is a way of asking about auditory hallucinations, which is not appropriate because the nurse has observed that the patient seems to be listening intently.

4. During group therapy, one patient says to another, When I first started in this group, you were unable to make a decision, but now you can. Youve made a lot of progress. I am beginning to think that maybe I can conquer my fears too. Which therapeutic factor is evident by this statement? a. Hope b. Altruism c. Catharsis d. Cohesiveness

ANS: A The patients profession that he may be able to learn to cope more effectively reflects hope. Groups can instill hope in individuals who are demoralized or pessimistic. Altruism refers to doing good for others, which can result in positive feelings about oneself. Catharsis refers to venting of strong emotions. Cohesion refers to coming together and developing a connection with other group members.

15. In the majority culture of the United States, which individual has the greatest risk to be labeled mentally ill? One who: a. describes hearing Gods voice speaking. b. is usually pessimistic but strives to meet personal goals. c. is wealthy and gives away $20 bills to needy individuals. d. always has an optimistic viewpoint about life and having own needs met.

ANS: A The question asks about risk. Hearing voices is generally associated with mental illness, but in charismatic religious groups, hearing the voice of God or a prophet is a desirable event. Cultural norms vary, which makes it more difficult to make an accurate diagnosis. The individuals described in the other options are less likely to be labeled mentally ill.

15. Insurance will not pay for continued private hospitalization of a mentally ill patient. The family considers transferring the patient to a public hospital but expresses concern that the patient will not get any treatment if transferred. Select the nurses most helpful reply. a. By law, treatment must be provided. Hospitalization without treatment violates patients rights. b. All patients in public hospitals have the right to choose both a primary therapist and a primary nurse. c. You have a justifiable concern because the right to treatment extends only to provision of food, shelter, and safety. d. Much will depend on other patients, because the right to treatment for a psychotic patient takes precedence over the right to treatment of a patient who is stable.

ANS: A The right to medical and psychiatric treatment was conferred on all patients hospitalized in public mental hospitals with the enactment of the federal Hospitalization of Mentally Ill Act in 1964.

9. Which principle has the highest priority when addressing a behavioral crisis in an inpatient setting? a. Resolve the crisis with the least restrictive intervention possible. b. Swift intervention is justified to maintain the integrity of a therapeutic milieu. c. Rights of an individual patient are superseded by the rights of the majority of patients. d. Patients should have opportunities to regain control without intervention if the safety of others is not compromised.

ANS: A The rule of using the least restrictive treatment or intervention possible to achieve the desired outcome is the patients legal right. Planned interventions are nearly always preferable. Intervention may be necessary when the patient threatens harm to self.

14. A psychiatric technician says, "Little of what takes place on the behavioral health unit seems to be theory based." A nurse educates the technician by identifying which common use of Sullivan's theory? a. Structure of the therapeutic milieu of most behavioral health units b. Frequent use of restraint and seclusion for behavior modification c. Assessment tools based on age-appropriate versus arrested behaviors d. Use of the nursing process to determine the best sequence for nursing actions

ANS: A The structure of the therapeutic environment has, as its foci, an accepting atmosphere and provision of opportunities for practicing interpersonal skills. Both constructs are directly attributable to Sullivan's theory of interpersonal relationships. Sullivan's interpersonal theory did not specifically consider the use of restraint or seclusion. Assessment based on the developmental level is associated with Erikson's theories. The nursing process applies concepts from multiple theories.

20. Consider this comment from a therapist: The patient is homosexual but has kept this preference secret. Severe anxiety and depression occur when the patient anticipates family reactions to this sexual orientation. Which perspective is evident in the speaker? a. Theory of interpersonal relationships b. Classical conditioning theory c. Psychosexual theory d. Behaviorism theory

ANS: A The theory of interpersonal relationships recognizes the anxiety and depression as resulting from unmet interpersonal security needs. Behaviorism and classical conditioning theories do not apply. A psychosexual formulation would focus on uncovering unconscious material that relates to the patient problem.

19. Which scenario best depicts a behavioral crisis? A patient is: a. waving fists, cursing, and shouting threats at a nurse. b. curled up in a corner of the bathroom, wrapped in a towel. c. crying hysterically after receiving a phone call from a family member. d. performing push-ups in the middle of the hall, forcing others to walk around.

ANS: A This behavior constitutes a behavioral crisis because the patient is threatening harm to another individual. Intervention is called for to defuse the situation. The other options speak of behaviors that may require intervention of a less urgent nature because the patients in question are not threatening harm to self or others.

3. A multidisciplinary health care team meets 12 hours after an adolescent is hospitalized after a suicide attempt. Members of the team report their assessments. What outcome can be expected from this meeting? a. A treatment plan will be formulated. b. The health care provider will order neuroimaging studies. c. The team will request a court-appointed advocate for the patient. d. Assessment of the patient's need for placement outside the home will be undertaken.

ANS: A Treatment plans are formulated early in the course of treatment to streamline the treatment process and reduce costs. It is too early to determine the need for alternative post-discharge living arrangements. Neuroimaging is not indicated for this scenario.

20. Which employer's health plan is required to include parity provisions related to mental illnesses? a. Employer with more than 50 employees b. Cancer thrift shop staffed by volunteers c. Daycare center that employs 7 teachers d. Church that employs 15 people

ANS: A Under federal parity laws, companies with more than 50 employees may not limit annual or lifetime mental health benefits unless they also limit benefits for physical illnesses.

26. A nurse says, I am the only one who truly understands this patient. Other staff members are too critical. The nurses statement indicates: a. boundary blurring. b. sexual harassment. c. positive regard. d. advocacy.

ANS: A When the role of the nurse and the role of the patient shift, boundary blurring may arise. In this situation the nurse is becoming over-involved with the patient as a probable result of unrecognized countertransference. When boundary issues occur, the need for supervision exists. The situation does not describe sexual harassment. Data are not present to suggest positive regard or advocacy.

3. A nurse leads a psychoeducational group for depressed patients. The nurse plans to implement an exercise regime for each patient. The rationale to use when presenting this plan to the treatment team is that exercise: a. has an antidepressant effect comparable to selective serotonin reuptake inhibitors. b. prevents damage from overstimulation of the sympathetic nervous system. c. detoxifies the body by removing metabolic wastes and other toxins. d. improves mood stability for patients with bipolar disorders.

ANS: A b-Endorphins produced during exercise result in improvement in mood and lowered anxiety. The other options are not accurate.

3. Which situations qualify as abandonment on the part of a nurse? (Select all that apply.) The nurse: a. allows a patient with acute mania to refuse hospitalization without taking further action. b. terminates employment without referring a seriously mentally ill for aftercare. c. calls police to bring a suicidal patient to the hospital after a suicide attempt. d. refers a patient with persistent paranoid schizophrenia to community treatment. e. asks another nurse to provide a patient's care because of concerns about countertransference.

ANS: A, B Abandonment arises when a nurse does not place a patient safely in the hands of another health professional before discontinuing treatment. Calling the police to bring a suicidal patient to the hospital after a suicide attempt and referring a patient with schizophrenia to community treatment both provide for patient safety. Asking another nurse to provide a patient's care because of concerns about countertransference demonstrates self-awareness.

1. An experienced nurse says to a new graduate, When youve practiced as long as I have, you instantly know how to take care of psychotic patients. What information should the new graduate consider when analyzing this comment? Select all that apply. a. The experienced nurse may have lost sight of patients individuality, which may compromise the integrity of practice. b. New research findings should be integrated continuously into a nurses practice to provide the most effective care. c. Experience provides mental health nurses with the essential tools and skills needed for effective professional practice. d. Experienced psychiatric nurses have learned the best ways to care for mentally ill patients through trial and error. e. An intuitive sense of patients needs guides effective psychiatric nurses.

ANS: A, B Evidence-based practice involves using research findings and standards of care to provide the most effective nursing care. Evidence is continuously emerging, so nurses cannot rely solely on experience. The effective nurse also maintains respect for each patient as an individual. Overgeneralization compromises that perspective. Intuition and trial and error are unsystematic approaches to care.

2. A nurse ends a relationship with a patient. Which actions by the nurse should be included in the termination phase? Select all that apply. a. Focus dialogues with the patient on problems that may occur in the future. b. Help the patient express feelings about the relationship with the nurse. c. Help the patient prioritize and modify socially unacceptable behaviors. d. Reinforce expectations regarding the parameters of the relationship. e. Help the patient to identify strengths, limitations, and problems.

ANS: A, B The correct actions are part of the termination phase. The other actions would be used in the working and orientation phases.

4. A professor's 4-year-old child has a temperature of 101.6° F, diarrhea, and complains of stomach pain. The professor is scheduled to teach three classes today. Which actions by the professor demonstrate effective parenting? Select all that apply. a. Telephoning a grandparent to stay with the child at home for the day. b. Telephoning a colleague to teach his classes and staying home with the sick child. c. Taking the child to the university and keeping the child in a private office for the day. d. Taking the child to a daycare center and hoping daycare workers will not notice the child is sick. e. Giving the child one dose of ibuprofen (Motrin) and taking the child to the daycare center.

ANS: A, B The correct responses demonstrate fulfillment of the role as a parent. The distractors indicate the professor has not cared for the sick child in an effective way. Taking the child to a daycare center exposes other children to a potential infection. Taking the child to one's office does not keep the child comfortable or provide for the child while the professor is teaching.

2. Which benefits are most associated with use of telehealth technologies? Select all that apply. a. Cost savings for patients b. Maximize care management c. Access to services for patients in rural areas d. Prompt reimbursement by third party payers e. Rapid development of trusting relationships with patients

ANS: A, B, C Telehealth has shown it can maximize health and improve disease management skills and confidence with the disease process. Many rural parents have felt disconnected from services; telehealth technologies can solve those problems. Although telehealths improved health outcomes regularly show cost savings for payers, one significant barrier is the current lack of reimbursement for remote patient monitoring by third party payers. Telehealth technologies have not shown rapid development of trusting relationships.

1. Which changes reflect short-term physiological responses to stress? Select all that apply. a. Muscular tension, blood pressure, and triglycerides increase. b. Epinephrine is released, increasing heart and respiratory rates. c. Corticosteroid release increases stamina and impedes digestion. d. Cortisol is released, increasing glucogenesis and reducing fluid loss. e. Immune system functioning decreases, and risk of cancer increases. f. Risk of depression, autoimmune disorders, and heart disease increases.

ANS: A, B, C, D The correct answers are all short-term physiological responses to stress. Increased risk of immune system dysfunction, cancer, cardiovascular disease, depression, and autoimmune disease are all long-term (chronic) effects of stress.

4. An individual says to the nurse, I feel so stressed out lately. I think the stress is affecting my body also. Which somatic complaints are most likely to accompany this feeling? Select all that apply. a. Headache b. Neck pain c. Insomnia d. Anorexia e. Myopia

ANS: A, B, C, D When individuals feel stressed-out, they often have accompanying somatic complaints, especially associated with sleep, eating, and headache or back pain. Changes in vision, such as myopia, would not be expected.

1. A nurse can best address factors of critical importance to successful community treatment by including making assessments relative to: (Select all that apply.) a. housing adequacy. b. family and support systems. c. income adequacy and stability. d. early psychosocial development. e. substance abuse history and current use.

ANS: A, B, C, E Early psychosocial developmental history is less relevant to successful outcomes in the community than the assessments listed in the other options. If a patient is homeless or fears homelessness, focusing on other treatment issues is impossible. Sufficient income for basic needs and medication is necessary. Adequate support is a requisite to community placement. Substance abuse undermines medication effectiveness and interferes with community adjustment.

1. A patient states, Im starting cognitive-behavioral therapy. What can I expect from the sessions? Which responses by the nurse would be appropriate? Select all that apply. a. The therapist will be active and questioning. b. You will be given some homework assignments. c. The therapist will ask you to describe your dreams. d. The therapist will help you look at your ideas and beliefs about yourself. e. The goal is to increase subjectivity about thoughts that govern your behavior.

ANS: A, B, D Cognitive therapists are active rather than passive during therapy sessions because they help patients reality-test their thinking. Homework assignments are given and completed outside the therapy sessions. Homework is usually discussed at the next therapy session. The goal of cognitive therapy is to assist the patient in identifying inaccurate cognitions and in reality- testing and formulating new, accurate cognitions. One distracter applies to psychoanalysis. Increasing subjectivity is not desirable.

4. Which statements by patients diagnosed with a serious mental illness best demonstrate that the case manager has established an effective long-term relationship? My case manager: (select all that apply) a. talks in language I can understand. b. helps me keep track of my medication. c. gives me little gifts from time to time. d. looks at me as a whole person with many needs. e. lets me do whatever I choose without interfering.

ANS: A, B, D Each correct answer is an example of appropriate nursing foci: communicating at a level understandable to the patient, providing medication supervision, and using holistic principles to guide care. The distracters violate relationship boundaries or suggest a laissez faire attitude on the part of the nurse.

3. A psychiatric nurse discusses rules of the therapeutic milieu and patients rights with a newly admitted patient. Which rights should be included? (Select all that apply.) The right to: a. have visitors b. confidentiality c. a private room d. complain about inadequate care e. select the nurse assigned to their care

ANS: A, B, D Patients rights should be discussed shortly after admission. Patients have rights related to receiving/refusing visitors, privacy, filing complaints about inadequate care, and accepting/refusing treatments (including medications). Patients do not have a right to a private room or selecting which nurse will provide care.

2. A nurse performing an assessment interview for a patient with a substance use disorder decides to use a standardized rating scale. Which scales are appropriate? Select all that apply. a. Addiction Severity Index (ASI) b. Brief Drug Abuse Screen Test (B-DAST) c. Abnormal Involuntary Movement Scale (AIMS) d. Cognitive Capacity Screening Examination (CCSE) e. Recovery Attitude and Treatment Evaluator (RAATE)

ANS: A, B, E Standardized scales are useful for obtaining data about substance use disorders. The ASI, B-DAST, and RAATE are scales related to substance abuse. AIMS assesses involuntary movements associated with anti-psychotic medications. The CCSE assesses cognitive function.

1. A basic level registered nurse works with patients in a community setting. Which groups should this nurse expect to lead? Select all that apply. a. Symptom management b. Medication education c. Family therapy d. Psychotherapy e. Self-care

ANS: A, B, E Symptom management, medication education, and self-care groups represent psychoeducation, a service provided by the basic level registered nurse. Advanced practice registered nurses provide family therapy and psychotherapy.

6. A person in the community asks, People with mental illnesses went to state hospitals in earlier times. Why has that changed? Select the nurses accurate responses. Select all that apply. a. Science has made significant improvements in drugs for mental illness, so now many persons may live in their communities. b. Theres now a better selection of less restrictive treatment options available in communities to care for people with mental illness. c. National rates of mental illness have declined significantly. There actually is not a need for state institutions anymore. d. Most psychiatric institutions were closed because of serious violations of patients rights and unsafe conditions. e. Federal legislation and payment for treatment of mental illness has shifted the focus to community rather than institutional settings.

ANS: A, B, E The community is a less restrictive alternative than hospitals for treatment of persons with mental illness. Funding for treatment of mental illness remains largely inadequate but now focuses on community rather than institutional care. Antipsychotic medications improve more symptoms of mental illness; hence, management of psychiatric disorders has improved. Rates of mental illness have increased, not decreased. Hospitals were closed because funding shifted to the community. Conditions in institutions have improved.

4. A nurse is interacting with patients in a psychiatric unit. Which statements reflect use of therapeutic communication? Select all that apply. a. Tell me more about that situation. b. Lets talk about something else. c. I notice you are pacing a lot. d. Ill stay with you a while. e. Why did you do that?

ANS: A, C, D The correct responses demonstrate use of the therapeutic techniques making an observation and showing empathy. The incorrect responses demonstrate changing the subject and probing, which are non-therapeutic techniques.

2. The health care team at an inpatient psychiatric facility drafts these criteria for admission. Which criteria should be included in the final version of the admission policy? Select all that apply. a. Clear risk of danger to self or others b. Adjustment needed for doses of psychotropic medication c. Detoxification from long-term heavy alcohol consumption needed d. Respite for caregivers of persons with serious and persistent mental illness e. Failure of community-based treatment, demonstrating need for intensive treatment

ANS: A, C, E Medication doses can be adjusted on an outpatient basis. The goal of caregiver respite can be accomplished without hospitalizing the patient. The other options are acceptable, evidence-based criteria for admission of a patient to an inpatient service.

7. A patient diagnosed with schizophrenia lives in the community. On a home visit, the community psychiatric nurse case manager learns that the patient: wants to attend an activity group at the mental health outreach center; is worried about being able to pay for the therapy; does not know how to get from home to the outreach center; has an appointment to have blood work at the same time an activity group meets; wants to attend services at a church that is a half-mile from the patients home. Which tasks are part of the role of a community mental health nurse? Select all that apply. a. Rearranging conflicting care appointments b. Negotiating the cost of therapy for the patient c. Arranging transportation to the outreach center d. Accompanying the patient to church services weekly e. Monitoring to ensure the patients basic needs are met

ANS: A, C, E The correct answers reflect the coordinating role of the community psychiatric nurse case manager. Negotiating the cost of therapy and accompanying the patient to church services are interventions the nurse would not be expected to undertake. The patient can walk to the church services; the nurse can provide encouragement.

2. Which nursing comments are likely to help a patient to cope by addressing the mediators of stress? Select all that apply. a. A divorce, while stressful, can be the beginning of a new, better phase of life. b. You said you used to jog; getting back to aerobic exercise could be helpful. c. Journaling often promotes awareness of how experiences have affected people. d. Slowing your breathing by counting to three between breaths will calm you. e. Would a short-term loan make your finances less stressful? f. There is a support group for newly divorced persons in your neighborhood.

ANS: A, C, E, F Stress mediators are factors that can help persons cope by influencing how they perceive and respond to stressors; they include personality, social support, perceptions, and culture. Suggesting that a divorce may have positive as well as negative aspects helps the patient to alter perceptions of the stressor. Journaling increases self-awareness regarding how life experiences may have shaped how we perceive and respond to stress (or how our personality affects how we respond to stressors). A loan could help the patient by reducing the financial pressures. Participation in support groups is an excellent way to expand ones support network relative to specific issues. However, neither aerobic exercise nor breathing-control exercises, while helpful in other ways, affect stress mediators.

2. Which comments by an elderly person best indicate successful completion of the developmental task? Select all that apply. a. I am proud of my childrens successes in life. b. I should have given to community charities more often. c. My relationship with my father made life more difficult for me. d. My experiences in the war helped me appreciate the meaning of life. e. I often wonder what would have happened if I had chosen a different career.

ANS: A, D The developmental crisis for an elderly person relates to integrity versus despair. Pride in ones offspring indicates a sense of fulfillment. Recognition of the wisdom gained from difficult experiences (such as being in a war) indicates a sense of integrity. Blaming and regret indicate despair and unsuccessful resolution of the crisis.

2. Which findings are signs of a person who is mentally healthy? Select all that apply. a. Says, I have some weaknesses, but I feel Im important to my family and friends. b. Adheres strictly to religious beliefs of parents and family of origin. c. Spends all holidays alone watching old movies on television. d. Considers past experiences when deciding about the future. e. Experiences feelings of conflict related to changing jobs.

ANS: A, D, E Mental health is a state of well-being in which each individual is able to realize his or her own potential, cope with the normal stresses of life, work productively, and make a contribution to the community. Mental health provides people with the capacity for rational thinking, communication skills, learning, emotional growth, resilience, and self-esteem.

3. Which comments by an adult best indicate self-actualization? Select all that apply. a. I am content with a good book. b. I often wonder if I chose the right career. c. Sometimes I think about how my parents would have handled problems. d. Its important for our country to provide basic health care services for everyone. e. When I was lost at sea for 2 days, I gained an understanding of what is important.

ANS: A, D, E Self-actualized persons enjoy privacy, have a sense of democracy, and show positive outcomes associated with peak experiences. Self-doubt, defensiveness, and blaming are not consistent with self-actualization.

3. A novice psychiatric nurse has a parent with bipolar disorder. This nurse angrily recalls feelings of embarrassment about the parents behavior in the community. Select the best ways for this nurse to cope with these feelings.Select all that apply. a. Seek ways to use the understanding gained from childhood to help patients cope with their own illnesses. b. Recognize that these feelings are unhealthy. The nurse should try to suppress them when working with patients. c. Recognize that psychiatric nursing is not an appropriate career choice. Explore other nursing specialties. d. The nurse should begin new patient relationships by saying, My own parent had mental illness, so I accept it without stigma. e. Recognize that the feelings may add sensitivity to the nurses practice, but supervision is important.

ANS: A, E The nurse needs support to explore these feelings. An experienced psychiatric nurse is a resource that may be helpful. The knowledge and experience gained from the nurses relationship with a mentally ill parent may contribute sensitivity to compassionate practice. Self-disclosure and suppression are not adaptive coping strategies. The nurse should not give up on this area of practice without first seeking ways to cope with the memories.

19. A group has two more sessions before it ends. One member was previously vocal and has shown much progress but has now grown silent. What explanation most likely underlies this behavior? The silent member: a. has participated in the group and now has nothing more to offer. b. is having trouble dealing with feelings about termination of this group. c. wants to give quieter members a chance to talk in the remaining sessions. d. is engaging in attention-seeking behavior aimed at continuation of the group.

ANS: B A chief task during the termination phase of a group is to take what has been learned in group and transition to life without the group. The end of a group can be a significant loss for members, who may experience loss and grief and respond with sadness or anger. It is unlikely he would have nothing to say; at the very least, he could be responding to the comments of others even if not focusing on his own issues. He may wish to give quieter members a chance to talk, but again, this would not require or explain his complete silence. Some members, faced with only two remaining sessions, may be becoming more dominant under this pressure of time, but here too this is unlikely to lead a previously active participant to fall completely silent. The member is not attention-seeking.

13. Which characteristic would be more applicable to a community mental health nurse than to a nurse working in an operating room? a. Kindness b. Autonomy c. Compassion d. Professionalism

ANS: B A community mental health nurse often works autonomously. Kindness, compassion, and professionalism apply to both nurses.

10. A nurse introduces the matter of a contract during the first session with a new patient because contracts: a. specify what the nurse will do for the patient. b. spell out the participation and responsibilities of each party. c. indicate the feeling tone established between the participants. d. are binding and prevent either party from prematurely ending the relationship.

ANS: B A contract emphasizes that the nurse works with the patient rather than doing something for the patient. Working with is a process that suggests each party is expected to participate and share responsibility for outcomes. Contracts do not, however, stipulate roles or feeling tone, and premature termination is forbidden.

3. A patient is admitted to the psychiatric hospital. Which assessment finding best indicates that the patient has a mental illness? The patient: a. describes coping and relaxation strategies used when feeling anxious. b. describes mood as consistently sad, discouraged, and hopeless. c. can perform tasks attempted within the limits of own abilities. d. reports occasional problems with insomnia.

ANS: B A patient who reports having a consistently negative mood is describing a mood alteration. The incorrect options describe mentally healthy behaviors and common problems that do not indicate mental illness.

5. Select the example of a tort. a. The plan of care for a patient is not completed within 24 hours of the patients admission. b. A nurse gives a PRN dose of an antipsychotic drug to an agitated patient because the unit is short-staffed. c. An advanced practice nurse recommends hospitalization for a patient who is dangerous to self and others. d. A patients admission status changed from involuntary to voluntary after the patients hallucinations subside.

ANS: B A tort is a civil wrong against a person that violates his or her rights. Giving unnecessary medication for the convenience of staff controls behavior in a manner similar to secluding a patient; thus, false imprisonment is a possible charge. The other options do not exemplify torts.

8. Which patient statement would lead the nurse to suspect unsuccessful completion of the developmental task of infancy? a. I have very warm and close friendships. b. Im afraid to allow anyone to really get to know me. c. Im always absolutely right, so dont bother saying more. d. Im ashamed that I didnt do things correctly in the first place.

ANS: B According to Erikson, the developmental task of infancy is the development of trust. The correct response is the only statement clearly showing lack of ability to trust others. Warm, close relationships suggest the developmental task of infancy was successfully completed; rigidity and self-absorption are reflected in the belief one is always right; and shame for past actions suggests failure to resolve the crisis of initiative versus guilt.

24. Which type of group is a staff nurse with 2 months psychiatric experience best qualified to conduct? a. Psychodynamic/psychoanalytic group b. Medication education group c. Existential/Gestalt group d. Family therapy group

ANS: B All nurses receive information about patient teaching strategies and basic information about psychotropic medications, making a medication education group a logical group for a beginner to conduct. The other groups would need a leader with more education and experience.

17. Which level of prevention activities would a nurse in an emergency department employ most often? a. Primary b. Secondary c. Tertiary

ANS: B An emergency department nurse would generally see patients in crisis or with acute illness, so secondary prevention is used. Primary prevention involves preventing a health problem from developing, and tertiary prevention applies to rehabilitative activities.

10. Complete this analogy. NANDA: clinical judgment: NIC: _________________ a. patient outcomes b. nursing actions c. diagnosis d. symptoms

ANS: B Analogies show parallel relationships. NANDA, the North American Nursing Diagnosis Association, identifies diagnostic statements regarding human responses to actual or potential health problems. These statements represent clinical judgments. NIC (Nursing Interventions Classification) identifies actions provided by nurses that enhance patient outcomes. Nursing care activities may be direct or indirect.

6. As part of the stress response, the HPA axis is stimulated. Which structures make up this system? a. Hippocampus, parietal lobe, and amygdala b. Hypothalamus, pituitary gland, and adrenal glands c. Hind brain, pyramidal nervous system, and anterior cerebrum d. Hepatic artery, parasympathetic nervous system, and acoustic nerve

ANS: B As part of the physiologic response of stress, the hypothalamus stimulates the HPA axis, which is made up of the hypothalamus, pituitary gland, and adrenal glands.

21. During a support group, a patient diagnosed with schizophrenia says, Sometimes I feel sad that I will never have a good job like my brother. Then I dwell on it and maybe I should not. Select the nurse leaders best comment to facilitate discussion of this issue. a. It is often better to focus on our successes rather than our failures. b. How have others in the group handled painful feelings like these? c. Grieving for what is lost is a normal part of having a mental disorder. d. I wonder if you might also experience feelings of anger and helplessness.

ANS: B Asking others to share their experiences will facilitate discussion of an issue. Giving information may serve to close discussion of the issue because it sounds final. Suggesting a focus on the positives implies a discussion of the issue is not appropriate. Suggesting other possible feelings is inappropriate at this point, considering the patient has identified feelings of sadness and seems to have a desire to explore this feeling. Focusing on other feelings will derail discussion of the patients grief for his perceived lost potential.

15. A nurse asks a patient, If you had fever and vomiting for 3 days, what would you do? Which aspect of the mental status examination is the nurse assessing? a. Behavior b. Cognition c. Affect and mood d. Perceptual disturbances

ANS: B Assessing cognition involves determining a patients judgment and decision making. In this case, the nurse would expect a response of Call my doctor if the patients cognition and judgment are intact. If the patient responds, I would stop eating or I would just wait and see what happened, the nurse would conclude that judgment is impaired. The other options refer to other aspects of the examination.

7. Before assessing a new patient, a nurse is told by another health care worker, I know that patient. No matter how hard we work, there isnt much improvement by the time of discharge. The nurses responsibility is to: a. document the other workers assessment of the patient. b. assess the patient based on data collected from all sources. c. validate the workers impression by contacting the patients significant other. d. discuss the workers impression with the patient during the assessment interview.

ANS: B Assessment should include data obtained from both the primary and reliable secondary sources. The nurse, bearing in mind the possible effects of counter-transference, should evaluate biased assessments by others as objectively as possible.

28. A patient says, All my life Ive been surrounded by stupidity. Everything I buy breaks because the entire American workforce is incompetent. This patient is experiencing a: a. self-esteem deficit. b. cognitive distortion. c. deficit in motivation. d. deficit in love and belonging.

ANS: B Automatic thoughts, or cognitive distortions, are irrational and lead to false assumptions and misinterpretations.

30. A patient says, I always feel good when I wear a size 2 petite. Which type of cognitive distortion is evident? a. Disqualifying the positive b. Overgeneralization c. Catastrophizing d. Personalization

ANS: B Automatic thoughts, or cognitive distortions, are irrational and lead to false assumptions and misinterpretations. The stem offers an example of overgeneralization.

13. An adolescent hospitalized after a violent physical outburst tells the nurse, Im going to kill my father, but you cant tell anyone. Select the nurses best response. a. You are right. Federal law requires me to keep clinical information private. b. I am obligated to share that information with the treatment team. c. Those kinds of thoughts will make your hospitalization longer. d. You should share this thought with your psychiatrist.

ANS: B Breach of nurse-patient confidentiality does not pose a legal dilemma for nurses in these circumstances because a team approach to delivery of psychiatric care presumes communication of patient information to other staff members to develop treatment plans and outcome criteria. The patient should also know that the team has a duty to warn the father of the risk for harm.

7. Cortisol is released in response to a patient's prolonged stress. Which initial effect would the nurse expect to result from the increased cortisol level? a. Diuresis and electrolyte imbalance b. Focused and alert mental status c. Drowsiness and lethargy d. Restlessness and anxiety

ANS: B Cortisol is the primary stress hormone and is released in response to prolonged stress. Cortisol helps supply cells with amino acids and fatty acids for energy supply, as well as diverting glucose from muscles for use by the brain. As a result, the brain stays alert and focused. The distractors present effects that would not be expected.

1. A nurse assesses a confused older adult. The nurse experiences sadness and reflects, The patient is like one of my grandparentsso helpless. Which response is the nurse demonstrating? a. Transference b. Countertransference c. Catastrophic reaction d. Defensive coping reaction

ANS: B Countertransference is the nurses transference or response to a patient that is based on the nurses unconscious needs, conflicts, problems, or view of the world.

20. An advanced practice nurse observes a novice nurse expressing irritability regarding a patient with a long history of alcoholism and suspects the new nurse is experiencing countertransference. Which comment by the new nurse confirms this suspicion? a. This patient continues to deny problems resulting from drinking. b. My parents were alcoholics and often neglected our family. c. The patient cannot identify any goals for improvement. d. The patient said I have many traits like her mother.

ANS: B Countertransference occurs when the nurse unconsciously and inappropriately displaces onto the patient feelings and behaviors related to significant figures in the nurses past. In this instance, the new nurses irritability stems from relationships with parents. The distracters indicate transference or accurate analysis of the patients behavior.

8. A patient presents to the emergency department with mixed psychiatric symptoms. The admission nurse suspects the symptoms may be the result of a medical problem. Lab results show elevated BUN (blood urea nitrogen) and creatinine. What is the nurses next best action? a. Report the findings to the health care provider. b. Assess the patient for a history of renal problems. c. Assess the patients family history for cardiac problems. d. Arrange for the patients hospitalization on the psychiatric unit.

ANS: B Elevated BUN (blood urea nitrogen) and creatinine suggest renal problems. Renal dysfunction can often imitate psychiatric disorders. The nurse should further assess the patients history for renal problems and then share the findings with the health care provider.

17. Which scenario best demonstrates an example of eustress? An individual: a. loses a beloved family pet. b. prepares to take a one-week vacation to a tropical island with a group of close friends. c. receives a bank notice there were insufficient funds in their account for a recent rent payment. d. receives notification their current employer is experiencing financial problems and some workers will be terminated.

ANS: B Eustress is beneficial stress; it motivates people to develop skills they need to solve problems and meet personal goals. Positive life experiences produce eustress. Going on a tropical vacation is an exciting, relaxing experience and is an example of eustress. Losing the family pet, worrying about employment security, and having financial problems are examples of distress, a negative experience that drains energy and can lead to significant emotional problems.

7. Which nursing intervention demonstrates false imprisonment? a. A confused and combative patient says, Im getting out of here, and no one can stop me. The nurse restrains this patient without a health care providers order and then promptly obtains an order. b. A patient has been irritating and attention-seeking much of the day. A nurse escorts the patient down the hall saying, Stay in your room, or youll be put in seclusion. c. An involuntarily hospitalized patient with suicidal ideation runs out of the psychiatric unit. The nurse rushes after the patient and convinces the patient to return to the unit. d. An involuntarily hospitalized patient with homicidal ideation attempts to leave the facility. A nurse calls the security team and uses established protocols to prevent the patient from leaving.

ANS: B False imprisonment involves holding a competent person against his or her will. Actual force is not a requirement of false imprisonment. The individual needs only to be placed in fear of imprisonment by someone who has the ability to carry out the threat. If a patient is not competent (confused), then the nurse should act with beneficence. Patients admitted involuntarily should not be allowed to leave without permission of the treatment team.

14. A patient tells the nurse, I will never be happy until Im as successful as my older sister. The nurse asks the patient to reassess this statement and reframe it. Which reframed statement by the patient is most likely to promote coping? a. People should treat me as well as they treat my sister. b. I can find contentment in succeeding at my own job level. c. I wont be happy until I make as much money as my sister. d. Being as smart or clever as my sister isnt really important.

ANS: B Finding contentment within ones own work, even when it does not involve success as others might define it, is likely to lead to a reduced sense of distress about achievement level. It speaks to finding satisfaction and happiness without measuring the self against another person. Focusing on salary is simply a more specific way of being as successful as the sister, which would not promote coping. Expecting others to treat her as they do her sister is beyond her control. Dismissing the sisters cleverness as unimportant indicates that the patient continues to feel inferior to the sibling.

10. A soldier returned home from active duty in a combat zone in Afghanistan and was diagnosed with post-traumatic stress disorder (PTSD). The soldier says, "If there's a loud noise at night, I get under my bed because I think we're getting bombed." What type of experience has the soldier described? a. Illusion b. Flashback c. Nightmare d. Auditory hallucination

ANS: B Flashbacks are dissociative reactions in which an individual feels or acts as if the traumatic event were recurring. Illusions are misinterpretations of stimuli; although the experience is similar, the more accurate term is flashback because of the diagnosis of PTSD. Auditory hallucinations have no external stimuli. Nightmares commonly accompany PTSD, but this experience is stimulated by an actual environmental sound.

14. A nurse wants to demonstrate genuineness with a patient diagnosed with schizophrenia. The nurse should: a. restate what the patient says. b. use congruent communication strategies. c. use self-revelation in patient interactions. d. consistently interpret the patients behaviors.

ANS: B Genuineness is a desirable characteristic involving awareness of ones own feelings as they arise and the ability to communicate them when appropriate. The incorrect options are undesirable in a therapeutic relationship.

12. A black patient says to a white nurse, Theres no sense talking. You wouldnt understand because you live in a white world. The nurses best action would be to: a. explain, Yes, I do understand. Everyone goes through the same experiences. b. say, Please give an example of something you think I wouldnt understand. c. reassure the patient that nurses interact with people from all cultures. d. change the subject to one that is less emotionally disturbing.

ANS: B Having the patient speak in specifics rather than globally will help the nurse understand the patients perspective. This approach will help the nurse engage the patient. Reassurance and changing the subject are not therapeutic techniques.

4. A recent immigrant from Honduras comes to the clinic with a family member who has been a U.S. resident for 10 years. The family member says, The immigration to America has been very difficult. Considering cultural background, which expression of stress by this patient would the nurse expect? a. Motor restlessness b. Somatic complaints c. Memory deficiencies d. Sensory perceptual alterations

ANS: B Honduras is in Central America. Many people from Central American cultures express distress in somatic terms. The other options are not specific to this patients cultural background and are less likely to be observed in persons from Central America.

1. A staff nurse completes orientation to a psychiatric unit. This nurse may expect an advanced practice nurse to perform which additional intervention? a. Conduct mental health assessments. b. Prescribe psychotropic medication. c. Establish therapeutic relationships. d. Individualize nursing care plans.

ANS: B In most states, prescriptive privileges are granted to masters-prepared nurse practitioners who have taken special courses on prescribing medication. The nurse prepared at the basic level is permitted to perform mental health assessments, establish relationships, and provide individualized care planning.

2. A patient diagnosed with schizophrenia tells the nurse, The CIA is monitoring us through the fluorescent lights in this room. Be careful what you say. Which response by the nurse would be most therapeutic? a. Lets talk about something other than the CIA. b. It sounds like youre concerned about your privacy. c. The CIA is prohibited from operating in health care facilities. d. You have lost touch with reality, which is a symptom of your illness.

ANS: B It is important not to challenge the patients beliefs, even if they are unrealistic. Challenging undermines the patients trust in the nurse. The nurse should try to understand the underlying feelings or thoughts the patients message conveys. The correct response uses the therapeutic technique of reflection. The other comments are non-therapeutic. Asking to talk about something other than the concern at hand is changing the subject. Saying that the CIA is prohibited from operating in health care facilities gives false reassurance. Stating that the patient has lost touch with reality is truthful, but uncompassionate.

20. A patient diagnosed with schizophrenia believes a local minister stirred evil spirits. The patient threatens to bomb a local church. The psychiatrist notifies the minister. Select the answer with the correct rationale. The psychiatrist: a. released information without proper authorization. b. demonstrated the duty to warn and protect. c. violated the patients confidentiality. d. avoided charges of malpractice.

ANS: B It is the health care professionals duty to warn or notify an intended victim after a threat of harm has been made. Informing a potential victim of a threat is a legal responsibility of the health care professional. It is not a violation of confidentiality.

9. A patient experiencing significant stress associated with a disturbing new medical diagnosis asks the nurse, Do you think saying a prayer would help? Select the nurses best answer. a. It could be that prayer is your only hope. b. You may find prayer gives comfort and lowers your stress. c. I can help you feel calmer by teaching you meditation exercises. d. We do not have evidence that prayer helps, but it wouldnt hurt.

ANS: B Many patients find that spiritual measures, including prayer, are helpful in mediating stress. Studies have shown that spiritual practices can enhance the sense of well-being. When a patient suggests a viable means of reducing stress, it should be supported by the nurse. Indicating that prayer is the patients only hope is pessimistic and would cause further distress. Suggesting meditation or other alternatives to prayer implies that the nurse does not think prayer would be effective.

3. The patient says, My marriage is just great. My spouse and I always agree. The nurse observes the patients foot moving continuously as the patient twirls a shirt button. The conclusion the nurse can draw is that the patients communication is: a. clear. b. mixed. c. precise. d. inadequate.

ANS: B Mixed messages involve the transmission of conflicting or incongruent messages by the speaker. The patients verbal message that all was well in the relationship was modified by the nonverbal behaviors denoting anxiety. Data are not present to support the choice of the verbal message being clear, explicit, or inadequate.

11. A nurse receives these three phone calls regarding a newly admitted patient. The psychiatrist wants to complete an initial assessment. An internist wants to perform a physical examination. The patients attorney wants an appointment with the patient. The nurse schedules the activities for the patient. Which role has the nurse fulfilled? a. Advocate b. Case manager c. Milieu manager d. Provider of care

ANS: B Nurses on psychiatric units routinely coordinate patient services, serving as case managers as described in this scenario. The role of advocate would require the nurse to speak out on the patients behalf. The role of milieu manager refers to maintaining a therapeutic environment. Provider of care refers to giving direct care to the patient.

5. A nurse inspects an inpatient psychiatric unit and finds that exits are free of obstructions, no one is smoking, and the janitors closet is locked. These observations relate to: a. coordinating care of patients. b. management of milieu safety. c. management of the interpersonal climate. d. use of therapeutic intervention strategies.

ANS: B Nursing staff are responsible for all aspects of milieu management. The observations mentioned in this question directly relate to the safety of the unit. The other options, although part of the nurses concerns, are unrelated to the observations cited.

3. Two hospitalized patients fight whenever they are together. During a team meeting, a nurse asserts that safety is of paramount importance, so treatment plans should call for both patients to be secluded to keep them from injuring each other. This assertion: a. reinforces the autonomy of the two patients. b. violates the civil rights of both patients. c. represents the intentional tort of battery. d. correctly places emphasis on safety.

ANS: B Patients have a right to treatment in the least restrictive setting. Safety is important, but less restrictive measures should be tried first. Unnecessary seclusion may result in a charge of false imprisonment. Seclusion violates the patients autonomy. The principle by which the nurse is motivated is beneficence, not justice. The tort represented is false imprisonment.

23. A patient experiencing psychosis asks a psychiatric technician, Whats the matter with me? The technician replies, Nothing is wrong with you. You just need to use some self-control. The nurse who overheard the exchange should take action based on: a. the technicians unauthorized disclosure of confidential clinical information. b. violation of the patients right to be treated with dignity and respect. c. the nurses obligation to report caregiver negligence. d. the patients right to social interaction.

ANS: B Patients have the right to be treated with dignity and respect. The technicians comment disregards the seriousness of the patients illness. The Code of Ethics for Nurses requires intervention. Patient emotional abuse has been demonstrated, not negligence. An interaction with the technician is not an aspect of social interaction. The technician did not disclose clinical information.

26. Which assessment finding for a patient in the community deserves priority intervention by the psychiatric nurse? The patient: a. receives Social Security disability income plus a small check from a trust fund every month. b. was absent from two of six planned Alcoholics Anonymous meetings in the past 2 weeks. c. lives in an apartment with two patients who attend partial hospitalization programs. d. has a sibling who was recently diagnosed with a mental illness.

ANS: B Patients who use alcohol or illegal substances often become medication noncompliant. Medication noncompliance, along with the disorganizing influence of substances on cellular brain function, promotes relapse. The distracters do not suggest problems.

9. A nurse prepares to administer a scheduled injection of haloperidol decanoate (Haldol depot) to an outpatient with schizophrenia. As the nurse swabs the site, the patient shouts, Stop! I dont want to take that medicine anymore. I hate the side effects. Select the nurses best action. a. Assemble other staff for a show of force and proceed with the injection, using restraint if necessary. b. Stop the medication administration procedure and say to the patient, Tell me more about the side effects youve been having. c. Proceed with the injection but explain to the patient that there are medications that will help reduce the unpleasant side effects. d. Say to the patient, Since Ive already drawn the medication in the syringe, Im required to give it, but lets talk to the doctor about delaying next months dose.

ANS: B Patients with mental illness retain their civil rights unless there is clear, cogent, and convincing evidence of dangerousness. The patient in this situation presents no evidence of dangerousness. The nurse, as an advocate and educator, should seek more information about the patients decision and not force the medication.

1. A new staff nurse completes an orientation to the psychiatric unit. This nurse will expect to ask an advanced practice nurse to perform which action for patients? a. Perform mental health assessment interviews. b. Prescribe psychotropic medication. c. Establish therapeutic relationships. d. Individualize nursing care plans.

ANS: B Prescriptive privileges are granted to masters-prepared nurse practitioners who have taken special courses on prescribing medication. The nurse prepared at the basic level performs mental health assessments, establishes relationships, and provides individualized care planning.

24. Which activity is appropriate for a nurse engaged exclusively in community-based primary prevention? a. Medication follow-up b. Teaching parenting skills c. Substance abuse counseling d. Making a referral for family therapy

ANS: B Primary prevention activities are directed to healthy populations to provide information for developing skills that promote mental health. The distracters represent secondary or tertiary prevention activities.

16. Select the example of primary prevention. a. Assisting a person diagnosed with a serious mental illness to fill a pill-minder b. Helping school-age children identify and describe normal emotions c. Leading a psychoeducational group in a community care home d. Medicating an acutely ill patient who assaulted a staff person

ANS: B Primary preventions are directed at healthy populations with a goal of preventing health problems from occurring. Helping school-age children describe normal emotions people experience promotes coping, a skill that is needed throughout life. Assisting a person with serious and persistent mental illness to fill a pill-minder is an example of tertiary prevention. Medicating an acutely ill patient who assaulted a staff person is a secondary prevention. Leading a psychoeducational group in a community care home is an example of tertiary prevention.

13. Which entry in the medical record best meets the requirement for problem-oriented charting? a. A: Pacing and muttering to self. P: Sensory perceptual alteration related to internal auditory stimulation. I: Given fluphenazine HCL (Prolixin) 2.5 mg po at 0900 and went to room to lie down. E: Calmer by 0930. Returned to lounge to watch TV. b. S: States, I feel like Im ready to blow up. O: Pacing hall, mumbling to self. A: Auditory hallucinations. P: Offer haloperidol (Haldol) 2 mg po. I: Haloperidol (Haldol) 2 mg po given at 0900. E: Returned to lounge at 0930 and quietly watched TV. c. Agitated behavior. D: Patient muttering to self as though answering an unseen person. A: Given haloperidol (Haldol) 2 mg po and went to room to lie down. E: Patient calmer. Returned to lounge to watch TV. d. Pacing hall and muttering to self as though answering an unseen person. haloperidol (Haldol) 2 mg po administered at 0900 with calming effect in 30 minutes. Stated, Im no longer bothered by the voices.

ANS: B Problem-oriented documentation uses the first letter of key words to organize data: S for subjective data, O for objective data, A for assessment, P for plan, I for intervention, and E for evaluation. The distracters offer examples of PIE charting, focus documentation, and narrative documentation.

23. QSEN refers to: a. Qualitative Standardized Excellence in Nursing b. Quality and Safety Education for Nurses c. Quantitative Effectiveness in Nursing d. Quick Standards Essential for Nurses

ANS: B QSEN represents national initiatives centered on patient safety and quality. The primary goal of QSEN is to prepare future nurses with the knowledge, skills, and attitudes to increase the quality, care, and safety in the health care setting in which they work.

12. Which action by a nurse constitutes a breach of a patients right to privacy? a. Documenting the patients daily behavior during hospitalization b. Releasing information to the patients employer without consent c. Discussing the patients history with other staff during care planning d. Asking family to share information about a patients pre-hospitalization behavior

ANS: B Release of information without patient authorization violates the patients right to privacy. The other options are acceptable nursing practices.

17. A community psychiatric nurse assesses that a patient diagnosed with a mood disorder is more depressed than on the previous visit a month ago; however, the patient says, "I feel the same." Which intervention supports the nurse's assessment while preserving the patient's autonomy? a. Arrange for a short hospitalization. b. Schedule weekly clinic appointments. c. Refer the patient to the crisis intervention clinic. d. Call the family and ask them to observe the patient closely.

ANS: B Scheduling clinic appointments at shorter intervals will give the opportunity for more frequent assessment of symptoms and allow the nurse to use early intervention. If the patient does not admit to having a crisis or problem, a referral would be useless. The remaining options may produce unreliable information, violate the patient's privacy, and waste scarce resources.

15. A patient says, One result of my chronic stress is that I feel so tired. I usually sleep from 11:00 PM to 6:30 AM. I started setting my alarm to give me an extra 30 minutes of sleep each morning, but I dont feel any better and Im rushed for work. Which nursing response would best address the patients concerns? a. You may need to speak to your doctor about taking a sedative to help you sleep. b. Perhaps going to bed a half-hour earlier would work better than sleeping later. c. A glass of wine in the evening might take the edge off and help you to rest. d. Exercising just before retiring for the night may help you to sleep better.

ANS: B Sleeping later in the morning may disturb circadian rhythms and in this case is adding, rather than reducing, stress. Going to bed earlier and arising at the usual time alleviates fatigue more effectively. Sedatives may offer some benefit but are a short-term intervention with potential side effects, and other nonpharmacological interventions might work as well or better. Exercise earlier in the evening could induce tiredness and ease the process of falling asleep, but doing so right before bedtime would stimulate and interfere with sleep. Alcohol is sedating but potentially addictive; encouraging its use could increase the risk of using alcohol maladaptively as a response to stress in general.

16. A nurse wants to enhance growth of a patient by showing positive regard. The nurses action most likely to achieve this goal is: a. making rounds daily. b. staying with a tearful patient. c. administering medication as prescribed. d. examining personal feelings about a patient.

ANS: B Staying with a crying patient offers support and shows positive regard. Administering daily medication and making rounds are tasks that could be part of an assignment and do not necessarily reflect positive regard. Examining feelings regarding a patient addresses the nurses ability to be therapeutic.

2. Which organization actively seeks to reduce the stigma associated with mental illness through public presentations such as "In Our Own Voice" (IOOV)? a. American Psychiatric Association (APA) b. National Alliance on Mental Illness (NAMI) c. United States Department of Health and Human Services (USDHHS) d. North American Nursing Diagnosis Association International (NANDA-I)

ANS: B Stigma represents the bias and prejudice commonly held regarding mental illness. NAMI actively seeks to dispel misconceptions about mental illness. NANDA-I defines approved nursing diagnoses. The APA publishes the DSM 5. The USDHHS regulates and administers health policies.

2. A patient diagnosed with liver failure has been on the transplant waiting list 8 months. The patient says, "Why is it taking so long to have the surgery? Maybe I'm meant to die for all the bad things I've done." The nurse should document the patient's comment in which section of the assessment? a. Physical b. Spiritual c. Financial d. Psychological

ANS: B Stress can be evident in a person's spirituality. This patient's comment indicates questioning of one's place in the universe and consequences for wrongdoing, both of which are elements of spirituality. Stress can be related to psychological, physical, or psychosocial well-being, but spirituality is the best answer.

29. A patient is fearful of riding on elevators. The therapist first rides an escalator with the patient. The therapist and patient then stand in an elevator with the door open for five minutes and later with the elevator door closed for five minutes. Which technique has the therapist used? a. Classic psychoanalytic therapy b. Systematic desensitization c. Rational emotive therapy d. Biofeedback

ANS: B Systematic desensitization is a form of behavior modification therapy that involves the development of behavior tasks customized to the patients specific fears. These tasks are presented to the patient while using learned relaxation techniques. The patient is incrementally exposed to the fear.

8. A nurse wants to find a description of diagnostic criteria for anxiety disorders. Which resource would have the most complete information? a. Nursing Outcomes Classification (NOC) b. Diagnostic and Statistical Manual of Mental Disorders (DSM-5) c. The ANAs Psychiatric-Mental Health Nursing Scope and Standards of Practice d. International Statistical Classification of Diseases and Related Health Problems (ICD-10)

ANS: B The DSM-5 details the diagnostic criteria for psychiatric clinical conditions. The other references are good resources but do not define the diagnostic criteria.

7. A nurse encounters an unfamiliar psychiatric disorder on a new patients admission form. Which resource should the nurse consult to determine criteria used to establish this diagnosis? a. International Statistical Classification of Diseases and Related Health Problems (ICD-10) b. Diagnostic and Statistical Manual of Mental Disorders (DSM-5) c. A behavioral health reference manual d. Wikipedia

ANS: B The DSM-5 gives the criteria used to diagnose each mental disorder. The distracters may not contain diagnostic criteria for a psychiatric illness.

2. A 26-month-old displays negative behavior, refuses toilet training, and often says, No! Which stage of psychosexual development is evident? a. Oral b. Anal c. Phallic d. Genital

ANS: B The anal stage occurs from age 1 to 3 years and has as its focus toilet training and learning to delay immediate gratification. The oral stage occurs between birth and 1 year. The phallic stage occurs between 3 and 5 years, and the genital stage occurs between age 13 and 20 years.

11. The nurse is planning a new sexuality group for patients. Which location would best enhance the effectiveness of this group? a. The hospital auditorium b. A small conference room c. A common area, such as a day room d. The corner of the music therapy room

ANS: B The conference room would provide a quiet, private area with few distractions, separate from other patient areas and effective for teaching and learning about a private topic. The auditorium is too large, and members anxiety or lack of trust might lead them to spread out too far from each other, interfering with group process. The day room and the music therapy room are too busy and exposed, reducing privacy and increasing distractions.

5. Which assessment finding most clearly indicates that a patient may be experiencing a mental illness? The patient: a. reports occasional sleeplessness and anxiety. b. reports a consistently sad, discouraged, and hopeless mood. c. is able to describe the difference between as if and for real. d. perceives difficulty making a decision about whether to change jobs.

ANS: B The correct response describes a mood alteration, which reflects mental illness. The distracters describe behaviors that are mentally healthy or within the usual scope of human experience.

9. A soldier returns to the United States from active duty in a combat zone in Afghanistan. The soldier is diagnosed with post-traumatic stress disorder (PTSD). Which comment by the soldier requires the nurse's immediate attention? a. "It's good to be home. I missed my family and friends." b. "I saw my best friend get killed by a roadside bomb. It should have been me instead." c. "Sometimes I think I hear bombs exploding, but it's just the noise of traffic in my hometown." d. "I want to continue my education but I'm not sure how I will fit in with other college students."

ANS: B The correct response indicates the soldier is thinking about death and feeling survivor's guilt. These emotions may accompany suicidal ideation, which warrants the nurse's follow-up assessment. Suicide is a high risk among military personnel diagnosed with PTSD. One distractor indicates flashbacks, which is common with individuals with PTSD but is not solely indicative of further problems. The other distractors are normal emotions associated with returning home and change.

25. As a patient diagnosed with a mental illness is being discharged from a facility, a nurse invites the patient to the annual staff picnic. What is the best analysis of this scenario? a. The invitation facilitates dependency on the nurse. b. The nurses action blurs the boundaries of the therapeutic relationship. c. The invitation is therapeutic for the patients diversional activity deficit. d. The nurses action assists the patients integration into community living.

ANS: B The invitation creates a social relationship rather than a therapeutic relationship.

21. Which component of treatment of mental illness is specifically recognized by Quality and Safety Education for Nurses (QSEN)? a. All genomes are unique. b. Care is centered on the patient. c. Healthy development is vital to mental health. d. Recovery occurs on a continuum from illness to health.

ANS: B The key areas of care promoted by QSEN are patient-centered care, teamwork and collaboration, evidence-based practice, quality improvement, safety, and informatics.

15. During a therapy group that uses existential/Gestalt theory, patients shared feelings that occurred at the time of their admission. After a brief silence, one member says, Several people have described feeling angry. I would like to hear from members who had other feelings. Which group role is evident by this comment? a. Energizer b. Encourager c. Compromiser d. Self-confessor

ANS: B The member is filling the role of encourager by acknowledging those who have contributed and encouraging input from others. An energizer encourages the group to make decisions or take an action. The compromiser focuses on reducing or resolving conflict to preserve harmony. A self-confessor verbalizes feelings or observations unrelated to the group.

2. A nurse finds a psychiatric advance directive in the medical record of a patient experiencing psychosis. The directive was executed during a period when the patient was stable and competent. The nurse should: a. review the directive with the patient to ensure it is current. b. ensure that the directive is respected in treatment planning. c. consider the directive only if there is a cardiac or respiratory arrest. d. encourage the patient to revise the directive in light of the current health problem.

ANS: B The nurse has an obligation to honor the right to self-determination. An advanced psychiatric directive supports that goal. Since the patient is currently psychotic, the terms of the directive now apply.

18. A nurse uses Peplau's interpersonal therapy while working with an anxious, withdrawn patient. Interventions should focus on: a. changing the patient's perceptions about self. b. improving the patient's interactional skills. c. using medications to relieve anxiety. d. reinforcing specific behaviors.

ANS: B The nurse-patient relationship is structured to provide a model for adaptive interpersonal relationships that can be generalized to others. Changing the patient's perceptions about his- or herself would be appropriate for cognitive therapy. Reinforcing specific behaviors would be used in behavioral therapy. Using medications is the focus of biological therapy.

18. A nurse influenced by Peplaus interpersonal theory works with an anxious, withdrawn patient. Interventions should focus on: a. rewarding desired behaviors. b. use of assertive communication. c. changing the patients self-concept. d. administering medications to relieve anxiety.

ANS: B The nurse-patient relationship is structured to provide a model for adaptive interpersonal relationships that can be generalized to others. Helping the patient learn to use assertive communication will improve the patients interpersonal relationships. The distracters apply to theories of cognitive, behavioral, and biological therapy.

27. An aide in a psychiatric hospital says to the nurse, We dont have time every day to help each patient complete a menu selection. Lets tell dietary to prepare popular choices and send them to our unit. Select the nurses best response. a. Thanks for the suggestion, but that idea may not work because so many patients take MAOI (monoamine oxidase inhibitor) antidepressants. b. Thanks for the idea, but its important to treat patients as individuals. Giving choices is one way we can respect patients individuality. c. Thank you for the suggestion, but the patients bill of rights requires us to allow patients to select their own diet. d. Thank you. That is a very good idea. It will make meal preparation easier for the dietary department.

ANS: B The nurses response to the aide should recognize patients rights to be treated with dignity and respect as well as promote autonomy. This response also shows respect for the aide and fulfills the nurses obligation to provide supervision of unlicensed personnel. The incorrect responses have flawed rationale or do not respect patients as individuals.

13. A student nurse says, I dont need to interact with my patients. I learn what I need to know by observation. An instructor can best interpret the nursing implications of Sullivans theory to this student by responding: a. Interactions are required in order to help you develop therapeutic communication skills. b. Nurses cannot be isolated. We must interact to provide patients with opportunities to practice interpersonal skills. c. Observing patient interactions will help you formulate priority nursing diagnoses and appropriate interventions. d. It is important to pay attention to patients behavioral changes, because these signify adjustments in personality.

ANS: B The nurses role includes educating patients and assisting them in developing effective interpersonal relationships. Mutuality, respect for the patient, unconditional acceptance, and empathy are cornerstones of Sullivans theory. The nurse who does not interact with the patient cannot demonstrate these cornerstones. Observations provide only objective data. Priority nursing diagnoses usually cannot be accurately established without subjective data from the patient. The other distracters relate to Maslow and behavioral theory.

17. The parent of a child diagnosed with schizophrenia tearfully asks the nurse, What could I have done differently to prevent this illness? Select the nurses best response. a. Although schizophrenia results from impaired family relationships, try not to feel guilty. No one can predict how a child will respond to parental guidance. b. Schizophrenia is a biological illness resulting from changes in how the brain and nervous system function. You are not to blame for your childs illness. c. There is still hope. Changing your parenting style can help your child learn to cope effectively with the environment. d. Most mental illnesses result from genetic inheritance. Your genes are more at fault than your parenting.

ANS: B The parents comment suggests feelings of guilt or inadequacy. The nurses response should address these feelings as well as provide information. Patients and families need reassurance that the major mental disorders are biological in origin and are not the fault of parents. One distracter places the burden of having faulty genes on the shoulders of the parents. The other distracters are neither wholly accurate nor reassuring.

5. A patient nervously says, Financial problems are stressing my marriage. Ive heard rumors about cutbacks at work; I am afraid I might get laid off. The patients pulse is 112/minute; respirations are 26/minute; and blood pressure is 166/88. Which nursing intervention will the nurse implement? a. Advise the patient, Go to sleep 30 to 60 minutes earlier each night to increase rest. b. Direct the patient in slow and deep breathing via use of a positive, repeated word. c. Suggest the patient consider that a new job might be better than the present one. d. Tell the patient, Relax by spending more time playing with your pet.

ANS: B The patient is responding to stress with increased arousal of the sympathetic nervous system, as evident by elevated vital signs. These will have a negative effect on his health and increase his perception of being anxious and stressed. Stimulating the parasympathetic nervous system will counter the sympathetic nervous systems arousal, normalizing these vital-sign changes and reducing the physiological demands stress is placing on his body. Other options do not address his physiological response pattern as directly or immediately.

23. A patient in a detoxification unit asks, What good it will do to go to Alcoholics Anonymous and talk to other people with the same problem? The nurses best response would be to explain that self-help groups such as AA provide opportunities for: a. newly discharged alcoholics to learn about the disease of alcoholism. b. people with common problems to share their experiences with alcoholism and recovery. c. patients with alcoholism to receive insight-oriented treatment about the etiology of their disease. d. professional counselors to provide guidance to individuals recovering from alcoholism.

ANS: B The patient needs basic information about the purpose of a self-help group. The basis of self-help groups is sharing by individuals with similar problems. Self-help is based on the belief that an individual with a problem can be truly understood and helped only by others who have the same problem. The other options fail to address this or provide incorrect information.

17. A nurse wants to assess an adult patients recent memory. Which question would best yield the desired information? a. Where did you go to elementary school? b. What did you have for breakfast this morning? c. Can you name the current president of the United States? d. A few minutes ago, I told you my name. Can you remember it?

ANS: B The patients recall of a meal provides evidence of recent memory. Two incorrect responses are useful to assess immediate and remote memory. The other distracter assesses the patients fund of knowledge.

2. When a nursing student expresses concerns about how mental health nurses lose all their nursing skills, the best response by the mental health nurse is: a. Psychiatric nurses practice in safer environments than other specialties. Nurse-to-patient ratios must be better because of the nature of the patients problems. b. Psychiatric nurses use complex communication skills as well as critical thinking to solve multidimensional problems. I am challenged by those situations. c. Thats a misconception. Psychiatric nurses frequently use high technology monitoring equipment and manage complex intravenous therapies. d. Psychiatric nurses do not have to deal with as much pain and suffering as medical-surgical nurses do. That appeals to me.

ANS: B The practice of psychiatric nursing requires a different set of skills than medical-surgical nursing, though there is substantial overlap. Psychiatric nurses must be able to help patients with medical as well as mental health problems, reflecting the holistic perspective these nurses must have. Nurse-patient ratios and workloads in psychiatric settings have increased, just like other specialties. Psychiatric nursing involves clinical practice, not just documentation. Psychosocial pain and suffering are as real as physical pain and suffering.

19. A patient had psychotherapy weekly for 5 months. The therapist used free association, dream analysis, and facilitated transference to help the patient understand conflicts and foster change. Select the term that applies to this method. a. Rational-emotive behavior therapy b. Psychodynamic psychotherapy c. Cognitive-behavioral therapy d. Operant conditioning

ANS: B The techniques are aspects of psychodynamic psychotherapy. The distracters use other techniques.

21. After formulating the nursing diagnoses for a new patient, what is a nurses next action? a. Designing interventions to include in the plan of care b. Determining the goals and outcome criteria c. Implementing the nursing plan of care d. Completing the spiritual assessment

ANS: B The third step of the nursing process is planning and outcome identification. Outcomes cannot be determined until the nursing assessment is complete and nursing diagnoses have been formulated.

1. A parent says, My 2-year-old child refuses toilet training and shouts No! when given directions. What do you think is wrong? Select the nurses best reply. a. Your child needs firmer control. It is important to set limits now. b. This is normal for your childs age. The child is striving for independence. c. There may be developmental problems. Most children are toilet trained by age 2. d. Some undesirable attitudes are developing. A child psychologist can help you develop a plan.

ANS: B This behavior is typical of a child around the age of 2 years, whose developmental task is to develop autonomy. The distracters indicate the childs behavior is abnormal.

10. During the first interview with a parent whose child died in a car accident, the nurse feels empathic and reaches out to take the patients hand. Select the correct analysis of the nurses behavior. a. It shows empathy and compassion. It will encourage the patient to continue to express feelings. b. The gesture is premature. The patients cultural and individual interpretation of touch is unknown. c. The patient will perceive the gesture as intrusive and overstepping boundaries. d. The action is inappropriate. Psychiatric patients should not be touched.

ANS: B Touch has various cultural and individual interpretations. Nurses should refrain from using touch until an assessment can be made regarding the way in which the patient will perceive touch. The other options present prematurely drawn conclusions.

24. A patient says to the nurse, My father has been dead for over 10 years, but talking to you is almost as comforting as the talks he and I had when I was a child. Which term applies to the patients comment? a. Superego b. Transference c. Reality testing d. Counter-transference

ANS: B Transference refers to feelings a patient has toward the health care workers that were originally held toward significant others in his or her life. Counter-transference refers to unconscious feelings that the health care worker has toward the patient. The superego represents the moral component of personality; it seeks perfection.

19. The relationship between a nurse and patient as it relates to status and power is best described by which term? a. Symmetric b. Complementary c. Incongruent d. Paralinguistic

ANS: B When a difference in power exists, as between a student and teacher or between a nurse and patient, the relationship is said to be complementary. Symmetrical relationships exist between individuals of like or equal status. Incongruent and paralinguistic are not terms used to describe relationships.

1. Which descriptors exemplify consistency regarding nurse-patient relationships? Select all that apply. a. Encouraging a patient to share initial impressions of staff b. Having the same nurse care for a patient on a daily basis c. Providing a schedule of daily activities to a patient d. Setting a time for regular sessions with a patient e. Offering solutions to a patients problems

ANS: B, C, D Consistency implies predictability. Having the same nurse see the patient daily and provide a daily schedule of patient activities and a set time for regular sessions will help a patient predict what will happen during each day and develop a greater degree of security and comfort. Encouraging a patient to share initial impressions of staff and giving advice are not related to consistency and would not be considered a therapeutic intervention.

1. A patient cries as the nurse explores the patients feelings about the death of a close friend. The patient sobs, I shouldnt be crying like this. It happened a long time ago. Which responses by the nurse facilitate communication? Select all that apply. a. Why do you think you are so upset? b. I can see that you feel sad about this situation. c. The loss of a close friend is very painful for you. d. Crying is a way of expressing the hurt you are experiencing. e. Lets talk about something else because this subject is upsetting you.

ANS: B, C, D Reflecting (I can see that you feel sad, This is very painful for you) and giving information (Crying is a way of expressing hurt) are therapeutic techniques. Why questions often imply criticism or seem intrusive or judgmental. They are difficult to answer. Changing the subject is a barrier to communication.

4. A patient is very suspicious and states, The FBI has me under surveillance. Which strategies should a nurse use when gathering initial assessment data about this patient? Select all that apply. a. Tell the patient that medication will help this type of thinking. b. Ask the patient, Tell me about the problem as you see it. c. Seek information about when the problem began. d. Tell the patient, Your ideas are not realistic. e. Reassure the patient, You are safe here.

ANS: B, C, E During the assessment interview, the nurse should listen attentively and accept the patients statements in a nonjudgmental way. Because the patient is suspicious and fearful, reassuring safety may be helpful, although trust is unlikely so early in the relationship. Saying that medication will help or telling the patient that the ideas are not realistic will undermine development of trust between the nurse and patient.

4. Which activities represent the art of nursing? Select all that apply. a. Administering medications on time to a group of patients b. Listening to a new widow grieve her husbands death c. Helping a patient obtain groceries from a food bank d. Teaching a patient about a new medication e. Holding the hand of a frightened patient

ANS: B, C, E Peplau described the science and art of professional nursing practice. The art component of nursing consists of the care, compassion, and advocacy nurses provide to enhance patient comfort and well-being. The science component of nursing involves the application of knowledge to understand a broad range of human problems and psychosocial phenomena, intervening to relieve patients suffering and promote growth.

3. A patient in the emergency department says, Voices say someone is stalking me. They want to kill me because I developed a cure for cancer. I have a knife and will stab anyone who is a threat. Which aspects of the patients mental health have the greatest and most immediate concern to the nurse? Select all that apply. a. Happiness b. Appraisal of reality c. Control over behavior d. Effectiveness in work e. Healthy self-concept

ANS: B, C, E The aspects of mental health of greatest concern are the patients appraisal of and control over behavior. The appraisal of reality is inaccurate. There are auditory hallucinations, delusions of persecution, and delusions of grandeur. In addition, the patients control over behavior is tenuous, as evidenced by the plan to stab anyone who seems threatening. A healthy self-concept is lacking, as evidenced by the delusion of grandeur. Data are not present to suggest that the other aspects of mental health (happiness and effectiveness in work) are of immediate concern.

1. The next-to-last meeting of an interpersonal therapy group is taking place. The leader should take which actions? Select all that apply. a. Support appropriate expressions of disagreement by the groups members. b. Facilitate discussion and resolution of feelings about the end of the group. c. Encourage members to reflect on their progress and that of the group itself. d. Remind members of the groups norms and rules, emphasizing confidentiality. e. Help members identify goals they would like to accomplish after the group ends. f. Promote the identification and development of new options for solving problems.

ANS: B, C, E The goals for the termination phase of groups are to prepare the group for separation, resolve related feelings, and prepare each member for the future. Contributions and accomplishments of members are elicited, post-group goals are identified, and feelings about the groups ending are discussed. Group norms are the focus of the orientation phase, and conflict and problem solving are emphasized in the working phase.

3. The nurse wishes to use guided imagery to help a patient relax. Which comments would be appropriate to include in the guided imagery script? Select all that apply. a. Imagine others treating you the way they should, the way you want to be treated b. With each breath, you feel calmer, more relaxed, almost as if you are floating c. You are alone on a beach, the sun is warm, and you hear only the sound of the surf d. You have taken control, nothing can hurt you now. Everything is going your way e. You have grown calm, your mind is still, there is nothing to disturb your well-being f. You will feel better as work calms down, as your boss becomes more understanding

ANS: B, C, E The intent of guided imagery to assist patients manage stress is to lead the patient to envision images that are calming and health-enhancing. Statements that involve the patient calming progressively with breathing, feeling increasingly relaxed, being in a calm and pleasant location, being away from stressors, and having a peaceful and calm mind are therapeutic and should be included in the script. However, items that raise stressful images or memories or that involve unrealistic expectations or elements beyond the patients control (e.g., that others will treat the patient as he desires, that everything is going the patients way, that bosses are understanding) interfere with relaxation and/or do not promote effective coping. Thus these are not health-promoting and should not be included in the script.

1. A nurse assessed a patient who reluctantly participated in activities, answered questions with minimal responses, and rarely made eye contact. What information should be included when documenting the assessment? Select all that apply. a. The patient was uncooperative b. The patients subjective responses c. Only data obtained from the patients verbal responses d. A description of the patients behavior during the interview e. Analysis of why the patient was unresponsive during the interview

ANS: B, D Both content and process of the interview should be documented. Providing only the patients verbal responses would create a skewed picture of the patient. Writing that the patient was uncooperative is subjectively worded. An objective description of patient behavior would be preferable. Analysis of the reasons for the patients behavior would be speculation, which is inappropriate.

3. What information is conveyed by nursing diagnoses? Select all that apply. a. Medical judgments about the disorder b. Unmet patient needs currently present c. Goals and outcomes for the plan of care d. Supporting data that validate the diagnoses e. Probable causes that will be targets for nursing interventions

ANS: B, D, E Nursing diagnoses focus on phenomena of concern to nurses rather than on medical diagnoses.

4. A novice nurse tells a mentor, I want to convey to my patients that I am interested in them and that I want to listen to what they have to say. Which behaviors will be helpful in meeting the nurses goal? Select all that apply. a. Sitting behind a desk, facing the patient b. Introducing self to a patient and identifying own role c. Maintaining control of discussions by asking direct questions d. Using facial expressions to convey interest and encouragement e. Assuming an open body posture and sometimes mirror imaging

ANS: B, D, E Trust is fostered when the nurse gives an introduction and identifies his or her role. Facial expressions that convey interest and encouragement support the nurses verbal statements to that effect and strengthen the message. An open body posture conveys openness to listening to what the patient has to say. Mirror imaging enhances patient comfort. A desk would place a physical barrier between the nurse and patient. A face-to-face stance should be avoided when possible and a less intense 90- or 120-degree angle used to permit either party to look away without discomfort.

2. A leader begins the discussion at the first meeting of a new group. Which comments should be included? Select all that apply. a. We use groups to provide treatment because its a more cost-effective use of staff in this time of budget constraints. b. When someone shares a personal experience, its important to keep the information confidential. c. Talking to family members about our group discussions will help us achieve our goals. d. Everyone is expected to share a personal experience at each group meeting. e. It is important for everyone to arrive on time for our group.

ANS: B, E The leader must set ground rules for the group before members can effectively participate. Confidentiality of personal experiences should be maintained. Arriving on time is important to the group process. Talking to family members would jeopardize confidentiality. While groups are cost-effective, blaming the budget would not help members feel valued. Setting an expectation to share may be intimidating for a withdrawn patient.

10. To provide comprehensive care to patients, which competency is more important for a nurse who works in a community mental health center than a psychiatric nurse who works in an inpatient unit? a. Problem-solving skills b. Calm and caring manner c. Ability to cross service systems d. Knowledge of psychopharmacology

ANS: C A community mental health nurse must be able to work with schools, corrections facilities, shelters, health care providers, and employers. The mental health nurse working in an inpatient unit needs only to be able to work within the single setting. Problem-solving skills are needed by all nurses. Nurses in both settings must have knowledge of psychopharmacology.

21. Which behavior shows that a nurse values autonomy? The nurse: a. suggests one-on-one supervision for a patient who has suicidal thoughts. b. informs a patient that the spouse will not be in during visiting hours. c. discusses options and helps the patient weigh the consequences. d. sets limits on a patients romantic overtures toward the nurse.

ANS: C A high level of valuing is acting on ones belief. Autonomy is supported when the nurse helps a patient weigh alternatives and their consequences before the patient makes a decision. Autonomy or self-determination is not the issue in any of the other behaviors.

6. According to the Recent Life Changes Questionnaire, which situation would most necessitate a complete assessment of a persons stress status and coping abilities? a. A person who has been assigned more responsibility at work b. A parent whose job required relocation to a different city c. A person returning to college after an employer ceased operations d. A man who recently separated from his wife because of marital problems

ANS: C A person returning to college after losing a job is dealing with two significant stressors simultaneously. Together, these stressors total more life change units than any of the single stressors cited in the other options.

14. A voluntarily hospitalized patient tells the nurse, Get me the forms for discharge. I want to leave now. Select the nurses best response. a. I will get the forms for you right now and bring them to your room. b. Since you signed your consent for treatment, you may leave if you desire. c. I will get them for you, but lets talk about your decision to leave treatment. d. I cannot give you those forms without your health care providers permission.

ANS: C A voluntarily admitted patient has the right to demand and obtain release in most states. However, as a patient advocate, the nurse is responsible for weighing factors related to the patients wishes and best interests. By asking for information, the nurse may be able to help the patient reconsider the decision. Facilitating discharge without consent is not in the patients best interests before exploring the reason for the request.

11. As a nurse escorts a patient being discharged after treatment for major depression, the patient gives the nurse a necklace with a heart pendant and says, Thank you for helping mend my broken heart. Which is the nurses best response? a. Accepting gifts violates the policies and procedures of the facility. b. Im glad you feel so much better now. Thank you for the beautiful necklace. c. Im glad I could help you, but I cant accept the gift. My reward is seeing you with a renewed sense of hope. d. Helping people is what nursing is all about. Its rewarding to me when patients recognize how hard we work.

ANS: C Accepting a gift creates a social rather than therapeutic relationship with the patient and blurs the boundaries of the relationship. A caring nurse will acknowledge the patients gesture of appreciation, but the gift should not be accepted.

16. An adolescent asks a nurse conducting an assessment interview, Why should I tell you anything? Youll just tell my parents whatever you find out. Which response by the nurse is appropriate? a. That isnt true. What you tell us is private and held in strict confidence. Your parents have no right to know. b. Yes, your parents may find out what you say, but it is important that they know about your problems. c. What you say about feelings is private, but some things, like suicidal thinking, must be reported to the treatment team. d. It sounds as though you are not really ready to work on your problems and make changes.

ANS: C Adolescents are very concerned with confidentiality. The patient has a right to know that most information will be held in confidence but that certain material must be reported or shared with the treatment team, such as threats of suicide, homicide, use of illegal drugs, or issues of abuse. The incorrect responses are not true, will not inspire the confidence of the patient, or are confrontational.

3. When a new bill introduced in Congress reduces funding for care of persons with mental illness, a group of nurses writes letters to their elected representatives in opposition to the legislation. Which role have the nurses fulfilled? a. Recovery b. Attending c. Advocacy d. Evidence-based practice

ANS: C An advocate defends or asserts anothers cause, particularly when the other person lacks the ability to do that for self. Examples of individual advocacy include helping patients understand their rights or make decisions. On a community scale, advocacy includes political activity, public speaking, and publication in the interest of improving the human condition. Since funding is necessary to deliver quality programming for persons with mental illness, the letter- writing campaign advocates for that cause on behalf of patients who are unable to articulate their own needs.

6. A patient discloses several concerns and associated feelings. If the nurse wants to seek clarification, which comment would be appropriate? a. What are the common elements here? b. Tell me again about your experiences. c. Am I correct in understanding that . . . d. Tell me everything from the beginning.

ANS: C Asking, Am I correct in understanding that permits clarification to ensure that both the nurse and patient share mutual understanding of the communication. Asking about common elements encourages comparison rather than clarification. The remaining responses are implied questions that suggest the nurse was not listening.

7. A suspicious, socially isolated patient lives alone, eats one meal a day at a local shelter, and spends the remaining daily food allowance on cigarettes. Select a community psychiatric nurses best initial action. a. Explore ways to help the patient stop smoking. b. Report the situation to the manager of the shelter. c. Assess the patients weight; determine foods and amounts eaten. d. Arrange hospitalization for the patient in order to formulate a new treatment plan.

ANS: C Assessment of biopsychosocial needs and general ability to live in the community is called for before any other action is taken. Both nutritional status and income adequacy are critical assessment parameters. A patient may be able to maintain adequate nutrition while eating only one meal a day. The rule is to assess before taking action. Hospitalization may not be necessary. Smoking cessation strategies can be pursued later.

6. A nurse must assess several new patients at a community mental health center. Conclusions concerning current functioning should be made on the basis of: a. the degree of conformity of the individual to society's norms. b. the degree to which an individual is logical and rational. c. a continuum from mentally healthy to unhealthy. d. the rate of intellectual and emotional growth.

ANS: C Because mental health and mental illness are relative concepts, assessment of functioning is made by using a continuum. Mental health is not based on conformity; some mentally healthy individuals do not conform to society's norms. Most individuals occasionally display illogical or irrational thinking. The rate of intellectual and emotional growth is not the most useful criterion to assess mental health or mental illness.

7. A patient tells the nurse, I dont think Ill ever get out of here. Select the nurses most therapeutic response. a. Dont talk that way. Of course you will leave here! b. Keep up the good work, and you certainly will. c. You dont think youre making progress? d. Everyone feels that way sometimes.

ANS: C By asking if the patient does not believe that progress has been made, the nurse is reflecting by putting into words what the patient is hinting. By making communication more explicit, issues are easier to identify and resolve. The remaining options are non-therapeutic techniques. Telling the patient not to talk that way is disapproving. Saying that everyone feels that way at times minimizes feelings. Telling the patient that good work will always result in success is falsely reassuring.

4. Which comment best indicates that a patient perceived the nurse was caring? My nurse: a. always asks me which type of juice I want to help me swallow my medication. b. explained my treatment plan to me and asked for my ideas about how to make it better. c. spends time listening to me talk about my problems. That helps me feel like I am not alone. d. told me that if I take all the medicines the doctor prescribes, then I will get discharged sooner.

ANS: C Caring evidences empathetic understanding as well as competency. It helps change pain and suffering into a shared experience, creating a human connection that alleviates feelings of isolation. The distracters give examples of statements that demonstrate advocacy or giving advice.

12. A patient tells the nurse, My doctor thinks my problems with stress relate to the negative way I think about things and suggested I learn new ways of thinking. Which response by the nurse would support the recommendation? a. Encourage the patient to imagine being in calm circumstances. b. Provide the patient with a blank journal and guidance about journaling. c. Teach the patient to recognize, reconsider, and reframe irrational thoughts. d. Teach the patient to use instruments that give feedback about bodily functions.

ANS: C Cognitive reframing focuses on recognizing and correcting maladaptive patterns of thinking that create stress or interfere with coping. Cognitive reframing involves recognizing the habit of thinking about a situation or issue in a fixed, irrational, and unquestioning manner. Helping the patient to recognize and reframe (reword) such thoughts so that they are realistic and accurate promotes coping and reduces stress. Thinking about being in calming circumstances is a form of guided imagery. Instruments that give feedback about bodily functions are used in biofeedback. Journaling is effective for helping to increase self-awareness. However, none of these last three interventions is likely to alter the patients manner of thinking.

23. A cognitive strategy a nurse could use to assist a very dependent patient would be to help the patient: a. reveal dream content. b. take prescribed medications. c. examine thoughts about being autonomous. d. role model ways to ask for help from others.

ANS: C Cognitive theory suggests that one's thought processes are the basis of emotions and behavior. Changing faulty learning makes the development of new adaptive behaviors possible. Revealing dream content would be used in psychoanalytical therapy. Taking prescribed medications is an intervention associated with biological therapy. A dependent patient needs to develop independence.

12. Which disorder is a culture-bound syndrome? a. Epilepsy b. Schizophrenia c. Running amok d. Major depression

ANS: C Culture-bound syndromes occur in specific sociocultural contexts and are easily recognized by people in those cultures. A syndrome recognized in parts of Southeast Asia is running amok, in which a person (usually a male) runs around engaging in furious, almost indiscriminate violent behavior.

7. What is the desirable outcome for the orientation stage of a nurse-patient relationship? The patient will demonstrate behaviors that indicate: a. self-responsibility and autonomy. b. a greater sense of independence. c. rapport and trust with the nurse. d. resolved transference.

ANS: C Development of rapport and trust is necessary before the relationship can progress to the working phase. Behaviors indicating a greater sense of independence, self-responsibility, and resolved transference occur in the working phase.

8. During which phase of the nurse-patient relationship can the nurse anticipate that identified patient issues will be explored and resolved? a. Preorientation b. Orientation c. Working d. Termination

ANS: C During the working phase, the nurse strives to assist the patient in making connections among dysfunctional behaviors, thinking, and emotions and offers support while alternative coping behaviors are tried.

25. After leaving work, a nurse realizes documentation of administration of a PRN medication was omitted. This off-duty nurse phones the nurse on duty and says, Please document administration of the medication for me. My password is alpha1. The nurse receiving the call should: a. fulfill the request promptly. b. document the callers password. c. refer the matter to the charge nurse to resolve. d. report the request to the patients health care provider.

ANS: C Fraudulent documentation may be grounds for discipline by the state board of nursing. Referring the matter to the charge nurse will allow observance of hospital policy while ensuring that documentation occurs. Notifying the health care provider would be unnecessary when the charge nurse can resolve the problem. Nurses should not provide passwords to others.

20. A category 5 tornado occurred in a community of 400 people resulting in destruction of many homes and businesses. In the 2 years after this disaster, 140 individuals were diagnosed with posttraumatic stress disorder (PTSD). Which term best applies to these newly diagnosed cases? a. Prevalence b. Co-morbidity c. Incidence d. Parity

ANS: C Incidence refers to the number of new cases of mental disorders in a healthy population within a given period of time. Prevalence describes the total number of cases, new and existing, in a given population during a specific period of time, regardless of when they became ill. Parity refers to equivalence, and legislation required insurers that provide mental health coverage to offer annual and lifetime benefits at the same level provided for medical/surgical coverage. Co-morbidity refers to having more than one mental disorder at a time.

6. A patient begins a new program to assist with building social skills. In which part of the plan of care should a nurse record the item, Encourage patient to attend one psychoeducational group daily? a. Assessment b. Analysis c. Implementation d. Evaluation

ANS: C Interventions are the nursing prescriptions to achieve the outcomes. Interventions should be specific.

26. Which individual diagnosed with a mental illness may need involuntary hospitalization? An individual: a. who has a panic attack after her child gets lost in a shopping mall b. with visions of demons emerging from cemetery plots throughout the community c. who takes 38 acetaminophen tablets after the persons stock portfolio becomes worthless d. diagnosed with major depression who stops taking prescribed antidepressant medication

ANS: C Involuntary hospitalization protects patients who are dangerous to themselves or others and cannot care for their own basic needs. Involuntary hospitalization also protects other individuals in society. An overdose of acetaminophen indicates dangerousness to self. The behaviors described in the other options are not sufficient to require involuntary hospitalization.

19. The spouse of a patient diagnosed with schizophrenia says, I dont understand how events from childhood have anything to do with this disabling illness. Which response by the nurse will best help the spouse understand the cause of this disorder? a. Psychological stress is the basis of most mental disorders. b. This illness results from developmental factors rather than stress. c. Research shows that this condition more likely has a biological basis. d. It must be frustrating for you that your spouse is sick so much of the time.

ANS: C Many of the most prevalent and disabling mental disorders have strong biological influences. Genetics are only one part of biological factors. Empathy does not address increasing the spouses level of knowledge about the cause of the disorder. The other distracters are not established facts.

18. A group is in the working phase. One member states, That is the stupidest thing Ive ever heard. Everyone whines and tells everyone else what to do. This group is a waste of my time. Which initial action by the group leader would be most therapeutic? a. Advise the member that hostility is inappropriate. Remove the member if it continues. b. Keep the groups focus on this member so the person can express the anger. c. Meet privately with the member outside of group to discuss the anger. d. Change to a more positive topic of discussion in this group session.

ANS: C Meeting privately with the member can convey interest and help defuse the anger so that it is less disruptive to the group. Removing the member would be a last resort and used only when the behavior is intolerably disruptive to the group process and all other interventions have failed. Decreasing the focus on the hostile member and focusing more on positive members can help soften the anger. Angry members often hide considerable vulnerability by using anger to keep others at a distance and intimidated. Changing the subject fails to respond to the behavior.

20. When a new patient is hospitalized, a nurse takes the patient on a tour, explains rules of the unit, and discusses the daily schedule. The nurse is engaged in: a. counseling. b. health teaching. c. milieu management. d. psychobiological intervention.

ANS: C Milieu management provides a therapeutic environment in which the patient can feel comfortable and safe while engaging in activities that meet the patients physical and mental health needs. Counseling refers to activities designed to promote problem solving and enhanced coping and includes interviewing, crisis intervention, stress management, and conflict resolution. Health teaching involves identifying health education needs and giving information about these needs. Psychobiological interventions involve medication administration and monitoring response to medications.

23. A patient says, Im still on restriction, but I want to attend some off-unit activities. Would you ask the doctor to change my privileges? What is the nurses best response? a. Why are you asking me when youre able to speak for yourself? b. I will be glad to address it when I see your doctor later today. c. Thats a good topic for you to discuss with your doctor. d. Do you think you cant speak to a doctor?

ANS: C Nurses should encourage patients to work at their optimal level of functioning. A nurse does not act for the patient unless it is necessary. Acting for a patient increases feelings of helplessness and dependency.

22. Select the most appropriate label to complete this nursing diagnosis: ___________ related to feelings of shyness and poorly developed social skills as evidenced by watching television alone at home every evening. a. Deficient knowledge b. Ineffective coping c. Social isolation d. Powerlessness

ANS: C Nursing diagnoses are selected based on the etiological factors and assessment findings, or evidence. In this instance, the evidence shows social isolation that is caused by shyness and poorly developed social skills.

16. Operant conditioning is part of the treatment plan to encourage speech in a child who is nearly mute. Which technique applies? a. Encourage the child to observe others talking. b. Include the child in small group activities. c. Give the child a small treat for speaking. d. Teach the child relaxation techniques.

ANS: C Operant conditioning involves giving positive reinforcement for a desired behavior. Treats are rewards and reinforce speech through positive reinforcement.

14. A nurse consistently encourages patient to do his or her own activities of daily living (ADLs). If the patient is unable to complete an activity, the nurse helps until the patient is once again independent. This nurses practice is most influenced by which theorist? a. Betty Neuman b. Patricia Benner c. Dorothea Orem d. Joyce Travelbee

ANS: C Orem emphasizes the role of the nurse in promoting self-care activities of the patient; this has relevance to the seriously and persistently mentally ill patient.

4. A patient diagnosed with schizophrenia had an exacerbation related to medication noncompliance and was hospitalized for 5 days. The patients thoughts are now more organized, and discharge is planned. The patients family says, Its too soon for discharge. We will just go through all this again. The nurse should: a. ask the case manager to arrange a transfer to a long-term care facility. b. notify hospital security to handle the disturbance and escort the family off the unit. c. explain that the patient will continue to improve if the medication is taken regularly. d. contact the health care provider to meet with the family and explain the discharge rationale.

ANS: C Patients do not stay in a hospital until every symptom disappears. The nurse must assume responsibility to advocate for the patients right to the least restrictive setting as soon as the symptoms are under control and for the right of citizens to control health care costs. The health care provider will use the same rationale. Shifting blame will not change the discharge. Security is unnecessary. The nurse can handle this matter.

15. A soldier who served in a combat zone returned to the United States. The soldier's spouse complains to the nurse, "We had planned to start a family, but now he won't talk about it. He won't even look at children." The spouse is describing which symptom associated with post-traumatic stress disorder (PTSD)? a. Reexperiencing b. Hyperarousal c. Avoidance d. Psychosis

ANS: C Physiologic reactions to reminders of the event include a persistent avoidance of the stimuli associated with the trauma; the individual avoids talking about the event or avoids activities, people, or places that arouse memories of the trauma. Avoidance is exemplified by a sense of foreshortened future and estrangement. No evidence suggests that this soldier is having a hyperarousal reaction or is re-experiencing war-related traumas. Psychosis is not evident.

21. A psychotherapist works with an anxious, dependent patient. Which strategy is most consistent with psychoanalytic psychotherapy? a. Identifying the patients strengths and assets b. Praising the patient for describing feelings of isolation c. Focusing on feelings developed by the patient toward the therapist d. Providing psychoeducation and emphasizing medication adherence

ANS: C Positive or negative feelings of the patient toward the therapist indicate transference. Transference is a psychoanalytic concept that can be used to explore previously unresolved conflicts. The distracters relate to biological therapy and supportive psychotherapy. Use of psychoeducational materials is a common homework assignment used in cognitive therapy.

6. A patient in a group therapy session listens to others and then remarks, I used to think I was the only one who felt afraid. I guess Im not as alone as I thought. This comment is an example of: a. altruism. b. ventilation. c. universality. d. group cohesiveness.

ANS: C Realizing that one is not alone and that others share the same problems and feelings is called universality. Ventilation refers to expressing emotions. Altruism refers to benefitting by being of help to others. Group cohesiveness refers to the degree of bonding among members of the group.

21. A patient usually watches television all day, seldom going out in the community or socializing with others. The patient says, I dont know what to do with my free time. Which member of the treatment team would be most helpful to this patient? a. Psychologist b. Social worker c. Recreational therapist d. Occupational therapist

ANS: C Recreational therapists help patients use leisure time to benefit their mental health. Occupational therapists assist with a broad range of skills, including those for employment. Psychologists conduct testing and provide other patient services. Social workers focus on the patients support system.

19. Which issues should a nurse address during the first interview with a patient with a psychiatric disorder? a. Trust, congruence, attitudes, and boundaries b. Goals, resistance, unconscious motivations, and diversion c. Relationship parameters, the contract, confidentiality, and termination d. Transference, countertransference, intimacy, and developing resources

ANS: C Relationship parameters, the contract, confidentiality, and termination are issues that should be considered during the orientation phase of the relationship. The remaining options are issues that are dealt with later.

2. A newly admitted patient diagnosed with major depression has gained 20 pounds over a few months and has suicidal ideation. The patient has taken an antidepressant medication for 1 week without remission of symptoms. Select the priority nursing diagnosis. a. Imbalanced nutrition: more than body requirements b. Chronic low self-esteem c. Risk for suicide d. Hopelessness

ANS: C Risk for suicide is the priority diagnosis when the patient has both suicidal ideation and a plan to carry out the suicidal intent. Imbalanced nutrition, hopelessness, and chronic low self-esteem may be applicable nursing diagnoses, but these problems do not affect patient safety as urgently as would a suicide attempt.

17. Which principle should guide the nurse in determining the extent of silence to use during patient interview sessions? a. A nurse is responsible for breaking silences. b. Patients withdraw if silences are prolonged. c. Silence can provide meaningful moments for reflection. d. Silence helps patients know that what they said was understood.

ANS: C Silence can be helpful to both participants by giving each an opportunity to contemplate what has transpired, weigh alternatives, and formulate ideas. A nurse breaking silences is not a principle related to silences. It is inaccurate to say that patients withdraw during long silences or that silence helps patients know that they are understood. Feedback helps patients know they have been understood.

3. After several therapeutic encounters with a patient who recently attempted suicide, which occurrence should cause the nurse to consider the possibility of countertransference? a. The patients reactions toward the nurse seem realistic and appropriate. b. The patient states, Talking to you feels like talking to my parents. c. The nurse feels unusually happy when the patients mood begins to lift. d. The nurse develops a trusting relationship with the patient.

ANS: C Strong positive or negative reactions toward a patient or over-identification with the patient indicate possible countertransference. Nurses must carefully monitor their own feelings and reactions to detect countertransference and then seek supervision. Realistic and appropriate reactions from a patient toward a nurse are desirable. One incorrect response suggests transference. A trusting relationship with the patient is desirable.

7. A patient newly diagnosed as HIV-positive seeks the nurses advice on how to reduce the risk of infections. The patient says, I used to go to church and I was in my best health then. Maybe I should start going to church again. Which response will the nurse offer? a. Religion does not usually affect health, but you were younger and stronger then. b. Contact with supportive people at church might help, but religion itself is not especially helpful. c. Studies show that spiritual practices can enhance immune system function and coping abilities. d. Going to church would expose you to many potential infections. Lets think about some other options.

ANS: C Studies have shown a positive correlation between spiritual practices and enhanced immune system function and sense of well-being. The other options wrongly suggest that spiritual practices have little effect on the immune system or reject the patients preferences regarding health management.

18. A patient hurriedly tells the community mental health nurse, "Everything's a disaster! I can't concentrate. My disability check didn't come. My roommate moved out, and I can't afford the rent. My therapist is moving away. I feel like I'm coming apart." Which nursing diagnosis applies? a. Decisional conflict, related to challenges to personal values b. Spiritual distress, related to ethical implications of treatment regimen c. Anxiety, related to changes perceived as threatening to psychological equilibrium d. Impaired environmental interpretation syndrome, related to solving multiple problems affecting security needs

ANS: C Subjective and objective data obtained by the nurse suggest the patient is experiencing anxiety caused by multiple threats to security needs. Data are not present to suggest Decisional conflict, Spiritual distress caused by ethical conflicts, or Impaired environmental interpretation syndrome.

6. Termination of a therapeutic nurse-patient relationship has been successful when the nurse: a. avoids upsetting the patient by shifting focus to other patients before the discharge. b. gives the patient a personal telephone number and permission to call after discharge. c. discusses with the patient changes that happened during the relationship and evaluates outcomes. d. offers to meet the patient for coffee and conversation three times a week after discharge.

ANS: C Summarizing and evaluating progress help validate the experience for the patient and the nurse and facilitate closure. Termination must be discussed; avoiding discussion by spending little time with the patient promotes feelings of abandonment. Successful termination requires that the relationship be brought to closure without the possibility of dependency-producing ongoing contact.

25. A college student received an invitation to attend the wedding of a close friend who lives across the country. The student is afraid of flying. Which type of therapy would be most helpful for this patient? a. Psychoanalysis b. Milieu therapy c. Systematic desensitization d. Short-term dynamic therapy

ANS: C Systematic desensitization is a type of therapy aimed at extinguishing a specific behavior, such as the fear of flying. Psychoanalysis and short-term dynamic therapy seek to uncover conflicts. Milieu therapy involves environmental factors.

19. A person with a fear of heights drives across a high bridge. Which division of the autonomic nervous system will be stimulated in response to this experience? a. Limbic system b. Peripheral nervous system c. Sympathetic nervous system d. Parasympathetic nervous system

ANS: C The autonomic nervous system is comprised of the sympathetic (fight or flight response) and parasympathetic nervous system (relaxation response). In times of stress, the sympathetic nervous system is stimulated. A person fearful of heights would experience stress associated with the experience of driving across a high bridge. The peripheral nervous system responds to messages from the sympathetic nervous system. The limbic system processes emotional responses but is not specifically part of the autonomic nervous system.

20. The case manager plans to discuss the treatment plan with a patients family. Select the case managers first action. a. Determine an appropriate location for the conference. b. Support the discussion with examples of the patients behavior. c. Obtain the patients permission for the exchange of information. d. Determine which family members should participate in the conference.

ANS: C The case manager must respect the patients right to privacy, which extends to discussions with family. Talking to family members is part of the case managers role. Actions identified in the distracters occur after the patient has given permission.

2. A patient was hospitalized for 24 hours after a reaction to a psychotropic medication. While planning discharge, the case manager learned that the patient received a notice of eviction immediately prior to admission. Select the case managers most appropriate action. a. Postpone the patients discharge from the hospital. b. Contact the landlord who evicted the patient to further discuss the situation. c. Arrange a temporary place for the patient to stay until new housing can be arranged. d. Determine whether the adverse medication reaction was genuine because the patient had nowhere to live.

ANS: C The case manager should intervene by arranging temporary shelter for the patient until an apartment can be found. This activity is part of the coordination and delivery of services that falls under the case manager role. None of the other options is a viable alternative.

15. The unit secretary receives a phone call from the health insurer for a hospitalized patient. The caller seeks information about the patients projected length of stay. How should the nurse instruct the unit secretary to handle the request? a. Obtain the information from the patients medical record and relay it to the caller. b. Inform the caller that all information about patients is confidential. c. Refer the request for information to the patients case manager. d. Refer the request to the health care provider.

ANS: C The case manager usually confers with insurers and provides the treatment team with information about available resources. The unit secretary should be mindful of patient confidentiality and should neither confirm that the patient is an inpatient nor disclose other information.

19. A new antidepressant is prescribed for an elderly patient with major depression, but the dose is more than the usual geriatric dose. The nurse should: a. consult a reliable drug reference. b. teach the patient about possible side effects and adverse effects. c. withhold the medication and confer with the health care provider. d. encourage the patient to increase oral fluids to reduce drug concentration.

ANS: C The dose of antidepressants for elderly patients is often less than the usual adult dose. The nurse should withhold the medication and consult the health care provider who wrote the order. The nurses duty is to practice according to professional standards as well as intervene and protect the patient.

18. A person with a fear of heights drives across a high bridge. Which structure will stimulate a response from the autonomic nervous system? a. Thalamus b. Parietal lobe c. Hypothalamus d. Pituitary gland

ANS: C The individual will find this experience stressful. The hypothalamus functions as the command-and-control center when receiving stressful signals. The hypothalamus responds to signals of stress by engaging the autonomic nervous system. The parietal lobe is responsible for interpretation of other sensations. The thalamus processes messages associated with pain and wakefulness. The pituitary gland may be involved in other aspects of the persons response but would not stimulate the autonomic nervous system.

17. A group is in the working phase. One member says, That is the stupidest thing Ive ever heard. Everyone whines and tells everyone else what to do. This group is a total waste of my time. Which comment by the group leader would be most therapeutic? a. You seem to think you know a lot already. Since you know so much, perhaps you can tell everyone why you are back in the hospital? b. I think you have made your views clear, but I wonder if others feel the same way. How does everyone else feel about our group? c. It must be hard to be so angry. Direct this comment to another group member, You were also angry at first but not now. What has helped you? d. I would like to remind you that one of our group rules is that everyone is to offer only positive responses to the comments of others.

ANS: C The members comments demean the group and its members and suggest that the member is very angry. Labeling the emotion and conveying empathy would be therapeutic. Focusing on members who are likely to be more positive can balance the influence of demoralizing members. You seem to know a lot conveys hostility from the leader, who confronts and challenges the member to explain how he came to be readmitted if he was so knowledgeable, implying that he is less knowledgeable than he claims. This comment suggests countertransference and is non-therapeutic. Shifting away from the complaining member to see if others agree seeks to have others express disagreement with this member, but that might not happen. In the face of his anger, they might be quiet or afraid to oppose him, or they could respond in kind by expressing hostility themselves. A rule that only positive exchanges are permitted would suppress conflict, reducing the effectiveness of the therapy group.

16. A patient reports, I am overwhelmed by stress. Which question by the nurse would be most important to use in the initial assessment of this the patient? a. Tell me about your family history. Do you have any relatives who have problems with stress? b. Tell me about your exercise. How much activity do you typically get in a day? c. Tell me about the kinds of things you do to reduce or cope with your stress. d. Stress can interfere with sleep. How much did you sleep last night?

ANS: C The most important data to collect during an initial assessment is that which reflects how stress is affecting the patient and how he is coping with stress at present. This data would indicate whether or not his distress is placing him in danger (e.g., by elevating his blood pressure dangerously or via maladaptive responses, such as drinking) and would help the nurse understand how he copes and how well his coping strategies and resources serve him. Of the choices presented, the highest priority would be to determine what he is doing to cope at present, preferably via an open-ended inquiry. Family history, the extent of his use of exercise, and how much sleep he is getting are all helpful but seek data that is less of a priority. Also, the manner in which such data is sought here is likely to provide only brief responses (e.g., how much sleep he got on one particular night is probably less important than how much he is sleeping in general).

7. A nurse at the well child clinic realizes that many parents have misconceptions about effective ways of disciplining their children. The nurse decides to form a group to address this problem. What should be the focus of the group? a. Support b. Socialization c. Health education d. Symptom management

ANS: C The nurse has diagnosed a knowledge deficit. The focus of the group should be education. Support and socialization are beneficial but should not be the primary focus of the group, and symptoms are not identified for intervention here.

8. A nurse works with a patient to establish outcomes. The nurse believes that one outcome suggested by the patient is not in the patient's best interest. What is the nurse's best action? a. Remain silent. b. Educate the patient that the outcome is not realistic. c. Explore with the patient possible consequences of the outcome. d. Formulate a more appropriate outcome without the patient's input.

ANS: C The nurse should not impose outcomes on the patient; however, the nurse has a responsibility to help the patient evaluate what is in his or her best interest. Exploring possible consequences is an acceptable approach.

2. A patient diagnosed with emphysema has severe shortness of breath and needs portable oxygen when leaving home. Recently the patient has reduced activity because of fear that breathing difficulty will occur. A nurse suggests using guided imagery. Which image should the patient be encouraged to visualize? a. Engaging in activity without using any supplemental oxygen b. Sleeping comfortably and soundly, without respiratory distress c. Feeling relaxed and taking regular deep breaths when leaving home d. Having a younger, healthier body that knows no exercise limitations

ANS: C The patient has dysfunctional images of dyspnea. Guided imagery can help replace the dysfunctional image with a positive coping image. Athletes have found that picturing successful images can enhance performance. Encouraging the patient to imagine a regular breathing depth and rate will help improve oxygencarbon dioxide exchange and help achieve further relaxation. Other options focus on unrealistic goals (being younger, not needing supplemental oxygen) or restrict her quality of life.

7. A student nurse prepares to administer oral medications to a patient diagnosed with major depressive disorder, but the patient refuses the medication. The student nurse should: a. tell the patient, "I'll get an unsatisfactory grade if I don't give you the medication." b. tell the patient, "Refusing your medication is not permitted. You are required to take it." c. discuss the patient's concerns about the medication, and report to the staff nurse. d. document the patient's refusal of the medication without further comment.

ANS: C The patient has the right to refuse medication in most cases. The patient's reason for refusing should be ascertained, and the refusal should be reported to a unit nurse. Sometimes refusals are based on unpleasant side effects that can be ameliorated. Threats and manipulation are inappropriate. Medication refusal should be reported to permit appropriate intervention.

4. The goal for a patient is to increase resiliency. Which outcome should a nurse add to the plan of care? Within 3 days, the patient will: a. describe feelings associated with loss and stress. b. meet own needs without considering the rights of others. c. identify healthy coping behaviors in response to stressful events. d. allow others to assume responsibility for major areas of own life.

ANS: C The patient's ability to identify healthy coping behaviors indicates adaptive, healthy behavior and demonstrates an increased ability to recover from severe stress. Describing feelings associated with loss and stress does not move the patient toward adaptation. The remaining options are maladaptive behaviors.

5. The parent of a 4-year-old rewards and praises the child for helping a younger sibling, being polite, and using good manners. The nurse supports this use of praise related to these behaviors. These qualities are likely to be internalized and become part of which system of the personality? a. Id b. Ego c. Superego d. Preconscious

ANS: C The superego contains the thou shalts, or moral standards internalized from interactions with significant others. Praise fosters internalization of desirable behaviors. The id is the center of basic instinctual drives, and the ego is the mediator. The ego is the problem-solving and reality-testing portion of the personality that negotiates solutions with the outside world. The preconscious is a level of awareness from which material can be retrieved easily with conscious effort.

7. An adult says, I never know the answers, and My opinion doesnt count. Which psychosocial crisis was unsuccessfully resolved for this adult? a. Initiative versus guilt b. Trust versus mistrust c. Autonomy versus shame and doubt d. Generativity versus self-absorption

ANS: C These statements show severe self-doubt, indicating that the crisis of gaining control over the environment was not met successfully. Unsuccessful resolution of the crisis of initiative versus guilt results in feelings of guilt. Unsuccessful resolution of the crisis of trust versus mistrust results in poor interpersonal relationships and suspicion of others. Unsuccessful resolution of the crisis of generativity versus self-absorption results in self-absorption that limits the ability to grow as a person.

8. Documentation in a patients chart shows, Throughout a 5-minute interaction, patient fidgeted and tapped left foot, periodically covered face with hands, and looked under chair while stating, I enjoy spending time with you. Which analysis is most accurate? a. The patient is giving positive feedback about the nurses communication techniques. b. The nurse is viewing the patients behavior through a cultural filter. c. The patients verbal and nonverbal messages are incongruent. d. The patient is demonstrating psychotic behaviors.

ANS: C When a verbal message is not reinforced with nonverbal behavior, the message is confusing and incongruent. Some clinicians call it a mixed message. It is inaccurate to say that the patient is giving positive feedback about the nurses communication techniques. The concept of a cultural filter is not relevant to the situation because a cultural filter determines what we will pay attention to and what we will ignore. Data are insufficient to draw the conclusion that the patient is demonstrating psychotic behaviors.

1. In which situations would a nurse have the duty to intervene and report? Select all that apply. a. A peer has difficulty writing measurable outcomes. b. A health care provider gives a telephone order for medication. c. A peer tries to provide patient care in an alcohol-impaired state. d. A team member violates relationship boundaries with a patient. e. A patient refuses medication prescribed by a licensed health care provider.

ANS: C, D Both keyed answers are events that jeopardize patient safety. The distracters describe situations that may be resolved with education or that are acceptable practices.

3. Which comments by a nurse demonstrate use of therapeutic communication techniques? Select all that apply. a. Why do you think these events have happened to you? b. There are people with problems much worse than yours. c. Im glad you were able to tell me how you felt about your loss. d. I noticed your hands trembling when you told me about your accident. e. You look very nice today. Im proud you took more time with your appearance.

ANS: C, D The correct responses demonstrate use of the therapeutic techniques making an observation and showing empathy. The incorrect responses demonstrate minimizing feelings, probing, and giving approval, which are non-therapeutic techniques.

2. Which experiences are most likely to precipitate post-traumatic stress disorder (PTSD)? Select all that apply. a. An 8-year-old child watches an R-rated movie with both parents. b. A young adult jumps from a bridge with a bungee cord with a best friend. c. An adolescent is kidnapped and held for 2 years in the home of a sexual predator. d. A passenger is in a bus that overturns on a sharp curve in the road, tumbling down an embankment. e. An adult is trapped for 3 hours at an angle in an elevator after a portion of the supporting cable breaks.

ANS: C, D, E PTSD usually follows a traumatic event that is outside the range of usual experience. Examples are childhood physical abuse, torture or kidnapping, military combat, sexual assault, and natural disasters such as floods, tornados, earthquakes, and tsunamis; human disasters such as a bus or elevator accident or crime-related events such being taken hostage are additional examples. The common element in these experiences is the individual's extraordinary helplessness or powerlessness in the face of such stressors. Bungee jumping by adolescents is part of the developmental task and might be frightening but in an exhilarating way rather than a harmful way. A child may be disturbed by an R-rated movie, but the presence of the parents would modify the experience in a positive way.

5. Which statements most clearly reflect the stigma of mental illness? Select all that apply. a. Many mental illnesses are hereditary. b. Mental illness can be evidence of a brain disorder. c. People claim mental illness so they can get disability checks. d. Mental illness results from the breakdown of American families. e. If people with mental illness went to church, their symptoms would vanish.

ANS: C, D, E Stigma is represented by judgmental remarks that discount the reality and validity of mental illness. Many mental illnesses are genetically transmitted. Neuroimaging can show changes associated with some mental illnesses.

1. A nurse volunteers for a committee that must revise the hospital policies and procedures for suicide precautions. Which resources would provide the best guidance? Select all that apply. a. Diagnostic and Statistical Manual of Mental Disorders (fifth edition) (DSM-5) b. State's nurse practice act c. State and federal regulations that govern hospitals d. Summary of common practices of several local hospitals e. American Nurses Association Scope and Standards of Practice for Psychiatric-Mental Health Nursing

ANS: C, E Regulations regarding hospitals provide information about the minimal standard. The American Nurses Association (ANA) national standards focus on elevating practice by setting high standards for nursing practice. The DSM-5 and the state's nurse practice act would not provide relevant information. A summary of common practices of several local hospitals cannot be guaranteed to be helpful because the customs may or may not comply with laws or best practices.

3. A nurse assesses the health status of soldiers returning from Afghanistan. Screening will be a priority for signs and symptoms of which health problems? Select all that apply. a. Schizophrenia b. Eating disorder c. Traumatic brain injury d. Seasonal affective disorder e. Post-traumatic stress disorder

ANS: C, E TBI and PTSD each occur in approximately 20% of soldiers returning from Afghanistan. Some soldiers have both problems. The incidence of disorders identified in the distractors would be expected to parallel the general population.

2. Which actions violate the civil rights of a psychiatric patient? The nurse: (select all that apply) a. performs mouth checks after overhearing a patient say, Ive been spitting out my medication. b. begins suicide precautions before a patient is assessed by the health care provider. c. opens and reads a letter a patient left at the nurses station to be mailed. d. places a patients expensive watch in the hospital business office safe. e. restrains a patient who uses profanity when speaking to the nurse.

ANS: C, E The patient has the right to send and receive mail without interference. Restraint is not indicated because a patient uses profanity; there are other less restrictive ways to deal with this behavior. The other options are examples of good nursing judgment and do not violate the patients civil rights.

17. Which belief will best support a nurses efforts to provide patient advocacy during a multidisciplinary patient care planning session? a. All mental illnesses are culturally determined. b. Schizophrenia and bipolar disorder are cross-cultural disorders. c. Symptoms of mental disorders are unchanged from culture to culture. d. Assessment findings in mental disorders reflect a persons cultural patterns.

ANS: D A nurse who understands that a patients symptoms are influenced by culture will be able to advocate for the patient to a greater degree than a nurse who believes that culture is of little relevance. The distracters are untrue statements.

5. During a group therapy session, a newly admitted patient suddenly says to the nurse, How old are you? You seem too young to be leading a group. Select the nurses most appropriate response. a. I am wondering what leads you to ask. Please tell me more. b. I am old enough to be a nurse, which qualifies me to lead this group. c. My age is not pertinent to why we are here and should not concern you. d. You are wondering whether I have enough experience to lead this group?

ANS: D A question such as this is common in the initial phase of group development when members are getting to know one another, dealing with trust issues, and testing the leader. Making explicit the implied serves to role model more effective communication and prompts further discussion of the patients concern. Asking the patient to tell the leader more about the question focuses on the reason for the members concern rather than on the issue raised (the experience and ability of the leader) and is a less helpful response. I am old enough to be a nurse and age is not pertinent are defensive responses and fail to address the patients valid concern.

6. The patients below were evaluated in the emergency department. The psychiatric unit has one bed available. Which patient should be admitted? The patient: a. feeling anxiety and a sad mood after separation from a spouse of 10 years. b. who self-inflicted a superficial cut on the forearm after a family argument. c. experiencing dry mouth and tremor related to taking haloperidol (Haldol). d. who is a new parent and hears voices saying, Smother your baby.

ANS: D Admission to the hospital would be justified by the risk of patient danger to self or others. The other patients have issues that can be handled with less restrictive alternatives than hospitalization.

11. A newly admitted acutely psychotic patient is a private patient of the medical director and a private-pay patient. To whom does the psychiatric nurse assigned to the patient owe the duty of care? a. Medical director b. Hospital c. Profession d. Patient

ANS: D Although the nurse is accountable to the health care provider, the agency, the patient, and the profession, the duty of care is owed to the patient.

4. The desired outcome for a patient experiencing insomnia is, Patient will sleep for a minimum of 5 hours nightly within 7 days. At the end of 7 days, review of sleep data shows the patient sleeps an average of 4 hours nightly and takes a 2-hour afternoon nap. The nurse will document the outcome as: a. consistently demonstrated. b. often demonstrated. c. sometimes demonstrated. d. never demonstrated.

ANS: D Although the patient is sleeping 6 hours daily, the total is not one uninterrupted session at night. Therefore, the outcome must be evaluated as never demonstrated.

8. Which outcome would be most appropriate for a symptom-management group for persons with schizophrenia? Group members will: a. state the names of their medications. b. resolve conflicts within their families. c. rate anxiety at least two points lower. d. describe ways to cope with their illness.

ANS: D An appropriate psychoeducational focus for patients with schizophrenia is managing their symptoms; coping with symptoms such as impaired memory or impaired reality testing can improve functioning and enhance their quality of life. Names of medications might be appropriate for a medication education group but would be a low priority for symptom management. Addressing intra-family issues would be more appropriate within a family therapy group or possibly a support group. Rating anxiety lower would be an expected outcome for a stress-management group.

18. The nurse assigned to assertive community treatment (ACT) should explain the programs treatment goal as: a. assisting patients to maintain abstinence from alcohol and other substances of abuse. b. providing structure and a therapeutic milieu for mentally ill patients whose symptoms require stabilization. c. maintaining medications and stable psychiatric status for incarcerated inmates who have a history of mental illness. d. providing services for mentally ill individuals who require intensive treatment to continue to live in the community.

ANS: D An assertive community treatment (ACT) program provides intensive community services to persons with serious, persistent mental illness who live in the community but require aggressive services to prevent repeated hospitalizations.

14. Which patient would be most appropriate to refer for assertive community treatment (ACT)? A patient diagnosed with: a. a phobic fear of crowded places. b. a single episode of major depression. c. a catastrophic reaction to a tornado in the community. d. schizophrenia and four hospitalizations in the past year.

ANS: D Assertive community treatment (ACT) provides intensive case management for persons with serious persistent mental illness who live in the community. Repeated hospitalization is a frequent reason for this intervention. The distracters identify mental health problems of a more episodic nature.

12. Which aspect of direct care is an experienced, inpatient psychiatric nurse most likely to provide for a patient? a. Hygiene assistance b. Diversional activities c. Assistance with job hunting d. Building assertiveness skills

ANS: D Assertiveness training relies on the counseling and psychoeducational skills of the nurse. Assistance with personal hygiene would usually be accomplished by a psychiatric technician or nursing assistant. Diversional activities are usually the province of recreational therapists. The patient would probably be assisted in job hunting by a social worker or vocational therapist.

6. What is the legal significance of a nurses action when a patient verbally refuses medication and the nurse gives the medication over the patients objection? The nurse: a. has been negligent. b. committed malpractice. c. fulfilled the standard of care. d. can be charged with battery.

ANS: D Battery is an intentional tort in which one individual violates the rights of another through touching without consent. Forcing a patient to take medication after the medication was refused constitutes battery. The charge of battery can be brought against the nurse. The medication may not necessarily harm the patient; harm is a component of malpractice.

11. A nurse listens to a group of recent retirees. One says, I volunteer with Meals on Wheels, coach teen sports, and do church visitation. Another laughs and says, Im too busy taking care of myself to volunteer to help others. Which developmental task do these statements contrast? a. Trust and mistrust b. Intimacy and isolation c. Industry and inferiority d. Generativity and self-absorption

ANS: D Both retirees are in middle adulthood, when the developmental crisis to be resolved is generativity versus self-absorption. One exemplifies generativity; the other embodies self-absorption. This developmental crisis would show a contrast between relating to others in a trusting fashion and being suspicious and lacking trust. Failure to negotiate this developmental crisis would result in a sense of inferiority or difficulty learning and working as opposed to the ability to work competently. Behaviors that would be contrasted would be emotional isolation and the ability to love and commit oneself.

18. A nurse is part of a multidisciplinary team working with groups of depressed patients. Half the patients receive supportive interventions and antidepressant medication. The other half receives only medication. The team measures outcomes for each group. Which type of study is evident? a. Incidence b. Prevalence c. Co-morbidity d. Clinical epidemiology

ANS: D Clinical epidemiology is a broad field that addresses studies of the natural history (or what happens if there is no treatment and the problem is left to run its course) of an illness, studies of diagnostic screening tests, and observational and experimental studies of interventions used to treat people with the illness or symptoms. Prevalence refers to numbers of new cases. Co-morbidity refers to having more than one mental disorder at a time.Incidence refers to the number of new cases of mental disorders in a healthy population within a given period.

10. Clinical pathways are used in managed care settings to: a. stabilize aggressive patients. b. identify obstacles to effective care. c. relieve nurses of planning responsibilities. d. streamline the care process and reduce costs.

ANS: D Clinical pathways provide guidelines for assessments, interventions, treatments, and outcomes as well as a designated timeline for accomplishment. Deviations from the timeline must be reported and investigated. Clinical pathways streamline the care process and save money. Care pathways do not identify obstacles or stabilize aggressive patients. Staff are responsible for the necessary interventions. Care pathways do not relieve nurses of the responsibility of planning; pathways may, however, make the task easier.

24. A single parent is experiencing feelings of inadequacy related to work and family since one teenaged child ran away several weeks ago. The parent seeks the help of a therapist specializing in cognitive therapy. The psychotherapist who uses cognitive therapy will treat the patient by: a. discussing ego states. b. focusing on unconscious mental processes. c. negatively reinforcing an undesirable behavior. d. helping the patient identify and change faulty thinking.

ANS: D Cognitive therapy emphasizes the importance of changing erroneous ways people think about themselves. Once faulty thinking changes, the individual's behavior changes. Focusing on unconscious mental processes is a psychoanalytic approach. Negatively reinforcing undesirable behaviors is behavior modification, and discussing ego states relates to transactional analysis.

21. A patient with psychosis became aggressive, struck another patient, and required seclusion. Select the best documentation. a. Patient struck another patient who attempted to leave day room to go to bathroom. Seclusion necessary at 1415. Plan: Maintain seclusion for 8 hours and keep these two patients away from each other for 24 hours. b. Seclusion ordered by physician at 1415 after command hallucinations told the patient to hit another patient. Careful monitoring of patient maintained during period of seclusion. c. Seclusion ordered by MD for aggressive behavior. Begun at 1415. Maintained for 2 hours without incident. Outcome: Patient calmer and apologized for outburst. d. Patient pacing, shouting. Haloperidol 5 mg given PO at 1300. No effect by 1315. At 1415 patient yelled, Ill punch anyone who gets near me, and struck another patient with fist. Physically placed in seclusion at 1420. Seclusion order obtained from MD at 1430.

ANS: D Documentation must be specific and detail the key aspects of care. It should demonstrate implementation of the least restrictive alternative. Justification for why a patient was secluded should be recorded, along with interventions attempted in an effort to avoid seclusion. Documentation should include a description of behavior and verbalizations, interventions tried and their outcomes, and the name of the health care provider ordering the use of seclusion.

10. A patient expresses a desire to be cared for by others and often behaves in a helpless fashion. Which stage of psychosexual development is most relevant to the patients needs? a. Latency b. Phallic c. Anal d. Oral

ANS: D Fixation at the oral stage sometimes produces dependent infantile behaviors in adults. Latency fixations often result in difficulty identifying with others and developing social skills, resulting in a sense of inadequacy and inferiority. Phallic fixations result in having difficulty with authority figures and poor sexual identity. Anal fixation sometimes results in retentiveness, rigidity, messiness, destructiveness, and cruelty.

13. In the majority culture of the United States, which individual is at greatest risk to be incorrectly labeled mentally ill? a. Person who is usually pessimistic but strives to meet personal goals b. Wealthy person who gives $20 bills to needy individuals in the community c. Person with an optimistic viewpoint about life and getting his or her own needs met d. Person who attends a charismatic church and describes hearing God's voice

ANS: D Hearing voices is generally associated with mental illness; however, in charismatic religious groups, hearing the voice of God or a prophet is a desirable event. In this situation, cultural norms vary, making it more difficult to make an accurate DSM-5 diagnosis. The individuals described in the other options are less likely to be labeled as mentally ill.

1. A patient tells members of a therapy group, I hear voices saying my doctor is poisoning me. Another patient replies, I used to hear voices too. They sounded real, but I found out later they were not. The voices you hear are not real either. Which therapeutic factor is exemplified in this interchange? a. Catharsis b. Universality c. Imitative behavior d. Interpersonal learning

ANS: D Here a member gains insight into his own experiences from hearing about the experiences of others through interpersonal learning. Catharsis refers to a therapeutic discharge of emotions. Universality refers to members realizing their feelings are common to most people and not abnormal. Imitative behavior involves copying or borrowing the adaptive behavior of others.

8. When a nurse asks a newly admitted patient to describe social supports, the patient says, My parents died last year and I have no family. I am newly divorced, and my former in-laws blame me. I dont have many friends because most people my age just want to go out drinking. Which action will the nurse apply? a. Advise the patient that being so particular about potential friends reduces social contact. b. Suggest using the Internet as a way to find supportive others with similar values. c. Encourage the patient to begin dating again, perhaps with members of the church. d. Discuss how divorce support groups could increase coping and social support.

ANS: D High-quality social support enhances mental and physical health and acts as a significant buffer against distress. Low-quality support relationships affect a persons coping negatively. Resuming dating soon after a divorce could place additional stress on the patient rather than helping her cope with existing stressors. Developing relationships on the Internet probably would not substitute fully for direct contact with other humans and could expose her to predators misrepresenting themselves to take advantage of vulnerable persons.

12. Which remark by a group participant would the nurse expect during the working stage of group therapy? a. My problems are very personal and private. How do I know people in this group will not tell others what you hear? b. I have enjoyed this group. Its hard to believe that a few weeks ago I couldnt even bring myself to talk here. c. One thing everyone seems to have in common is that sometimes its hard to be honest with those you love most. d. I dont think I agree with your action. It might help you, but it seems like it would upset your family.

ANS: D In the working stage, members actively interact to help each other accomplish goals, and because trust has developed, conflict and disagreement can be expressed. Focusing on trust and confidentiality typically occur in the orientation phase as part of establishing group norms. Commonality and universality are also themes typically expressed in the orientation phase, whereas reflecting on progress is a task addressed in the termination phase.

11. A patient tells the nurse, I know that I should reduce the stress in my life, but I have no idea where to start. What would be the best initial nursing response? a. Physical exercise works to elevate mood and reduce anxiety. b. Reading about stress and how to manage it might be a good place to start. c. Why not start by learning to meditate? That technique will cover everything. d. Lets talk about what is going on in your life and then look at possible options.

ANS: D In this case, the nurse lacks information about what stressors the patient is coping with or about what coping skills are already possessed. Further assessment is indicated before potential solutions can be explored. Suggesting exploration of the stress facing the patient is the only option that involves further assessment rather than suggesting a particular intervention.

22. A person says, I was the only survivor in a small plane crash. Three business associates died. I got depressed and saw a counselor twice a week for 4 weeks. We talked about my feelings related to being a survivor, and Im better now. Which type of therapy was used? a. Milieu therapy b. Psychoanalysis c. Behavior modification d. Interpersonal psychotherapy

ANS: D Interpersonal psychotherapy returned the patient to his former level of functioning by helping him come to terms with the loss of friends and guilt over being a survivor. Milieu therapy refers to environmental therapy. Psychoanalysis would call for a long period of exploration of unconscious material. Behavior modification would focus on changing a behavior rather than helping the patient understand what is going on in his life.

8. Which patient meets criteria for involuntary hospitalization for psychiatric treatment? The patient who: a. is noncompliant with the treatment regimen. b. fraudulently files for bankruptcy. c. sold and distributed illegal drugs. d. threatens to harm self and others.

ANS: D Involuntary hospitalization protects patients who are dangerous to themselves or others and cannot care for their own basic needs. Involuntary commitment also protects other individuals in society. The behaviors described in the other options are not sufficient to require involuntary hospitalization.

15. A Puerto Rican American patient uses dramatic body language when describing emotional discomfort. Which analysis most likely explains the patients behavior? The patient: a. has a histrionic personality disorder. b. believes dramatic body language is sexually appealing. c. wishes to impress staff with the degree of emotional pain. d. belongs to a culture in which dramatic body language is the norm.

ANS: D Members of Hispanic American subcultures tend to use high affect and dramatic body language as they communicate. The other options are more remote possibilities.

8. A patient tells a nurse, "I have psychiatric problems and am in and out of hospitals all the time. Not one of my friends or relatives has these problems." Select the nurse's best response. a. "Comparing yourself with others has no real advantages." b. "Why do you blame yourself for having a psychiatric illness?" c. "Mental illness affects 50% of the adult population in any given year." d. "It sounds like you are concerned that others don't experience the same challenges as you."

ANS: D Mental illness affects many people at various times in their lives. No class, culture, or creed is immune to the challenges of mental illness. The correct response also demonstrates the use of reflection, a therapeutic communication technique. It is not true that mental illness affects 50% of the population in any given year. Asking patients if they blame themselves is an example of probing.

11. Nursing behaviors associated with the implementation phase of nursing process are concerned with: a. participating in mutual identification of patient outcomes. b. gathering accurate and sufficient patient-centered data. c. comparing patient responses and expected outcomes. d. carrying out interventions and coordinating care.

ANS: D Nursing behaviors relating to implementation include using available resources, performing interventions, finding alternatives when necessary, and coordinating care with other team members.

4. A nurse interacts with a newly hospitalized patient. Select the nurses comment that applies the communication technique of offering self. a. Ive also had traumatic life experiences. Maybe it would help if I told you about them. b. Why do you think you had so much difficulty adjusting to this change in your life? c. I hope you will feel better after getting accustomed to how this unit operates. d. Id like to sit with you for a while to help you get comfortable talking to me.

ANS: D Offering self is a technique that should be used in the orientation phase of the nurse-patient relationship. Sitting with the patient, an example of offering self, helps to build trust and convey that the nurse cares about the patient. Two incorrect responses are ineffective and non-therapeutic. The other incorrect response is therapeutic but is an example of offering hope.

16. A soldier returned home last year after deployment to a war zone. The soldier's spouse complains, "We were going to start a family but now he won't talk about it. He will not look at children. I wonder if we're going to make it as a couple." Select the nurse's best response. a. "Post-traumatic stress disorder often changes a person's sexual functioning." b. "I encourage you to continue to participate in social activities where children are present." c. "Have you talked with your spouse about these reactions? Sometimes we just need to confront behavior." d. "Post-traumatic stress disorder often strains relationships. I will suggest some community resources for help and support."

ANS: D PSTD precipitates changes that often lead to divorce. Providing support to both the veteran and spouse is important. Confrontation will not be effective. Although providing information is important, ongoing support is more effective.

12. A soldier served in combat zones in Iraq in 2010 and was deployed to Afghanistan in 2013. When is it most important for the nurse to screen for signs and symptoms of post-traumatic stress disorder (PTSD)? a. Immediately upon return to the United States from Afghanistan b. Before departing Afghanistan to return to the United States c. One year after returning from Afghanistan d. Screening should be ongoing

ANS: D PTSD can have a long lag time—months to years. Screening should be ongoing.

17. A nurse assessing a new patient asks, "What is meant by the saying, 'You can't judge a book by its cover'?" Which aspect of cognition is the nurse assessing? a. Mood b. Attention c. Orientation d. Abstraction

ANS: D Patient interpretation of proverbial statements gives assessment information regarding the patient's ability to abstract, which is an aspect of cognition. Mood, orientation, and attention span are assessed in other ways.

4. The relapse of a patient diagnosed with schizophrenia is related to medication noncompliance. The patient is hospitalized for 5 days, medication is restarted, and the patient's thoughts are now more organized. The patient's family members are upset and say, "It's too soon for discharge. Hospitalization is needed for at least a month." The nurse should: a. call the psychiatrist to come explain the discharge rationale. b. explain that health insurance will not pay for a longer stay for the patient. c. call security to handle the disturbance and escort the family off the unit. d. explain that the patient will continue to improve if medication is taken regularly.

ANS: D Patients no longer stay in the hospital until every vestige of a symptom disappears. The nurse must assume responsibility to advocate for the patient's right to the least restrictive setting as soon as the symptoms are under control and for the right of citizens to control health care costs. The health care provider will use the same rationale. Shifting blame will not change the discharge. Calling security is unnecessary. The nurse can handle this matter.

15. Which assessment finding for a patient living in the community requires priority intervention by the nurse? The patient: a. receives Social Security disability income plus a small check from a trust fund. b. lives in an apartment with two patients who attend day hospital programs. c. has a sibling who is interested and active in care planning. d. purchases and uses marijuana on a frequent basis.

ANS: D Patients who regularly buy illegal substances often become medication noncompliant. Medication noncompliance, along with the disorganizing influence of illegal drugs on cellular brain function, promotes relapse. The remaining options do not suggest problems.

23. Which nursing intervention below is part of the scope of an advanced practice psychiatric/mental health nurse only? a. Coordination of care b. Health teaching c. Milieu therapy d. Psychotherapy

ANS: D Psychotherapy is part of the scope of practice of an advanced practice nurse. The distracters are within a staff nurses scope of practice.

9. Which individual is demonstrating the highest level of resilience? One who: a. is able to repress stressors. b. becomes depressed after the death of a spouse. c. lives in a shelter for two years after the home is destroyed by fire. d. takes a temporary job to maintain financial stability after loss of a permanent job.

ANS: D Resilience is closely associated with the process of adapting and helps people facing tragedies, loss, trauma, and severe stress. It is the ability and capacity for people to secure the resources they need to support their well-being. Repression and depression are unhealthy. Living in a shelter for two years shows a failure to move forward after a tragedy.

1. An 86-year-old, previously healthy and independent, falls after an episode of vertigo. Which behavior by this patient best demonstrates resilience? The patient: a. says, "I knew this would happen eventually." b. stops attending her weekly water aerobics class. c. refuses to use a walker and says, "I don't need that silly thing." d. says, "Maybe some physical therapy will help me with my balance."

ANS: D Resiliency is the ability to recover from or adjust to misfortune and change. The correct response indicates that the patient is hopeful and thinking positively about ways to adapt to the vertigo. Saying "I knew this would happen eventually" and discontinuing healthy activities suggest a hopeless perspective on the health change. Refusing to use a walker indicates denial.

5. The desired outcome for a patient experiencing insomnia is, Patient will sleep for a minimum of 5 hours nightly within 7 days. At the end of 7 days, review of sleep data shows the patient sleeps an average of 4 hours nightly and takes a 2-hour afternoon nap. What is the nurses next action? a. Continue the current plan without changes. b. Remove this nursing diagnosis from the plan of care. c. Write a new nursing diagnosis that better reflects the problem. d. Examine interventions for possible revision of the target date.

ANS: D Sleeping a total of 5 hours at night remains a reasonable outcome. Extending the period for attaining the outcome may be appropriate. Examining interventions might result in planning an activity during the afternoon rather than permitting a nap. Continuing the current plan without changes is inappropriate. Removing this nursing diagnosis from the plan of care would be correct when the outcome was met and the problem resolved. Writing a new nursing diagnosis is inappropriate because no other nursing diagnosis relates to the problem.

12. Although ego defense mechanisms and security operations are mainly unconscious and designed to relieve anxiety, the major difference is that: a. defense mechanisms are intrapsychic and not observable. b. defense mechanisms cause arrested personal development. c. security operations are masterminded by the id and superego. d. security operations address interpersonal relationship activities.

ANS: D Sullivans theory explains that security operations are interpersonal relationship activities designed to relieve anxiety. Because they are interpersonal, they are observable. Defense mechanisms are unconscious and automatic. Repression is entirely intrapsychic, but other mechanisms result in observable behaviors. Frequent, continued use of many defense mechanisms often results in reality distortion and interference with healthy adjustment and emotional development. Occasional use of defense mechanisms is normal and does not markedly interfere with development. Security operations are ego-centered.

14. A citizen at a community health fair asks the nurse, What is the most prevalent mental disorder in the United States? Select the nurses best response. a. Schizophrenia b. Bipolar disorder c. Dissociative fugue d. Alzheimers disease

ANS: D The 12-month prevalence for Alzheimers disease is 10% for persons older than 65 and 50% for persons older than 85. The prevalence of schizophrenia is 1.1% per year. The prevalence of bipolar disorder is 2.6%. Dissociative fugue is a rare disorder.

13. The Diagnostic and Statistical Manual of Mental Disorders (DSM-5) classifies: a. deviant behaviors. b. present disability or distress. c. people with mental disorders. d. mental disorders people have.

ANS: D The DSM-5 classifies disorders people have rather than people themselves. The terminology of the tool reflects this distinction by referring to individuals with a disorder rather than as a schizophrenic or alcoholic, for example. Deviant behavior is not generally considered a mental disorder. Present disability or distress is only one aspect of the diagnosis.

15. A nurse wants to find a description of diagnostic criteria for a person diagnosed with schizophrenia. Which resource should the nurse consult? a. U.S. Department of Health and Human Services b. Journal of the American Psychiatric Association c. North American Nursing Diagnosis Association International (NANDA-I) d. Diagnostic and Statistical Manual of Mental Disorders (DSM-5)

ANS: D The DSM-5 identifies diagnostic criteria for psychiatric diagnoses. The other sources have useful information but are not the best resources for finding a description of the diagnostic criteria for a psychiatric disorder.

25. A health care provider prescribed depot injections every 3 weeks at the clinic for a patient with a history of medication noncompliance. For this plan to be successful, which factor will be of critical importance? a. The attitude of significant others toward the patient b. Nutrition services in the patients neighborhood c. The level of trust between the patient and nurse d. The availability of transportation to the clinic

ANS: D The ability of the patient to get to the clinic is of paramount importance to the success of the plan. The depot medication relieves the patient of the necessity to take medication daily, but if he or she does not receive the injection at 3-week intervals, non-adherence will again be the issue. Attitude toward the patient, trusting relationships, and nutrition are important but not fundamental to this particular problem.

13. A Filipino American patient had a nursing diagnosis of situational low self-esteem related to poor social skills as evidenced by lack of eye contact. Interventions were used to raise the patients self-esteem, but after 3 weeks, the patients eye contact did not improve. What is the most accurate analysis of this scenario? a. The patients eye contact should have been directly addressed by role-playing to increase comfort with eye contact. b. The nurse should not have independently embarked on assessment, diagnosis, and planning for this patient. c. The patients poor eye contact is indicative of anger and hostility that were unaddressed. d. The nurse should have assessed the patients culture before making this diagnosis and plan.

ANS: D The amount of eye contact a person engages in is often culturally determined. In some cultures, eye contact is considered insolent, whereas in others eye contact is expected. Asian Americans, including persons from the Philippines, often prefer not to engage in direct eye contact.

10. Guidelines followed by the leader of a therapeutic group include focusing on recognizing dysfunctional behavior and thinking patterns, followed by identifying and practicing more adaptive alternate behaviors and thinking. Which theory is evident by this approach? a. Behavioral b. Interpersonal c. Psychodynamic d. Cognitive-behavioral

ANS: D The characteristics described are those of cognitive-behavioral therapy, in which patients learn to reframe dysfunctional thoughts and extinguish maladaptive behaviors. Behavioral therapy focuses solely on changing behavior rather than thoughts, feelings, and behaviors together. Interpersonal theory focuses on interactions and relationships. Psychodynamic groups focus on developing insight to resolve unconscious conflicts.

1. The adult child of a patient diagnosed with major depression asks, Do you think depression and physical illness are connected? Since my fathers death, my mother has had shingles and the flu, but shes usually not one who gets sick. Which answer by the nurse best reflects current knowledge about psychoneuroimmunology? a. It is probably a coincidence. Emotions and physical responses travel on different tracts of the nervous system. b. You may be paying more attention to your mother since your father died and noticing more things such as minor illnesses. c. So far, research on emotions or stress and becoming ill more easily is unclear. We do not know for sure if there is a link. d. Negative emotions and stress may interfere with the bodys ability to protect itself and can increase the likelihood of infection.

ANS: D The correct answer best explains the research. Research supports a link between negative emotions and/or prolonged stress and impaired immune system functioning. Activation of the immune system sends proinflammatory cytokines to the brain, and the brain in turn releases its own cytokines that signal the central nervous system to initiate myriad responses to stress. Prolonged stress suppresses the immune system and lowers resistance to infections. Although the adult child may be more aware of issues involving the mother, the pattern of illnesses described may be an increase from the mothers baseline.

19. A school age child tells the school nurse, Other kids call me mean names and will not sit with me at lunch. Nobody likes me. Select the nurses most therapeutic response. a. Just ignore them and they will leave you alone. b. You should make friends with other children. c. Call them names if they do that to you. d. Tell me more about how you feel.

ANS: D The correct response uses exploring, a therapeutic technique. The distracters give advice, a non-therapeutic technique.

3. A 26-month-old displays negative behavior, refuses toilet training, and often says, No! Which psychosocial crisis is evident? a. Trust versus mistrust b. Initiative versus guilt c. Industry versus inferiority d. Autonomy versus shame and doubt

ANS: D The crisis of autonomy versus shame and doubt relates to the developmental task of gaining control of self and environment, as exemplified by toilet training. This psychosocial crisis occurs during the period of early childhood. Trust versus mistrust is the crisis of the infant. Initiative versus guilt is the crisis of the preschool and early-school-aged child. Industry versus inferiority is the crisis of the 6- to 12-year-old child.

20. During the first interview, a nurse notices that the patient does not make eye contact. The nurse can correctly analyze that: a. the patient is not truthful. b. the patient is feeling sad. c. the patient has a poor self-concept. d. more information is needed to draw a conclusion.

ANS: D The data are insufficient to draw a conclusion. The nurse must continue to assess.

24. A nurse documents: Patient is mute despite repeated efforts to elicit speech. Makes no eye contact. Inattentive to staff. Gazes off to the side or looks upward rather than at speaker. Which nursing diagnosis should be considered? a. Defensive coping b. Decisional conflict c. Risk for other-directed violence d. Impaired verbal communication

ANS: D The defining characteristics are more related to the nursing diagnosis of impaired verbal communication than to the other nursing diagnoses.

24. Which documentation of a patients behavior best demonstrates a nurses observations? a. Isolates self from others. Frequently fell asleep during group. Vital signs stable. b. Calmer; more cooperative. Participated actively in group. No evidence of psychotic thinking. c. Appeared to hallucinate. Frequently increased volume on television, causing conflict with others. d. Wore four layers of clothing. States, I need protection from evil bacteria trying to pierce my skin.

ANS: D The documentation states specific observations of the patients appearance and the exact statements made. The other options are vague or subjective statements and can be interpreted in different ways.

18. A family member of a patient with delusions of persecution asks the nurse, Are there any circumstances under which the treatment team is justified in violating a patients right to confidentiality? The nurse should reply that confidentiality may be breached: a. under no circumstances. b. at the discretion of the psychiatrist. c. when questions are asked by law enforcement. d. if the patient threatens the life of another person.

ANS: D The duty to warn a person whose life has been threatened by a psychiatric patient overrides the patients right to confidentiality. The right to confidentiality is not suspended at the discretion of the therapist or for legal investigations.

6. A nurse supports a parent for praising a child behaving in a helpful way. When this child behaves with politeness and helpfulness in adulthood, which feeling will most likely result? a. Guilt b. Anxiety c. Humility d. Self-esteem

ANS: D The individual will be living up to the ego ideal, which will result in positive feelings about self. The other options are incorrect because each represents a negative feeling.

16. A patients relationships are intense and unstable. The patient initially idealizes the significant other and then devalues him or her, resulting in frequent feelings of emptiness. This patient will benefit from interventions to develop which aspect of mental health? a. Effectiveness in work b. Communication skills c. Productive activities d. Fulfilling relationships

ANS: D The information given centers on relationships with others that are described as intense and unstable. The relationships of mentally healthy individuals are stable, satisfying, and socially integrated. Data are not present to describe work effectiveness, communication skills, or activities.

22. Select the best response for the nurse who receives a question from another health professional seeking to understand the difference between a Diagnostic and Statistical Manual of Mental Disorders (DSM-5) diagnosis and a nursing diagnosis. a. There is no functional difference between the two. Both identify human disorders. b. The DSM-5 diagnosis disregards culture, whereas the nursing diagnosis takes culture into account. c. The DSM-5 diagnosis describes causes of disorders whereas a nursing diagnosis does not explore etiology. d. The DSM-5 diagnosis guides medical treatment, whereas the nursing diagnosis offers a framework for identifying interventions for issues a patient is experiencing.

ANS: D The medical diagnosis is concerned with the patients disease state, causes, and cures, whereas the nursing diagnosis focuses on the patients response to stress and possible caring interventions. Both tools consider culture. The DSM-5 is multiaxial. Nursing diagnoses also consider potential problems.

15. A nurse caring for a withdrawn, suspicious patient recognizes development of feelings of anger toward the patient. The nurse should: a. suppress the angry feelings. b. express the anger openly and directly with the patient. c. tell the nurse manager to assign the patient to another nurse. d. discuss the anger with a clinician during a supervisory session.

ANS: D The nurse is accountable for the relationship. Objectivity is threatened by strong positive or negative feelings toward a patient. Supervision is necessary to work through countertransference feelings.

4. In a team meeting a nurse says, Im concerned about whether we are behaving ethically by using restraint to prevent one patient from self-mutilation, while the care plan for another self-mutilating patient requires one-on-one supervision. Which ethical principle most clearly applies to this situation? a. Beneficence b. Autonomy c. Fidelity d. Justice

ANS: D The nurse is concerned about justice, that is, fair distribution of care, which includes treatment with the least restrictive methods for both patients. Beneficence means promoting the good of others. Autonomy is the right to make ones own decisions. Fidelity is the observance of loyalty and commitment to the patient.

25. A nurse prepares to assess a new patient who moved to the United States from Central America three years ago. After introductions, what is the nurses next comment? a. How did you get to the United States? b. Would you like for a family member to help you talk with me? c. An interpreter is available. Would you like for me to make a request for these services? d. Are you comfortable conversing in English, or would you prefer to have a translator present?

ANS: D The nurse should determine whether a translator is needed by first assessing the patient for language barriers. Accuracy of the assessment depends on the ability to communicate in a language that is familiar to the patient. Family members are not always reliable translators. An interpreter may change the patients responses; a translator is a better resource.

9. A patient states, Im not worth anything. I have negative thoughts about myself. I feel anxious and shaky all the time. Sometimes I feel so sad that I want to go to sleep and never wake up. Which nursing intervention should have the highest priority? a. Self-esteem-building activities b. Anxiety self-control measures c. Sleep enhancement activities d. Suicide precautions

ANS: D The nurse would place a priority on monitoring and reinforcing suicide self-restraint because it relates directly and immediately to patient safety. Patient safety is always a priority concern. The nurse should monitor and reinforce all patient attempts to control anxiety, improve sleep patterns, and develop self-esteem, while giving priority attention to suicide self-restraint.

10. Select the best outcome for a patient with the nursing diagnosis: Impaired social interaction related to sociocultural dissonance as evidenced by stating, Although Id like to, I dont join in because I dont speak the language very well. Patient will: a. show improved use of language. b. demonstrate improved social skills. c. become more independent in decision making. d. select and participate in one group activity per day.

ANS: D The outcome describes social involvement on the part of the patient. Neither cooperation nor independence has been an issue. The patient has already expressed a desire to interact with others. Outcomes must be measurable. Two of the distracters are not measurable.

17. An adult patient recently diagnosed with cancer states, "I've lived my life according to the Bible. I don't understand why God has forsaken me." Which nursing diagnosis applies? a. Hopelessness b. Spiritual distress c. Spiritual dysfunction d. Disturbed thought processes

ANS: D The patient is experiencing distress rather than dysfunction related to spirituality.

10. A patient is brought to the Emergency Department after a motorcycle accident. The patient is alert, responsive, and diagnosed with a broken leg. The patients vital signs are pulse (P) 72 and respiration (R) 16. After being informed surgery is required for the broken leg, which vital sign readings would be expected? a. P 64, R 14 b. P 68, R 12 c. P 72, R 16 d. P 80, R 20

ANS: D The patient would experience stress associated with anticipation of surgery. In times of stress, the sympathetic nervous system takes over (fight or flight response) and sends signals to the adrenal glands, thereby releasing norepinephrine. The circulating norepinephrine increases the heart rate. Respirations increase, bringing more oxygen to the lungs.

14. A participant at a community education conference asks, "What is the most prevalent type of mental disorder in the United States?" Select the nurse's best response. a. "Why do you ask?" b. "Schizophrenia" c. "Affective disorders" d. "Anxiety disorders"

ANS: D The prevalence for schizophrenia is 1.1% per year. The prevalence of all affective disorders (e.g., depression, dysthymic disorder, bipolar) is 9.5%. The prevalence of anxiety disorders is 13.3%.

31. Which comment best indicates a patient is self-actualized? a. I have succeeded despite a world filled with evil. b. I have a plan for my life. If I follow it, everything will be fine. c. Im successful because I work hard. No one has ever given me anything. d. My favorite leisure is walking on the beach, hearing soft sounds of rolling waves.

ANS: D The self-actualized personality is associated with high productivity and enjoyment of life. Self-actualized persons experience pleasure in being alone and an ability to reflect on events.

12. Which statement made by a patient during an initial assessment interview should serve as the priority focus for the plan of care? a. I can always trust my family. b. It seems like I always have bad luck. c. You never know who will turn against you. d. I hear evil voices that tell me to do bad things.

ANS: D The statement regarding evil voices tells the nurse that the patient is experiencing auditory hallucinations and may create risks for violence. The other statements are vague and do not clearly identify the patients chief symptom.

1. A patient says to the nurse, I dreamed I was stoned. When I woke up, I felt emotionally drained, as though I hadnt rested well. Which response should the nurse use to clarify the patients comment? a. It sounds as though you were uncomfortable with the content of your dream. b. I understand what youre saying. Bad dreams leave me feeling tired, too. c. So you feel as though you did not get enough quality sleep last night? d. Can you give me an example of what you mean by stoned?

ANS: D The technique of clarification is therapeutic and helps the nurse examine the meaning of the patients statement. Asking for a definition of stoned directly asks for clarification. Restating that the patient is uncomfortable with the dreams content is parroting, a non-therapeutic technique. The other responses fail to clarify the meaning of the patients comment.

19. A patient underwent psychotherapy weekly for 3 years. The therapist used free association, dream analysis, and facilitated transference to help the patient understand unconscious processes and foster personality changes. Which type of therapy was used? a. Short-term dynamic psychotherapy b. Transactional analysis c. Cognitive therapy d. Psychoanalysis

ANS: D The therapy described is traditional psychoanalysis. Short-term dynamic psychotherapy would last less than 1 year. Neither transactional analysis nor cognitive therapy makes use of the techniques described.

3. A young female member in a therapy group says to an older female member, You are just like my mother, always trying to control me with your observations and suggestions. Which therapeutic factor of a group is evident by this behavior? a. Instillation of hope b. Existential resolution c. Development of socializing techniques d. Corrective recapitulation of the primary family group

ANS: D The younger patient is demonstrating an emotional attachment to the older patient that mirrors patterns within her own family of origin, a phenomenon called corrective recapitulation of the primary family group. Feedback from the group then helps the member gain insight about this behavior and leads to more effective ways of relating to her family members. Instillation of hope involves conveying optimism and sharing progress.Existential resolution refers to the realization that certain existential experiences such as death are part of life, aiding the adjustment to such realities. Development of socializing techniques involves gaining social skills through the groups feedback and practice within the group.

12. Which remark by a patient indicates passage from orientation to the working phase of a nurse-patient relationship? a. I dont have any problems. b. It is so difficult for me to talk about problems. c. I dont know how it will help to talk to you about my problems. d. I want to find a way to deal with my anger without becoming violent.

ANS: D Thinking about a more constructive approach to dealing with anger indicates a readiness to make a behavioral change. Behavioral change is associated with the working phase of the relationship. Denial is often seen in the orientation phase. It is common early in the relationship, before rapport and trust are firmly established, for a patient to express difficulty in talking about problems. Stating skepticism about the effectiveness of the nurse-patient relationship is more typically a reaction during the orientation phase.

27. The nurse should refer which of the following patients to a partial hospitalization program? A patient who: a. has a therapeutic lithium level and reports regularly for blood tests and clinic follow-up. b. needs psychoeducation for relaxation therapy related to agoraphobia and panic episodes. c. spent yesterday in a supervised crisis care center and continues to have active suicidal ideation. d. states, Im not sure I can avoid using alcohol when my spouse goes to work every morning.

ANS: D This patient could profit from the structure and supervision provided by spending the day at the partial hospitalization program. During the evening, at night, and on weekends, the spouse could assume responsibility for supervision. A suicidal patient needs inpatient hospitalization. The other patients can be served in the community or with individual visits.

16. Which individual with mental illness may need emergency or involuntary admission? The individual who: a. resumes using heroin while still taking naltrexone (ReVia). b. reports hearing angels playing harps during thunderstorms. c. does not keep an outpatient appointment with the mental health nurse. d. throws a heavy plate at a waiter at the direction of command hallucinations.

ANS: D Throwing a heavy plate is likely to harm the waiter and is evidence of dangerousness to others. This behavior meets the criteria for emergency or involuntary hospitalization for mental illness. The behaviors in the other options evidence mental illness but not dangerousness.

18. A patient says, People should be allowed to commit suicide without interference from others. A nurse replies, Youre wrong. Nothing is bad enough to justify death. What is the best analysis of this interchange? a. The patient is correct. b. The nurse is correct. c. Neither person is correct. d. Differing values are reflected in the two statements.

ANS: D Values guide beliefs and actions. The individuals stating their positions place different values on life and autonomy. Nurses must be aware of their own values and be sensitive to the values of others.

16. During an interview, a patient attempts to shift the focus from self to the nurse by asking personal questions. The nurse should respond by saying: a. Why do you keep asking about me? b. Nurses direct the interviews with patients. c. Do not ask questions about my personal life. d. The time we spend together is to discuss your concerns.

ANS: D When a patient tries to focus on the nurse, the nurse should refocus the discussion back onto the patient. Telling the patient that interview time should be used to discuss patient concerns refocuses discussion in a neutral way. Telling patients not to ask about the nurses personal life shows indignation. Saying that nurses prefer to direct the interview reflects superiority. Why questions are probing and non-therapeutic.

5. A nurse is talking with a patient, and 5 minutes remain in the session. The patient has been silent most of the session. Another patient comes to the door of the room, interrupts, and says to the nurse, I really need to talk to you. The nurse should: a. invite the interrupting patient to join in the session with the current patient. b. say to the interrupting patient, I am not available to talk with you at the present time. c. end the unproductive session with the current patient and spend time with the interrupting patient. d. tell the interrupting patient, This session is 5 more minutes; then I will talk with you.

ANS: D When a specific duration for sessions has been set, the nurse must adhere to the schedule. Leaving the first patient would be equivalent to abandonment and would destroy any trust the patient had in the nurse. Adhering to the contract demonstrates that the nurse can be trusted and that the patient and the sessions are important. The incorrect responses preserve the nurse-patient relationship with the silent patient but may seem abrupt to the interrupting patient, abandon the silent patient, or fail to observe the contract with the silent patient.

19. At what point in an assessment interview would a nurse ask, How does your faith help you in stressful situations? During the assessment of: a. childhood growth and development b. substance use and abuse c. educational background d. coping strategies

ANS: D When discussing coping strategies, the nurse might ask what the patient does when upset, what usually relieves stress, and to whom the patient goes to talk about problems. The question regarding whether the patients faith helps deal with stress fits well here. It would be out of place if introduced during exploration of the other topics.

14. A nurse assesses an older adult patient brought to the emergency department by a family member. The patient was wandering outside saying, I cant find my way home. The patient is confused and unable to answer questions. Select the nurses best action. a. Record the patients answers to questions on the nursing assessment form. b. Ask an advanced practice nurse to perform the assessment interview. c. Call for a mental health advocate to maintain the patients rights. d. Obtain important information from the family member.

ANS: D When the patient (primary source) is unable to provide information, secondary sources should be used, in this case, the family member. Later, more data may be obtained from other information sources familiar with the patient. An advanced practice nurse is not needed for this assessment; it is within the scope of practice of the staff nurse. Calling a mental health advocate is unnecessary.

11. A soldier returned 3 months ago from Afghanistan and was diagnosed with post-traumatic stress disorder (PTSD). Which social event would most likely be disturbing for this soldier? a. Halloween festival with neighborhood children b. Singing carols around a Christmas tree c. Family outing to the seashore d. Fireworks display on July 4th

ANS: D exploding noises associated with fireworks are most likely to provoke exaggerated responses for this soldier. The distractors are not associated with offensive sounds.

11. A college student said, Most of the time Im happy and feel good about myself. I have learned that what I get out of something is proportional to the effort I put into it. Which number on this mental health continuum should the nurse select? 1 (Mental Illness) --> 5 (Mental Health) a. 1 b. 2 c. 3 d. 4 e. 5

ANS: E The student is happy and has an adequate self-concept. The student is reality-oriented, works effectively, and has control over own behavior. Mental health does not mean that a person is always happy.

26. The nurse records this entry in a patients progress notes: Patient escorted to unit by ER nurse at 2130. Patients clothing was dirty. In interview room, patient sat with hands over face, sobbing softly. Did not acknowledge nurse or reply to questions. After several minutes, abruptly arose, ran to window, and pounded. Shouted repeatedly, Let me out of here. Verbal intervention unsuccessful. Order for stat dose 2 mg haloperidol PO obtained; medication administered at 2150. By 2215, patient stopped shouting and returned to sit wordlessly in chair. Patient placed on one-to-one observation. How should this documentation be evaluated? a. Uses unapproved abbreviations b. Contains subjective material c. Too brief to be of value d. Excessively wordy e. Meets standards

ANS: E This narrative note describes patient appearance, behavior, and conversation. It mentions that less-restrictive measures were attempted before administering medication and documents patient response to medication. This note would probably meet standards. A complete nursing assessment would be in order as soon as the patient is able to participate. Subjective material is absent from the note. Abbreviations are acceptable.

As a fact witness, what will the nurse be asked to testify about? A. What he/she was told by staff caring for the client. B. What he/she personally witnessed regarding the client's care. C. What was documented in the client's health record. D. The complexity of the client's care.

B A fact witness testifies regarding what was personally seen or heard, performed, or documented regarding a patient's care and testifies to first-hand experience only. Fact witnesses cannot testify to what was told to them by another person, only what they witnessed themselves. The nurse may testify only to documentation that he/she performed. An expert witness is recognized by the Court as having a certain level of skill or expertise in a designated area and possesses superior knowledge because of education or specialized experience. This is not the same as a fact witness.

When considering client rights, which client can be legally medicated against his or her wishes? A. The client has accepted the medication in the past. B. The client may cause imminent harm to himself or others. C. The client's primary provider orders the medication. D. The client's mental illness may relate to cognitive impairment.

B A patient may be medicated against his or her will without a court hearing in an emergency if the patient poses a danger to himself or herself or to others. The other options are not legally valid reasons to give medication against a patient's will.

Consider both Sullivan's term security operations and Freud's term defense mechanisms. Which statement suggests that the client's specialized treatment goal has been successfully met? A. "I really think I can succeed in school now." B. "I'm experiencing much less anxiety about school now." C. "Going back to school is hard and I'll need support." D. "I know that I'm not the only person who has a difficult time in school."

B Both Sullivan and Freud coined terms to mean actions that individuals do that are an attempt to reduce anxiety. The terms to do not refer to activities that increase self-esteem. Security operations and defense mechanisms are not conscious and therefore do not increase self-awareness. These terms do not refer to reducing cognitive distortions.

Which statement by a patient who has been taught cognitive reframing indicates that the teaching was successful? A. "I do not have the ability to handle that job." B. "I can be successful if I do all the things required to learn the job." C. "I may be fired from the job but eventually I will find something else to do with my life." D. "I can never learn all there is to know for the job."

B Cognitive reframing changes the individual's perceptions of stress by reassessing a situation and replacing irrational beliefs with more positive self-statements. The other options are all negative cognitive distortions that would prevent the individual from success.

Which client behavior illustrates eustress? A. A college student fails an exam. B. A bride is planning for her wedding. C. A man is laid off from his job. D. An adolescent gets into a fight at school.

B Eustress is the result of a positive perception toward a stressor, such as having a baby, planning a wedding, or getting a new job. The other options all describe distress, or a negative energy.

The nurse is caring for an admitted client with a history of becoming aggressive when angry and has caused physical injury to another client and two staff members. When this client begins to demonstrate signs of anger while in the day room what intervention should the nurse implement to address the safety of the milieu? A. Alert security to come to the unit for a show of strength B. Request that the client accompany the nurse to the client's room C. Inform the client that restraints will be used if the behavior continues D. Prepare to administer a prn chemical restraint to the client

B Least restrictive alternative doctrine requires using the least drastic means of achieving a specific goal. By first attempting to remove the client to a safer location, the nurse is respecting the client's right to treatment that is less restrictive than the other options.

A nurse, active in local consumer mental health groups and in local and state mental health associations, keeps aware of state and national legislation affecting mental illness treatment. How can this nurse positively affect the climate for effective, mental health treatment? A. "By becoming active in politics leading to a potential political career." B. "By educating the public on the effects that stigmatizing has on mental health clients." C. "Advocating for laws that would make the involuntary long-term commitment process easier and faster for caregivers of mentally ill persons." D. "Advocating for reduced mental health insurance benefits to discourage abuse of the system by inappropriate psychiatric admissions."

B Nurses who are aware of legislative concerns and who are active in organizations that promote mental health awareness and appropriate and equal treatment for mental illness help achieve the goal of parity, or equality of treatment for mentally ill individuals. Becoming active in politics may be a personal goal but does not directly or necessarily reduce stigma or encourage treatment equality. The other options are undesirable outcomes.

Resilience is characteristic of mental health that allows people to adapt to tragedies, trauma, and loss. Which client behavior demonstrates this characteristic? A. "My mother made decisions about my husband's funeral when I just couldn't do that." B. "Losing my job was hard but my skills will help me get another one." C. "In spite of all the treatment, I know I'll never be really healthy." D. "My kids, happiness is worth any sacrifice I have to make."

B Resilience is a characteristic that helps individuals cope with loss and trauma that may occur in life. Dependence is described as relying on others for decision making and care. Pessimism is a life philosophy that things are more likely to go wrong than right. Altruism is described as putting others before yourself.

A client states, "I will always be alone because nobody could ever love me." The nurse recognizes that the client is expressing what cognitive-behavioral concept? A. Emotional consequence B. Schema C. Actualization D. Aversion

B Schemas are unique assumptions about ourselves, according to Beck's theory. This statement is an example of a negative schema. Emotional consequence is the end result of negative thinking process, as described by Ellis. Actualization is a level of Maslow's Hierarchy of Needs. Aversion is a therapy characterized by punishment.

A 55-year-old patient recently came to the United States from England on a work visa. The patient was admitted for severe depression following the death of a life partner weeks ago. While discussing the death and its effects the patient shows little emotion. Which of the following explanations is most plausible for this lack of emotion? A. The patient in denial. B. The response may reflect cultural norms. C. The response may reflect personal guilt. D. The patient may have an antisocial personality.

B Showing little emotion while in distress may be a cultural phenomenon. Some cultures, such as the British and German cultures, tend to value highly the concept of self-control and may show little facial emotion in the presence of emotional turmoil. There is no evidence to suggest the patient's lack of emotion is a result of any of the other options.

The nurse is conducting an admission interview with a client who was raped 2 weeks ago. When asked about the rape, the client becomes very anxious and upset and begins to sob. What should be the nurse's response to the client's reaction? A. Push gently for more information about the rape because the information needs to be documented. B. Acknowledge that the topic of the rape is upsetting and reassure the client that it can be discussed at another time when she feels more comfortable. C. Use silence as a therapeutic tool and wait until the client is done sobbing to continue discussing the rape. D. Reassure the client that anything she says to you will remain confidential.

B The best atmosphere for conducting an assessment is one with minimal anxiety on the patient's part. If a topic causes distress, it is best to abandon the topic at that time. It is important not to pry or push for information that is difficult for the patient to discuss. The use of silence continues to expect the patient to discuss the topic now. Reassurance of confidentiality continues to expect the patient to discuss the topic now.

A 43-year-old client being seen in the mental health clinic states, "I have always been a practicing Jew, but in the past few months I am questioning everything. I just don't know if I believe in it anymore." Which of the following nursing diagnoses best describes the client's comment? A. Ineffective coping B. Spiritual distress C. Risk for self-harm D. Hopelessness

B The client is expressing distress regarding his religion and spiritual well-being. The client could be experiencing ineffective coping, but this does not directly relate to his comment. There is nothing in the client's comment that would lead to the conclusion that the patient is having thoughts of harming himself or experiencing hopelessness.

After interviewing a client regarding his/her incarceration, a forensic psychiatric nurse examiner feels the client is quite possibly innocent. What principle should the nurse use to guide his/her actions regarding this opinion in an official report? A. Gut feelings are usually a good indicator of guilt or innocence. B. One must separate personal opinion from professional opinion. C. A personal opinion must be identified when included in an evidence-based report. D. The opinion of a second forensic nurse examiner should be included in the report.

B The forensic psychiatric nurse examiner must be able to separate personal opinion from professional opinion. Personal opinion is based on one's background, upbringing, education, and value system. Professional opinion is based on scientific principle, advanced education in a specific field of endeavor, and the unbiased standards set by research in that area. Professionals cannot rely on gut feelings. Personal opinions are not given. Calling in another forensic nurse examiner is not needed or appropriate. The nurse should report a professional opinion based only on facts and unbiased standards.

A nurse on the psychiatric unit has a past history of alcoholism and has regular meetings with a mentor. Which statement made to the nurse's mentor would indicate the presence of countertransference? A. "My patient is being discharged tomorrow. I provided discharge teaching and stressed the importance of calling the help line number should she become suicidal again." B. "My patient has been abusing alcohol. I told her that the only way to recover was to go 'cold turkey' and to get away from her dysfunctional family and to do it now!" C. "My patient started drinking after 14 years of sobriety. We are focusing on his treatment plan of attending AA (Alcoholics Anonymous) meetings five times a week after discharge." D. "My patient, is an elderly woman with depression. She calls me by her daughter's name because she says I remind her of her daughter."

B This statement indicates countertransference; the nurse may be overidentifying with the patient because of a past history of alcoholism. Providing adamant advice to the patient that, besides being nontherapeutic, may be more relevant to personal past experiences than to the patient's. The discharge teaching for a patient being discharged and focusing on the treatment plan for the alcoholic patient are appropriate and show no signs of countertransference. The patient calling the nurse by her daughter's name is transference rather than countertransference.

When considering the functions of a forensic psychiatric nurse, which statement regarding the role of hostage negotiator is true? A. It is not within the forensic psychiatric nurse's scope of practice. B. It is not covered by malpractice nursing insurance. C. It is a function of a forensic psychiatric nurse. D. It is performed only by forensic psychiatric nurses with special training.

C Acting as a hostage negotiator is a sanctioned function of the forensic psychiatric nurse examiner. Since it is within the scope of practice for this nursing specialty, malpractice insurance would cover such situations.

A 17-year-old patient confides to the nurse that they have been thinking of ways to kill a peer. What response should the nurse give when the patient states, "you have to keep it a secret because its confidential information"? A. "I will keep it a secret, but you and I need to discuss ways to deal with this situation appropriately without committing a crime." B. "Yes, I will keep it confidential. We have laws to protect patients' confidentiality." C. "Issues of this kind have to be shared with the treatment team and your parents." D. "I will have to share this with the treatment team, but we will not share it with your parents."

C Although adolescent patients request confidentiality, issues of sexual abuse, threats of suicide or homicide, or issues that put the patient at risk for harm must be shared with the treatment team and the parents. A threat of this nature must be discussed with the treatment team and the parents. Confidentiality laws do not protect information that would lead to harm to the patient or others.

A nurse is providing care to a 28-year-old patient diagnosed with bipolar disorder who was admitted in a manic state. According to Maslow's Hierarchy of Needs theory, the nurse should identify which patient symptom as having priority? A. Rapid, pressured speech B. Grandiose thoughts C. Lack of sleep D. Hyperactive behavior

C Based on Maslow's theory, physiological needs such as food, water, air, sleep, etc., are the priority and must be taken care of first. The other options are symptoms of mania but not as critical as lack of sleep.

A new nurse has accepted a position as staff nurse on a psychiatric unit. Which statement made by the new nurse requires additional instructions regarding the therapies provided on the unit? A. "You will participate in unit activities and groups daily." B. "You will be given a schedule daily of the groups we would like you to attend." C. "You will attend a psychotherapy group that I lead that will help you care for yourself." D. "You will see your provider daily in a one-to-one session."

C Basic level RNs cannot perform psychotherapy. The other options are all appropriate expectations of a patient's schedule on a psychiatric unit.

A patient is sitting with arms crossed over his or her chest, his or her left leg is rapidly moving up and down, and there is an angry expression on his or her face. When approached by the nurse, the patient states harshly, "I'm fine! Everything's great." Which statement related to communication should the nurse focus on when working with this patient? A. Verbal communication is always more accurate than nonverbal communication. B. Verbal communication is more straightforward, whereas nonverbal communication does not portray what a person is thinking. C. Nonverbal and verbal communication may be different; nurses must pay attention to the nonverbal communication being presented to get an accurate message. D. Nonverbal communication is about 10% of all communication, and verbal communication is about 90%.

C Communication is roughly 10% verbal and 90% nonverbal, so nurses must pay close attention to nonverbal cues to accurately assess what the patient is really feeling. The other options are all untrue of verbal and nonverbal communication and are actually the opposite of what is believed of communication.

Which scenarios describe a HIPAA violation associated with a nurse's behavior? A. An ED (Emergency Department) nurse gives the intensive care unit nurse a client report from a telephone at the nurse's station. B. A nurse on the medical-surgical floor calls a patient's primary care provider to obtain a list of current medications. C. A nurse on the cardiac unit gives report to the nurse on the step-down unit while transporting a client in the staff elevator. D. A nurse on the psychiatric unit gives discharge information to the counseling office regarding a client's outpatient treatment.

C Discussing a patient's information in public places where it may be overheard is a violation of a patient's confidentiality. The other options describe appropriate interactions for patient continuity of care and support of the treatment plan by the health care team.

Which of the following patients meets the criteria for an involuntary admission to a psychiatric mental health unit? A. A 23-year-old college student who has developed symptoms of anxiety and is missing classes and work B. A 30-year-old accountant who has developed symptoms of depression C. A 26-year-old kindergarten teacher who is not in touch with reality and was found wandering in and out of traffic on a busy road D. A 76-year-old retired librarian who is experiencing memory loss and some confusion at times

C Inpatient involuntary admission is reserved for patients who are at risk for self-harm or who cannot adequately protect themselves from harm because of their illness (e.g., a psychotic patient). The other options can all be managed at this point in the community setting and don't meet criteria (risk of harm to self and/or others) for admission.

Which of the following patients would be appropriate to refer to a partial hospitalization program (PHP)? A. A depressed patient with a suicidal plan B. A patient being discharged from an inpatient alcohol rehabilitation unit C. A client who has stopped taking his or her antipsychotic medication and is neglecting his or her basic needs D. Jeff, who has mild depression symptoms and is starting outpatient therapy

C PHP is for patients who may need a "step-down" environment from inpatient status or for those who are being diverted from hospitalization with intensive, short-term care from which they return home each day. This patient would be a good candidate after completing alcohol rehab; PHP could possibly help prevent relapse in the early stages after rehab. This patient can be managed with regular outpatient therapy and does not need intensive short-term therapy such as PHP. Someone who is suicidal would require inpatient hospitalization for safety as would someone who is decompensated and not caring for herself. A patient exhibiting mild depression would be managed with outpatient therapy and would not need intensive short-term therapy such as PHP.

Jacob is a college student whose friend recently committed suicide. Jacob rates his stress as low. Melissa was also friend with the person who committed suicide, but she rates her stress as high. The difference in how Jacob and Melissa rate their stress may be explained by which coping mechanism? A. Projection B. Denial C. Perception D. Repression

C Perception, which is influenced by gender, culture, age, and life experience, plays a part in how someone will respond to a stress. The perception of a stressor determines the person's emotional and psychological reactions to it. The other options are all defense mechanisms that do not explain the difference in reactions to a stressor.

A 26-year-old patient is brought to the emergency room by a friend. The patient is unable to give any coherent history. Which response should the nurse provide when the patient's friend offers to provide information regarding the patient? A. "I'm sorry, but I cannot take any information from you as it would violate confidentiality laws." B. "There is no need for that as I will call his primary care provider to obtain the information we need." C. "Yes, I will be happy to get any information and history that you can provide." D. "Yes, however, we will have to get a release signed from the patient for you to be able to talk with me."

C The friend is a secondary source of information that will be helpful since the patient is not able to give any history or information at this time. Confidentiality laws do not prohibit obtaining information from a secondary source. The friend can provide information and/or history immediately and may be able to relate events that happened just before coming to the hospital. A release would not be necessary to take information about the patient from a secondary source, and a psychotic patient would not be competent to sign a release.

A patient who recently loss a parent begins crying during a one-to-one session with the nurse. Which of the following responses by the nurse illustrates empathy? A. "I'm so sorry. My father died 2 years ago, so I know how you are feeling." B. "You need to focus on yourself right now. You deserve to take time just for you." C. "That must have been such a hard situation for you to deal with." D. "I know that you will get over this. It just takes time."

C This response reflects understanding of the patient's feelings, which is empathy. Feeling sorry for the client represents sympathy, whereas not addressing the patient's concern belittles the patient's feelings of grief she is expressing by changing the subject. Telling the patient she will get over it does not reflect empathy and is closed-ended.

The primary goal and benefit of assertive community treatment (ACT) is demonstrated by which situation? A. A client and family members attend counseling sessions together at a neighborhood clinic B. Implementation of a more flexible work schedule for staff C. Improved reimbursement for services provided in the community D. A client diagnosed with schizophrenia has avoided being rehospitalization for 16 months.

D A primary goal of ACT is working intensely with the patient in the community to prevent rehospitalization. The other options are not goals of ACT.

A nurse expresses an exclusive belief in the biological model for mental illness when stating "it's the only one I really believe." What conclusion should be drawn from this statement? A. The biological model is the oldest and most reliable model for explaining mental illness. B. The biological model has been proven to be successful in finding the cause of most symptoms of mental illness. C. The biological model is the most popular theory among leading psychiatrists and therefore the one that should be fully embraced. D. In believing only in the biological model, other influences on mental health including cultural, environmental, social, and spiritual influences are not taken into account.

D In believing only in the biological model to the exclusion of other theories and perspectives, influences such as educational, social, spiritual, cultural, environmental, and economic are not considered, and these have also been proven to play a part in mental health and mental illness. The other options are untrue.

A client, whose friend recently committed suicide, asks the nurse about some ways to help cope with the stress regarding the event. Which option should the nurse discuss with the client? A. Isolation for a short time so that the pain isn't reinforced by explaining her feelings over and over B. Antianxiety medication to help her relax C. Starting a hobby to keep her mind off the troubling event D. Talking with friends and attending a loss support group

D Social supports and support groups are two effective ways to cope with stress and stressful events. Isolation is never a healthy option; talking about feelings usually decreases stress, not increases. There is no evidence to suggest Melissa is anxious. Trying to "keep her mind off" the stressor does not develop coping mechanisms to deal with stress but rather encourages not dealing with the problem.

Which response should the nurse provide a client who asks, "Why you need to conduct an assessment interview"? A. "I need to find out more about you and the way you think in order to best help you." B. "The assessment interview lets you have an opportunity to express your feelings." C. "You are able to tell me in detail about your past so that we can determine why you are experiencing mental health alterations." D. "We will be able to form a relationship together where we can discuss the current problems and come up with goals and a plan for treatment."

D Some of the purposes of the assessment interview are to establish rapport, learn more about the presenting issues, and form mutual goals and a plan for treatment. The other options do not appropriately explain the assessment purpose.

Consider the nurse-patient relationship on an inpatient psychiatric unit. Which of the following statements made by the nurse reflects an accurate understanding of when the issue of termination should first be discussed? A. "You are being discharged today, so I'd like to bring up the subject of termination—discussing your time here and summarizing what coping skills you have attained." B. "I haven't met my new patient yet, but I am working through my feelings of anxiety in dealing with a patient who wanted to kill herself." C. "Now that we are working on your problem-solving skills and behaviors you'd like to change, I'd like to bring up the issue of termination." D. "Now that we've discussed your reasons for being here and how often we will meet, I'd like to talk about what we will do at the time of your discharge."

D The issue of termination is brought up first in the orientation phase. All the other options describe other phases of the nurse-patient relationship—the termination phase, the preorientation phase, and the working phase.

A client has been admitted to your inpatient psychiatric unit with suicidal ideation. In a one-to-one session with the nurse, he shares the terrible guilt he feels over sexually abusing his stepdaughter and wanting to die because of it. Which of the following responses you could make reflects a helpful trait in a therapeutic relationship? A. "It's good that you feel guilty. That means you still have a chance of being helped." B. "Of course you feel guilty. You did a horrendous thing. You shouldn't even forget what you did." C. "The biggest question is, will you do it again? You will end up having even worse guilt feelings because you hurt someone again." D. "You are suffering with guilt over what you did. Let's talk about some goals we could work on that may make you want to keep living."

D This response demonstrates suspending value judgment, a helpful trait in establishing and maintaining a therapeutic relationship. Although it is difficult, nurses are more effective when they don't use their own value systems to judge patients' thoughts, feelings, or behaviors. The other options are all judgmental responses. Judgment on the part of the nurse will most likely interfere with further explorations of feelings and hinder the therapeutic relationship.

A patient is presenting with behaviors that indicate anger. When approached, the patient states harshly, "I'm fine! Everything's great." Which response should the nurse provide to the patient? A. "Okay, but we are all here to help you, so come get one of the staff if you need to talk." B. "I'm glad everything is good. I am going to give you your schedule for the day and we can discuss how the groups are going." C. "I don't believe you. You are not being truthful with me." D. "It looks as though you are saying one thing but feeling another. Can you tell me what may be upsetting you?"

D This response uses the therapeutic technique of clarifying; it addresses the difference between the patient's verbal and nonverbal communication and encourages sharing of feelings. The other options do not address the patient's obvious distress or are confrontational and judgmental. None of the other options provides this support.

Which hospitalized patient should the nurse identify as being a candidate for the appropriate use of a release from hospitalization known as against medical advice (AMA)? A. A 37-year-old patient scheduled for discharge in 24 hours wishes to be discharged immediately B. A 75-year-old patient with dementia who demands to be allowed to go back to his own home C. A 21-year-old actively suicidal patient who wants to be discharged to home and do outpatient counseling D. A 32-year-old female patient who wishes to stay in the hospital but whose husband demands that she be discharged into his care

A AMS discharges are sometimes used when the patient does not agree with the provider, as long as the patient is not a danger to himself or herself or to others. The patient with dementia and the patient who is actively suicidal would pose a safety risk and would be not allowed to be discharged AMA. A patient who wishes to stay in the hospital can make that decision; a family member's opinion doesn't impact an AMA discharge.

A 29-year-old patient has been admitted following a suicide attempt. Which nursing statement illustrates the concept of patient advocacy? A. "Dr. Raye, I notice you ordered fluoxetine for this patient. During the admission interview, the client stated they will refuse the medication because of adverse effects they experienced previously." B. "Dr. Raye, during the admissions interview the patient stated that there is a family history of three other suicide attempts in the past." C. "I'd like you tell me more about your depression and your suicide attempt?" D. "I will take you on a tour of the unit and orient you to the rules so you can get adjusted here."

A By letting the provider know that the patient does not want the treatment the provider is prescribing, you have advocated for the patient and her right to make decisions regarding her treatment. The other selections do not describe patient advocacy since they do not represent actions by the nurse that the patient is incapable of on their own.

When preparing to hold an admission interview with a client, the nurse pulls up a chair and sits facing the client with his or her knees almost touching. When the nurse leans in close to speak, the client becomes visibly flustered and gets up and leaves the room. What is the most likely explanation for client's behavior? A. The nurse violated the client's personal space by physically being too close. B. The client has issues with sharing personal information. C. The nurse failed to explain the purpose of the admission interview. D. The client is responding to the voices by ending the conversation.

A By sitting and leaning in so closely, the nurse has entered into intimate space (0 to 18 inches), rather than social distance. This has likely made the patient may feel uncomfortable with being so close to someone unknown to them. All the other options lack evidence and jump to conclusions regarding the patient's behavior.

Which statement is true regarding incarcerated persons demonstrating behaviors associated with serious mental illness? A. Incarceration often causes decompensation in those with serious mental illness. B. Those with serious mental illness who are incarcerated see remission of symptoms while in prison. C. Incarceration doesn't appear to play a role in how a person with serious mental illness functions. D. Incarceration plays a role in that people with serious mental illness are ensured of receiving treatment they may have not had outside of prison.

A Correctional nurses provide care for many patients with serious mental illness. Because psychiatric facilities for the management of such emergencies are scarce, often these patients end up in jail instead of in a hospital. Once they are in jail, their psychiatric condition often worsens without adequate psychiatric intervention. The other options are not correct and incarceration may in fact prevent the inmate from receiving appropriate treatment.

Which of the following structural safety precautions is most important when attempting to prevent a common type of inpatient suicide? A. Break-away closet bars to prevent hanging B. Bedroom and dining areas with locked windows to prevent jumping C. Double-locked doors to prevent escaping from the unit D. Platform beds to prevent crush injuries

A Hangings are the most common method of inpatient suicide. The other options are important safety measures but don't directly address the suicide method of hanging.

Which statement by a forensic client supports the conclusion that he/she is legally sane? A. "I know what I did was wrong and it hurt a lot of people, but I had no money and had to figure out a way to get some." B. "I don't understand what I did that was so wrong." C. "I was responding to the voices in my head telling me to hurt those people." D. "I was raised in lots of foster homes and I never did learn about doing right."

A Legal sanity is defined as the individual's ability to distinguish right from wrong with reference to the act charged, capacity to understand the nature and quality of the act charged, and capacity to form the intent to commit the crime. The other options do not illustrate the requirements for legal sanity.

Which nursing intervention demonstrates the theory behind operant conditioning? A. Rewarding the client with a token for avoiding an argument with another client B. Showing the client how to be assertive without being aggressive C. Demonstrating deep breathing techniques to a group of clients D. Explaining to the client the consequences of not following unit rules

A Operant conditioning is the basis for behavior modification and uses positive reinforcement to increase desired behaviors. For example, when desired goals are achieved or behaviors are performed, patients might be rewarded with tokens. These tokens can be exchanged for food, small luxuries, or privileges. This reward system is known as a token economy. None of the remaining options demonstrate reward for positive behaviors, climate, and structure, for healing.

When considering mental illness, recovery is best described to a client by which statement? A. Working, living, and participating in the community B. Never having to visit a mental health provider again C. Being able to understand the nature of the diagnosed illness D. A period of time when signs and symptoms are being managed

A Recovery is described as the ability of the individual to work, live, and participate in the community. Never having to visit a mental health provider is unrealistic. While important to recovery understanding of the disorder is not a demonstration of recovery. Remission is a period of time when signs and symptoms are being managed.

A patient admitted with anxiety asks, "What exactly are stressors?" What is the nurse's best response to the patient's question? A. "Stressors are events that happen that threaten your current functioning and require you to adapt." B. "Stressors are complicated neuro stimuli that cause mental illness." C. "It's best if you ask questions like that of your provider for a complete answer." D. "Instead of focusing on what stressors are, let's explore your coping skills."

A Stressors are psychological or physical stimuli that are incompatible with current functioning and require adaptation. Stressors are not complicated neuro stimuli; telling the patient to address these questions to her provider fails to educate the patient, which is the nurse's responsibility. Exploring coping skills would be a good intervention at a later time but does not address the patient's question and changes the subject.

When considering the duty to warn and protect third parties, which client statement should the nurse report to the treatment team members? A. "That judge is going to really regret putting me in here." B. "All politicians need to be shot." C. "When I'm elected president, I'll make them all pay for doubting me." D. "The man out there who is laughing at me is going to die."

A The duty to protect is an ethical and legal obligation of health care workers to protect patients from physically harming themselves or others. This duty arises when the patient presents a serious danger to another. While all that statements infer the client's intention to harm, only the correct option is credible since it actually identifies the possible victim.

Which of the following statements are true regarding the differences between a social relationship and a therapeutic relationship? Select all that apply. A. In a social relationship, both parties' needs are met; in a therapeutic relationship, only the patient's needs are to be considered. B. A social relationship is instituted for the main purpose of exploring one member's feelings and issues; a therapeutic relationship is instituted for the purpose of friendship. C. Giving advice is done in social relationships; in therapeutic relationships giving advice is not usually therapeutic. D. In a social relationship, both parties come up with solutions to problems and solutions may be implemented by both (a friend may lend the other money, etc.); in a therapeutic relationship, solutions are discussed but are only implemented by the patient. E. In a social relationship, communication is usually deep and evaluated; in a therapeutic relationship, communication remains on a more superficial level, allowing patients to feel comfortable.

A, C, D The other options describe the opposite meanings of social and therapeutic relationships.

Which of the following statements represent a nontherapeutic communication technique? Select all that apply. A. "Why didn't you attend group this morning?" B. "From what you have said, you have great difficulty sleeping at night." C. "What did your boyfriend do that made you leave? Are you angry at him? Did he abuse you in some way?" D. "If I were you, I would quit the stressful job and find something else." E. "I'm really proud of you for the way you stood up to your brother when he visited today." F. "You mentioned that you have never had friends. Tell me more about that." G. "It sounds like you have been having a very hard time at home lately."

A, C, D, E All these options reflect the nontherapeutic techniques of (in order) asking "why" questions; using excessive questioning; giving advice; and giving approval. The other options describe therapeutic techniques of restating, exploring, and reflecting.

Match the group members comment to the role it most closely represents. 1. We arent getting much done; lets speed things up and make a decision. 2. Last week we finished our first goal and today we are starting on our second. 3. Everyone has had good ideas so far; so, whatever you say is fine with me. 4. It doesnt really matter what we do. Im just here because they make me attend. a. Playboy b. Energizer c. Organizer d. Follower

1. B 2. C 3. D 4. A


संबंधित स्टडी सेट्स

Strategic management: Q7 The resource-based view of the firm

View Set

Improving Vocabulary Skills Chapter 20

View Set

Unit 1. Electrical Test Equipment & Testing Components (SA)

View Set

Anatomy Unit 2: Skeletal System and Muscle Tissue

View Set

USMLE Step 1 Biochemistry: Genetics

View Set

Chapter 21 Microbiology Learnsmart

View Set

Combo with "Biology Exam #2" and 1 other

View Set

Chapter 17 Patho taken from http://thepoint.lww.com/Book LEVEL 3 MASTERY

View Set

Chapter 02: Research Questions, Hypotheses, and Clinical Questions

View Set

Chapter 2. ACC Learnsmart armoo72

View Set